Location via proxy:   [ UP ]  
[Report a bug]   [Manage cookies]                

Endocrinology - Internal Medicine, Dr. A. Mowafy (2020-2021)

Download as pdf or txt
Download as pdf or txt
You are on page 1of 172
At a glance
Powered by AI
The key takeaways are that the document discusses endocrinology and covers topics like the pituitary gland, thyroid gland, parathyroid gland, and suprarenal gland. It also provides guidance on studying different endocrine diseases.

The subject being covered is endocrinology, which is the study of glands and hormones in the body.

Some of the major topics discussed within endocrinology include diseases of the pituitary gland like acromegaly and hyperprolactinemia, diseases of the thyroid gland like hyperthyroidism and hypothyroidism, calcium metabolism and diseases of the parathyroid glands, and diseases of the adrenal glands like Cushing's syndrome.

• SERIES s MA RTE RN OT HARDER

EN DOCRI NO LOGY



In Capsule Series
Internal medicine

Smarter, not
Harder!"

P,ndocrino[ogy

Dr. Ahmed Mowafy


'· 1 ,,.:.•/a ~I \ ..,i; 4ill /~<', '1
·\ \J~I~~ '1 ~,, /.. ,,,:.f,<I ta \~ Jc.'a /.. ,',:.,< ta 1"'e""
0

U j ., ~ -.,I • .. _, • ~_,I i ..

'1'~ ~,l) tlli§


,. ~ -: ~1 c.s-
• (.J:t t:. 4=h;._ ~ 1 ~· ,1 u:Jc... ~ , '1~' ~, l) utbJJ .J ·i n,.. .., ,;
~ u~ts u'i_j;a d n.ujt, w~t, \:ic ~t, ~ w~Cb '1 ta ue;~,
-: .\.<ll e..,-,w
t..>J~ ·-~'I

~tint~~
(2864.:1~ WI~
{lJrejace

- First and foremost, thanks are due to ALLAH to whom I relate

any success in achieving any work in n:,y life.

- Great thanks to those who helped me, and greater thanks to those

who try to break my wings as they make me more able to fly.

- My thanks are extende d to all my medical student s whom I produce

this series 'Tn Capsule Series" to obtain a higher degree in internal

medicine by a simple effort.

- Finally, I wish to thank all members of My Family, my colleagues ,

my Friends and even all my patients for their continuous help,

encouragement and support.

Ahme d Mowa fy
INDEX
Subject Page
Endocrinology scheme 1

Pituitary gland
• Pituitary gland 3
• Acromegaly 6
• Hyperprolactinaemia
11
• Hypopituitarism in adults
14
18
• Diabetes insipidus
22
• SIADH

Thyroid gland
• Thyroid gland 23
• Thyroid diseases 25
• lry thyrotoxicosis 26
• Mxyedema 39
• Cretinism 44
• Goiter 45
• Thyroid cancer 46
Parathyroid gland
• Ca metabolism 47
• Hyperparathyroidism 48
• Hypoparathyroidism 53

Suprarenal gland
• Conn's syndrome 57
• Cushing's syndrome 60
• Adrenogenital syndrome 65
• Addison's disease 66
• Pheochromocytoma 72
• Steroid preparations 73

Diabetes mellitus 75
Hypoglycemia 109

Hirsutism & Gynaecomastia 111


Obesity 112
Puberty 115
Loss of weight 116
Multiple endocrine neoplasia {MEN ) 117
Polyglandular autoimmune syndromes 117
Oyslipidemia 119
Endocrinology Revision 126
MCQ 149
In Capsule Series Endocrinology

These points should be discussed in any endocri.nal disease:


1. Physiology.
2. Etiology.
3. Clinical picture.
4. Differential diagnosis.
5. Investigation.
6. Treatment.
1. Physiology : 4items

a. Hormone.
b. Gland.
c. Function of the hormone.
d. Regulation of this hormone.

2. Etiology :
a. Hyper:
1. Tumor : adenoma. ( benign > malignant )
ll. Hyperplasia.
m. Paramalignant syndrome.

b. Hypo
1. Surgery.
ii. Irradiation.
m. Tuberculosis, sarcoidosis, hemochromatosis. fl fl
3.c, P:
a. C / p of the cause : e.g. pressure manifestations , tumor ....
b. C / p of the hormone :
1. Hyperfunction = function of the hormone.

ii. Hypofunction = opposite the function of the hormone.


N. B. Neurological manifestations : depression &polyneuropathy- may
occur in any endocrinal disease.

1
In Capsule Series Endocrinology
4. Differential diagnosis : Differs according to each hormone.

5. Investigation :
a. Inv. for the cause : e.g. Imaging for the gland : U/S , CT , MRI .
b. Assay of the hormone level :
i. In blood : j in hyper , t in hypo
• 2 - 3 times to avoid the diurnal variation.
• It's not an ideal method because the plasma level of all hormones
varies through the day ( because of pulsatile secretions ).
11. In urine:
1. hormone.
2. metabolite of the hormone.

c. Investigations of the function of hormone : see function .


d. Dynamic test : The principle is :

• If a hormone level is high - suppress it.


• If a hormone level is low - stimulate it.

o Suppression test - for Hyperfunction.


- Normal : +ve ( suppression of the hormone)
- In this case( ...... ) : - ve (no suppression).

o Stimulatory test - for Hypofunction.


- Normal : +ve ( sti.mulati.on of the hormone)
- In this case (. ..... ) : - ve ( no sti.mulati.on ).
6. Treatment :
a. Hyper: ~

1. Surgery.
11. Irradiation.
111. Medical antagonist.
b . Hypo :
1. Replacement therapy.
11. Treatment of the cause.

2
In Capsule Series Endocrinology

• Present in Sella turcica at the base of the skull.


• Weight 0.5 gm.

Cerebrum

Pineal gland

Optic nerve

.
• It is divided into 2 lobes :
i. Anterior lobe.
ii. Posterior lobe.

Optic chlasma

Anterior lobe
(adenohypophysls):
Pars tuberalls- -----·, ,.........--lnfundlbulum
Pars dlstalls

Posterior lobe
(neurohypophysls)

- Anterior lobe - connected to the hypothalamus by vascular connection


- Posterior lobe - connected to the hypothalamus by neural connection.

3
In Capsule Series Endocrinology
• anterior lobe :
- Chromophobes : non functioning ( non secreting ) .
- Chromophils :
• acidophil : GH & prolactin.
• basophile : ACTH, TSH, FSH, LH & MSH.

• Posterior lobe : secretes ADH & Oxytocin.


In fact, ADH & oxytocin are synthesized in the hypothalamus & stored in the
posterior pituitary.

• Regulation of pituitary hormones :

1. hypothalamus secretes :
► Releasing factors : GHRH , TRH , CRH , GnRH
- GHRH ( GH releasing hormone) - t the secretion of GH.
-TRH (thyrotropin-releasing hormone) - t the secretion ofTSH & Prolactin?
- CRH ( corticotropin releasing hormone ) - t the secretion of ACTH
- GnRH ( gonadotropin releasing hormone ) ---t t the secretion of LH , FSH

► Release inhibiting factors : ! secretion of pituitary hormones.

ii. Secretion of hormones is also regulated by -ve feedback mechanism :


e.g. i fa-.! TSH j cortisol-➔ ! ACTH.

• Function of the pituitary hormones :


- GH
• CHO : hyperglycemia.
- Decreases glucose uptake by the cells.
- Increases the insulin resistance.
• Fat : lipolysis.
• Protein: anabolic (bone, muscle & viscera)
• Minerals: j ( Na, K, Ca & P04).

Prolactin milk secretion.

4
In Capsule Series Endocrinology
FSH

. ~ --+ maturation of graffian follicle.

• 0 --+ spermatogenesis.

LH
. ~ --+ ovulation.

• 0 --+ testosterone formation by the testis.

ACTH
• Secretion of suprarenal hormones :
o cortisone +++
o androgen ++
o aldosterone + ( under stress only)

TSH Secretion of thyroxin by thyroid gland.


MSH Skin pigmentation.

Hypothalamus

Blood
v e a e e l - - - - - -~~'lllll
ReleHlng hormones
from hypothalamus-===;;.,_.,,_.~~ •
Posterior
pituitary - - .!'.---c:'l

Anterior
Blood pituitary
vessel
Pituitary hormonea

(
I TSH I
l l l IEndorphinsI
lo•,r•• I !ACTH !
[ru Growth
hormone
Prolactln
(PAL)

Uterine muscles Kidney t ubules


! !
Thyroid
cortex
+
Adrenal Teatea or
ovaries
(GH)

+
Entire
body
i
Mammary
glands
!
Pain
receptors
Mammary glands (In mammals) In the brain

5
In Capsule Series Endocrinology

Acromegaly
Excessive secretion of GH in adults ( after fusion of the epiphysis )

Physiology :
a. Hormone Growth hormone ( GH )

b. Gland Pituitary gland ( anterior lobe )

c. Function of the hormone


i. CHO hyperglycemia.
ii. Fat lipolysis.
iii. Protein anabolic (bone, muscle & viscera).

iv. Minerals : j ( Na, K, Ca & P04).


d. Regulation of this hormone
I. i GH:
1. GHRF. ( growth hormone releasing factor)
2. Hypoglycemia.
3. Sleep.
4. After arginine infusion.
ii. ! GH
1. GHRIF( growth hormone release inhibiting factor)
2. Hyperglycemia.
3. Stress & exercise.

2. Etiology :
a. Pituitary adenoma. ( may secrete GH only, or co-secrete GH
& prolactin ). Rarely, carcinoma.
b. Paraneoplastic syndrome. ( e.g. bronchogenic carcinoma)
c. Multiple endocrine neoplasia type l syndrome ( MEN l ) :
Tumors in Pituitary, Parathyroid & Pancreas. ( 3 Ps)
d. Excess GHRH: rare e.g. hypothalamic hamartoma.

6
In Capsule Series Endocrinology
3.C IP:
a. C I p of the cause :
Pressure manifestations of pituitary adenoma : visual field defect.
b. C I p of the hormone :

i. CHO OM in 25 % of cases.
Ii. Fat excess lipolysis ---+ loss of sc fat , wrinkling of the skin.
iii. Protein excess growth of bone, muscles, viscera.
1. bone:
a. Skull : Periosteal new bone formation in
the skull & mandible results in :
i. Frontal bossing, maxillary overgrowth,
nasal broadening.
ii. Prominent supra-orbital ridges &
mastoid processes.
iii. Prognathism : prominent lower jaw
with separated teeth.
iv. Hypertrophy of larynx & tongue :
hollow deep voice & sleep apnea.
These changes result in coarsening offacial features that are noticed when
old patient pictures are seen.

b. Hands & feet:


i. Spade hands & enlargement of feet.
ii. Frequent change in rings & shoes size.
111. Hypertrophy of joint cartilage & osseous overgrowth result in
carpal tunnel syndrome & osteoarthritis.
2. Visceromegaly :
a. Hepatosplenomegaly.
b. Cardiomegaly---+ congestive heart failure.

7
In Capsule Series Endocrinology
c. Colonic polyps & increased risk of colonic cancer.
d. Endothelial hyperplasia~ hypertension.
e. Hypertrophy of the skin, sweat & sebaceous glands resulting in
moist greasy skin.
3. Muscle power :
a. ii early. !LH Late. ( needless to say why ? © )
iv. Minerals :
a. i Na ~ hypertension.
b. j K ~ muscle weakness, arrhythmias.
v. Neurological manifestations:
Causes of carpal tunnel
a . Depression. syndrome : Just remember
DR Ahmed Mowafy ©
b. Carpal tunnel syndrome.
D:DM
c. Peripheral neuropathy. R: Rheumatoid arthritis
d. Spinal cord compression. A : Acromegaly
M :Myxedema & Musicians.
e. Proximal myopathy.
t Pressure manifestations of the tumor in the skull.

vi. Associated endocrinal manifestation :


• DM in 25 % of cases. ( GH is an anti-insulin hormone ) .
• Hyperprolactinemia due to :
- Co secretion from acidophils. (25% of pituitary adenomas)
- Pituitary stalk compression ~ decrease of hypothalamic
prolactin release inhibitory factor (PIF).
• Deficiency of other anterior pituitary hormones due to compression
of the anterior pituitary.
• Hypogonadism if associated with Hyperprolactinemia or
decreased pituitary gonadotropins.
• Rarely, diabetes insipidus : due to compression of the posterior
pituitary.

8
In Capsule Series Endocrinology

4. Investigation:
a. Investigations for the cause : Imaging for the gland : Skull X-ray, CT & MRI
b. Assay of the hormone level :
1. Fasting GH : j [normal< 5 n.gm/ml]
2. j Insulin growth factor-1 ( IGF-1 , somatomedin )
3. Hyperprolactinemia & suppression of other anterior pituitary
hormones may occur.
GH is difficult to measure because ofits pulsatile secretions So , insulin growth
factor ( IGF-1) produced by the liver in response to GH is measured instead.

c. Investigations of the function of the hormone :


i. Blood : j glucose, Na, K, Ca & P. What laboratory tests should
Ii. X-ray : be performed initially?

1. Skull: IGF-1 confirmed by glucose


suppression test.
• Thick cortex.
• Widening of sella turcica & nasal sinuses.
• Prominent supraorbital ridges & occipital protuberance.
• Prognathism with wide separation of teeth.
2. Hand : tufting of the terminal phalanges ( mushroom shape)

d. Suppression test : glucose IV ( oral glucose tolerance test)


1. Normally -+ +ve ( ! GH level )
2. In acromegaly - -ve ( no suppression ) .

After overnight fasting, 50 -100 g oforal glucose causes suppression ofGH over the
subsequent 30 - 90 minutes to < 2 ng/ml in normal persons but remain above 2 ng/ml in
acromegalic patients.

e. Others :
Echocardiography and colonoscopy should be performed to
evaluate for cardiomegaly and colon polyps.

9
In Capsule Series Endocrinology

5. Treatment :
a. Surgery : hypophysectomy. ( The preferred initial treatment)
b. Radiotherapy : if GH is still elevated after surgery or if complete
excision of the tumor is not possible or in a case of recurrence.
c. Medical treatment :
& SC Somatostatin analogues [GHRIF] : e.g.
✓ Octereotide.
✓ Lanreotide, Pasireotide : newer somatostatin analogues.
l'5. GH receptor antagonists : Pegvisomant.
& Dopamine agonists : less effective than octreotide. A large
dose is needed with increased incidence of side effects.
l'5. Symptomatic treatment : DM , Hypertension.

II Gigantism II
Definition : ii GH secretion before fusion of epiphysis.

1. Generalized overgrowth of skeleton & soft tissue.


2. Disproportionate gigantism. ( span more than height) .
3. Later on features of acromegaly develops.
4. Lastly decline stage usually occurs ( pituitary hypofunction)

IDifferential diagnosis oftall stature~


1. Familial & racial : [ the commonest ]
2. Gigantism.
3. Excess androgen --+ tall children , short adult
4. 1ry hypogonadism : Disproportionate gigantism with feminine character.
5. Marfan & Klinefelter syndromes.

10
In Capsule Series Endocrinology

Hyperprolactinemia
1. Physiology :
a. Hormone : Prolactin.
b. Gland : Pituitary gland [ anterior lobe, prolactin acidophils ] .
~ Function of the hormone : milk secretion.
d. Regulation of this hormone :
i. l prolactin : Recently it's thought that TRH ( thyrotropin releasing
hormone ) is a prolactin releasing hormone.
ii. ! prolactin : prolactin release inhibiting hormone ( PRIH, dopamine)

2. Etiology :
a. Physiological :
o Pregnancy, Breast feeding, Breast stimulation.
o Sleep, Stress, Suckling.
b. Drugs : Antidopaminergic drugs MCQ
i. Antipsychotics: Phenothiazine (chlorpromazine), Haloperidol.
ii. Antidepressant :
o Tricyclic Antidepressants : Amitriptyline.

o Selective Serotonin Reuptake Inhibitors (SSRI) : Paroxetine.

iii. Antiemetic : Metoclopramide, Domperidone.


iv. Antihypertensive: a-Methyl-dopa, reserpine, verapamil.
v. Others: Morphine, Estrogen, Cimetidine.
c. Pathological :
i. Pituitary : Prolactinoma ( prolactin secreting pituitary adenoma ),
Mixed pituitary adenomas : GH/Prolactin
ii. Hypothalamic tumors.
iii. Thyroid : l ry hypothyroidism -l TRH- l prolactin.

11
In Capsule Series Endocrinology
iv. Renal : CRF -. l prolactin clearance.
v. Liver cirrhosis : -. J metabolism.

3. Clinical picture :
a. CI p of the cause: visual disturbance due to pituitary adenoma.
b. CI p of the hormone:
i. In female :
1. Glactorrhea : persistent milk production in a ~ who is not postpartum.
2. Amenorrhea.
3. Osteoprosis ( due to estrogen deficiency ) .
ii. In male :
l. Gynecomastia.
Hyperprolactinemia suppresses
2. Impotence & t libido. GnRH causing decreased FSH & LH

3. Infertility.

4. Investigation:
a. Investigations for the cause : Pituitary imaging: X ray, CT, MRI.
b. Assay of prolactin level:
Prolactin > 250 ng/ml suggest probable pituitary adenoma. [ n < 20 ng/ml J

5. Treatment :
a. Dopamine agonist:
i. Bromocriptine: 2.5 mg orally, j gradually to 10 mg/d.
ii. Cabergoline : drug of choice : less side effects.
o Effect : Lower prolactin level with reduction of the tumor size
in about 70-90%
o S/E : nausea, vomiting, abdominal pain, hallucinations.

b. Surgery: for macroadenoma not responsive to medical treatment.


c. Irradiation: proton or a particles.
d. Treatment of the cause: e.g. myxoedema.

12
In Capsule Series Endocrinology
Pituitary tumors

[ umor Ty~ ISecretory Product(s) IRelative Frequency (%)


IProlactinoma (Lactotroph Adenoma) IProlactin I 50

1somatotroph Adenoma !Growth Hormone I 10

1corticotroph Adenoma IACTH I 5

IThyrotroph Adenoma ITSH I 1


INon secreting A~enoma 1----- I 34

Nelson's syndrome :
Big pituitary adenoma with very high ACTH & hyperpigmentation after
bilateral adrnalectomy in a case of pituitary Cushing.

Pituitary apoplexy : It is an endocrinology emergency


Sudden enlargement of the pituitary by hemorrhage into the tumor,
causing combination of pressure manifestations ( e.g. visual defec t) &
panhypopituitarism.

Try to learn something about everything and every thing about something.

Thomas Henry Huxley

13
In Capsule Series Endocrinology

I Panhypopituitarism in adults
( Loss ofanterior pituitary function )
11

1 . Etiology :
a. Destruction of pituitary gland by:
i. Surgery.
ii. Irradiation.
111. Tuberculosis, sarcoidosis, hemochromatosis. JJ JJ
b . Sheehan syndrome : pituitary infarction after sever postpartum hemorrhage
c. Pituitary apoplexy : Hemorrhage in pituitary tumor.
d. Genetic: Isolated GH or GnRH deficiency.

e. Autoimmune.

2.C IP:
a. C / p of the cause: Surgery, Sever post-partum hemorrhage.

b . C / p of the hormones :
According to the frequency of appearance : 2 G - t GH & Gondotrophins ( FSH , LH)
are lost early then TSH then ACTH deficiency.

1. Gonadal deficiency :
1. ~ : amenorrhea, ! libido, infertility, loss of pubic & axillary hair.

2. c3 : impotence,! libido, infertility, loss of pubic & axillary hair.


ii. GH deficiency :
a) Adult: Fasting hypoglycemia, osteoporosis & fine wrinkles.
b) In Children :
• GH deficiency only : short stature with proportionate
dwarfism.
• GH with FSH & LH deficiency : dwarfism with infantile
proportions ( upper> lower segment & span shorter
than height.

14
In Capsule Series Endocrinology
111 . TSH deficiency : result in 2ry hypothyroidism. In contrast to 1ry
hypothyroidism hypercholesterolemia is rare. Skin is thin with
wrinkling. Also, no menorrhagia in 2ry hypothyroidism. MCQ
iv. ACTH deficiency : Results in 2ry adrenal insufficiency, it simulates
Addison's disease but :
1. No pigmentation ( absent ACTH )
2. No marked hypotension , due to normal aldosterone.
v. Prolactin : Low with failure of lactation in Sheehan syndrome. It
may be high in lesions causing pituitary stalk compression.
vi. Coma : due to
1. Hypoglycemic coma due tot GH & cortisone.
2. Myxedema coma.
3. Pressure in cases of pituitary tumors.
NB : If the patient has associated DI: the problem is at the hypothalamus.

3. Differential diagnosis :
a . From 1ry Addison's disease & 1ry hypothyroidism : see later
b . From anorexia nervosa :
1. Normal hair & breast. ii. Aggressive attitude.
111. Normal cortisol & High GH due to hypoglycemia.

4. Investigation :
a . Investigations for the cause: Pituitary imaging : X-ray, CT & MRI
b. Assay of the hormones level :
1. ! GH [ n : 1 - s n.gm/ml] & ! IGF-1.
2. ! FSH, ! LH & ! sex hormones.
3. ! TSH, ! T3 & ! T4.
4. ! ACTH & ! cortisol.
5. Prolactin : Low or high (see above)

15
In Capsule Series Endocrinology
c. Investigations of the function of the hormone : e.g.
l. Hypoglycemia.
2. Hyponatremia.
d. Stimulatory test : insulin tolerance test
• It is a gold standard for diagnosis of GH deficiency.
• After overnight fasting, regular insulin (O. l U/Kg) is given :
l. Normal : +ve ( j GH > 7 ng/ml )
2. In pan-hypopituitarism : -ve ( no stimulation ) .
5. Treatment :
a. Treatment of the cause.
b. Replacement therapy: Multiple hormones must be replaced.
i. Hydrocortisone : 30 mg/ d. ( 20 mg a.m. & 10 mg p.m. )
11. Gonadotrophin deficiency
If fertility is not a problem :
& In males: testosterone.
& In females: estrogen/ progesterone.
If fertility is a problem : ( both males & females )
& LH : Human chorionic gonadotropin ( hCG ) .
& FSH : human menopausal gonadotropin.
& GnRH : may be occasionally used.
iii . TSH deficiency: Oral Levo-thyroxine, should be started after
steroid replacement, to avoid acute adrenal failure.
1v. ADH deficiency: desmopressin ( intranasal, SC, oral )
v. Recently :
l. Purified pituitary hormones.
2. Hypothalamic releasing factors.

16
In Capsule Series Endocrinology

Differential diagnosis ofshort stature I


1. Familial & constitutional : the commonest.

2. Endocrinal :
a. ! GH : Levi-lorain , Frolich's, Laurance Moon Biedle *
b. ! T4 : cretinism & juvenile myxedema.
c. i sex hormones : precocious puberty.
d. i cortisol : Cushing or excess cortisone therapy.
e. Type1DM.

* Levi-Lorain syndrome : proportionate dwarfism , hypogonadism , normal mentality.


* Frolich 's syndrome : trunk obesity , hypogonadism , polyphagia & mental retardation.

* Laurance moon biedle: like Frolich's + Skull deformity & retinitis pigmentosa.

3. Chronic sever illness during childhood :


a. CVS : sever rheumatic fever, congenital heart diseases.
b. Chest : polycystic lung disease, TB.
c. GIT : malabsorption syndrome & malnutrition.
d. Kidney : chronic nephritis.
4. Skeletal causes
a. Congenital :
i. Achondroplasia : short limbs with normal trunk.
ii. Osteochondrodystropby : both limbs & trunk are short & deformed.
b. Acquired :
1. Rickets.
11. Pott's disease of spine.
m. Paget's disease of bone.

5. Genetic causes
a. Mongolism [Down's syndrome] trisomy 21.
b. Turner's syndrome [ 45+ XO].
c. Noonan's syndrome.

17
In CapsLJ!e Series Endocrinology

Diabetes insipidus
Deficiency of ADH

1. Physiology :
a. Hormone: ADH ( antidiuretic hormone , vasopressin ).
b. Gland : Pituitary gland ( posterior lobe ).
ADH is synthesized in hypothalamus & then transported along axons & stored in
the posterior Pituitary.

c. Function of the hormone :


i. ii water reabsorption by collecting tubules.
ii. Large dose ~ vasoconstriction of blood vessels.

d. Regulation of this hormone :


i. it ADH by :
1. j osmolality.
2. Hypovolemia & hypotension.
3. Drugs : nicotine , barbiturates.
11. H ADH by : l osmolality, Hypertension & Cold weather.
2. Etiology :
a . Central DI : { damage of hypothalmo-hypophyseal axis)
1. Idiopathic : Hereditary ( autosomal dominant)

ii. Injury: after hypophysectomy.

iii. Infection : T.B.


1v. Infiltration : sarcoidosis

v. Infarction: Sheehan syndrome.


vi. pituitary tumor.
vii. Pregnancy : deficiency of ADH can result from increased
metabolism by a placental enzyme (vasopressinase
enzyme). It resolves within a few weeks after delivery.

18
In Capsule Series Endocrinology
b. Familial DI ( Wolfram syndrome, DIDMOAD)
hereditary condition in which there is a defect in osmo-receptors
i. Diabetes lnsipidus. ..,
Osmoreceptors are present in
ii. Diabetes Mellitus. anterior hypothalamus.

iii. Optic atrophy.

iv. Deafness.

c . Nephrogenic DI ( Renal tubules not responding to ADH )


i. Hereditary ( X-linked recessive so, males are predominantly affected ®)

ii. Acquired MCQ


1. Chronic renal failure.
2. Kidney amyloidosis.
3. Polycystic kidney & Pyelonephritis.
4. Hypercalcemia & Hypokalemia.
5. Drugs : Lithium , Cholchicine, Demeclocycline.

3.C / P:
a. CI p of the cause: e.g. history of hypophysectomy.
b. C / p of the hormone :
i. Polyuria ( up to 20 L/day) & polydepsia.
ii. Severe dehydration -+ weakness , weight loss & fever.
111. Hypovitamonosis : of water soluble vitamins.
iv. Complications: shock & death.

4. Differential diagnosis : DD of polyuria. [ discussed later]

5. Investigation :
a. Investigations for the cause: Pituitary Imaging: X-ray , CT & MRI
b. Assay of the hormone level: J.

19
In Capsule Series Endocrinology
c. Investigations for the function of ADH :
1. Urine analysis :
1. Polyuria with low Specific gravity.
2. No pathological constitutes.
3. After fluid deprivation : Polyuria persists.
11. Plasma osmolality : i due to loss of free water.

d. Stimulation tests :
1. Test the hypothalamus: nicotine test ( 1-3 mg nicotine)
1. normal : oliguria due to stimulation of ADH.
2. central DI : -ve ( no stimulation )
ii. Test osmo-receptrs : IV hypertonic saline ( NaCl 2.5 % ) .
1. normal : oliguria.
2. osmo-receptrs defects : -ve ( no olguria )
iii. Test kidney ( vasopressin test ) :
to differentiate between central DI & nephrogenic DI
1. in Central DI : oliguria ( Concentrated urine )

2. in Nephrogenic DI : No change.

6. Treatment :
a. Replacement therapy: synthetic ADH (Desmopressine) 20 µg/d intra nasal

b. Diet : ! Na , t vitamins.
c . Drugs : 3 C © MCO
& Cholropropamide : for central DI.
"& Carbemazepine : in nephrogenic DI.
& Thiazide may be used in nephrogenic DI.
'& Amiloride (potassium-sparing diuretic) : is the drug of choice in lithium
toxicity induced DI.

20
In Capsule Series Endocrinology

Differential diagnosis ofpolyuria


Urine output> 3L/d in adult

1. Physiological :
► Winter months.
► Excess coffee or tea.
► Diuretics.
2. Psychological : Hysterical polydepsia.

DI Hysterical polydepsia
C/P: Polyuria then polydepsia Polydepsia then polyuria
Fluid deprivation test : Polyuria persists No polyuria
Osmolality ii H

3. Pathological
a. Endocrinal :
i. DI : with its causes.
ii. DM
iii. Adrenal : Conn 's (hypokalemia), Addison's.
iv. Thyroid : thyrotoxicosis ( j metabolic water)
v. Parathyroid : hyperparathyroidism.

b. Renal
i. Nephrogenic DI.
ii. Chronic renal failure.
iii. Diuretic phase of acute renal failure.
c. After any attack
i. Migraine
ii. Epilepsy
iii. Bronchial asthma
iv. Also after the internal medicine exam. Q

21
In Capsule Series Endocrinology

Syndrome ofinappropriate ADH secretion


(SIADH)
It means an excessive release of ADH

1. Physiology : Refer to diabetes insipidus.

2. Etiology:
a. Tumors release ADH : Oat cell carcinoma & lymphoma.
b. Pulmonary lesion : Legionella pneumonia, TB.
c. CNS : meningitis, encephalitis & head injuries.
d. Drugs : cholropropamide, carbamazepine, cyclophosphamide.

• Increased ADH causes water retention ~ dilutional hyponatremia


(weight gain , weakness , confusion, convulsions & coma )
• No edema because of natriuresis.

4. Investigation:
1. J Na (< 130 mEq /L)
2. J serum osmolality (< 270 mosmol/kg) .
3. j urine Na concentration ( > 20 mEq/L ).

5. Treatment :
a. Fluid restriction~ 0.8 -1 L / day.
b. Demeclocycline:
i. 600 -1200 mg /day~ inhibit the action of ADH.
11. Given to patient unresponsive to fluid restriction.
c. For sever hyponatremia : hypertonic saline ( 5% ) IV slowly.

22
In Capsule Series Endocrinology

Anatomy:
• The thyroid gland is an H shaped or butterfly

shaped gland located in the lower neck in front of

the trachea and between the carotid arteries.

• It is formed of 2 lobes connected to each other by

an isthmus. Its normal weight is about 10 - 25 gm.

• It's attached to the thyroid cartilage & to the upper

end of trachea so , it moves with swallowing.

Biosynthesis of thyroid hormones : (Trapping -+ Binding -+ Coupling ➔ Release ) ©

1. Iodine trapping : Iodine uptake by the thyroid gland .

2. Binding Tyrosine bind to iodine to form mono & di-iodotyrosine.

3. Coupling

• 2 molecules of di-iodotyrosine ---+ T4 ( Thyroxin) : 80 mcg per day

• Mono+ di-iodotyrosine ---+ T3 ( Tri iodothyronine): 4 mcg per day

• T4 is converted to T3 in the peripheral tissue.

• 20% ofT3 is produced by the thyroid gland, 80% ofT3 is a result of break

down of T4 ( monodeionation of the outer ring ofT4 mainly in the liver).

23
In Capsule Series Endocrinology
4. Release

• T3 & T4 release under the control of TSH.

• Thyroid hormones circulate in 2 forms :

✓ Protein bound, mainly thyroxin binding globulin ( TBG) : > 99 %

✓ Free part ( active part ) :

o 0.03 % of total serum T4 is free.

o 0.3 % of total serum T3 is free

• T3 is 3 - 8 more potent than T4 and acts more rapidly.

• The halflife ofT3 is~ 24 hours.

• The half life of T4 is about one week.

Follicle colloid Endothelium


Thyroid fol/lculsr cell ........
. :-:·:•:.
. Blood
..."'..: ··•·.

,, •
O~Ellaop;~smic
reticulum
Thyroglobulin secretion
Pendrin

◄ ■--~--------~--
C I" •. . J
10 • Oxidation

Proteolysis
I
I
/If~ :O--P-·
- Trilodothyronine

24
In Capsule Series Endocrinology

Thyroid diseases

Causes of hyperthyroidism ( thyrotoxicosis ) :


1. Toxic diffuse goiter : The most common cause of hyperthyroidism.
( Grave's disease in USA, Basedow's disease in Europe).

2. Toxic multinodular goiter.


3. Toxic solitary adenoma.
4. Excess production of TSH by pituitary tumor ( very rare ).
5. Hashimoto's thyroiditis with transient hyperthyroid phase.
6. Extrathyroid source of hormone :
a. Thyrotoxicosis factiscia : exogenous intake of thyroxin.
b. Ectopic thyroid tissue e.g. strauma ovarii.
7. Iatrogenic: amiodarone.

Classification of hypothyroidism :
► According to the age of onset :
1. Cretinism during infancy.
2. Juvenile myxedema before puberty.
3. Myxedema after puberty.
► According to the site of the cause :
1. 1ry hypothyroidism : cause in the thyroid gland.
2. 2ry hypothyroidism : cause in the pituitary gland.
3. 3ry hypothyroidism : cause in the hypothalamus.

25
In Capsule Series Endocrinology

1 ry thyrotoxicosis
(Graves' disease)
- It is the most common cause of hyperthyroidism.

- Autoimmune disorder consists of :

• Hyperthyroidism plus one or more of the following triad :


• Painless diffuse goiter ( 90 % )
• Ophthalmopathy ( 60 % )
• Dermopathy. ( 2 % )

1. Physiology :
a. Hormone : T3 (triiodothyronine) & T4 (thyroxine)

b. Gland : Thyroid gland.


c. Function of the hormone: Hormone of metabolism
i. Stimulate physical, mental & sexual growth.
ii. i energy production & 02 consumption.
iii. j glucose absorption & uptake by cells.
1v. ! cholesterol & prolactin levels.
v. i the respiratory & heart rate.
vi. Stimulation of erythropoiesis ( j RB Cs ) .
vii. j the tissue responsiveness to catecholamine.
viii. Hepatic conversion of carotene to vitamin A .

d. Regulation of this hormone :


i. TSH.

ii. TRH.

26
In Capsule Series Endocrinology
2. Etiology :
- Autoimmune disorder characterized by a variety of circulating
antibodies( Thyroid stimulating immunoglobulin - TSI ) that bind to and
activate the thyroid TSH receptors.
- May be associated with other autoimmune disorders e .g. autoimmune
polyglandular syndrome II ( see later )

C,cnctlc c l o ~ ?
lack of TSH receptor antibodies
suppressor bind to TSH receptors In
Tcclls retro-orbital connective tissue

Bcells
~~ TSH produce
TSH receptor

I 0
antibodies
T cells
produce
inflammatory
,/
I
cytoklnes
Pituitary
,end , .
, I

D
I
-- t Glycosam noglycans

I
Eye muscle
antibodies'\

Swelling In muscle
and connective tl$Sues
behind eyes

Ophthalmopathy
I
Clinical p!'ftffltation
of hypmhyroldism

27
In Capsule Series Endocrinology

3. Clinical picture : (~ 30 - 40 years ) .


a. C I p of the cause :
Association of autoimmune diseases : Polyglandular autoimmune
syndrome II ( Typel DM, Adrenal failure & Grave's disease).
b. C / p of the hormone :
!General :I

o Polyphagia with loss of weight.


o Intolerance to hot weather.
o Progressive muscle weakness.
o Bone : pain, pathological fracture due to bone Ca turnover.

JGIT :I
o ii appetite ( with loss of weight ).
o Diarrhea .
o Just palpable spleen.

!G enital :I
o ~ : Amenorrhea.

o 3 : Impotence & gynecomastia.

Jcutaneous :I
o Skin is thin ,warm & flushed with generalized sweating.
o Hair : fine & thin.
o Nails : clubbing of fingers. ( Thyroid acropachy )
o Preitibial mvxoedema : ( dermopathy )
Irregular, Itchy, non- pitting, tender & hairy swelling over the

chin of the tibia. It is usually symmetrical, due to deposition


of mucopolysaccaride substance in the S.C. tissue.

28
In Capsule Series Endocrinology

lcvs :I (more common in older patients)

1- pulse :
o rate : j , the sleeping pulse rate is usually > 100 b/m .
o rhythm : may be irregular due to AF or extra systole.
o character : water hummer pulse due to big pulse volume.
o equality : unequal on both sides - retro-sternal goiter.
Thyroid junction tests are mandatory in any patient with atrial.fibrillation

2- heart : Hyperdynamic circulation ¢ volume overload ¢ cardiac


enlargement then heart failure.

3- Blood Pressure :
o systolic j ( isolated systolic hypertension )
o diastolic t ( thyroxin - VD )
So, there is big pulse volume

lcNs :I ( more common in younger patients).

- Psychic: 4 mnemonic : naim


o Nervousness.
o Anxiety & agitation.
o Insomnia.
o Mental disturbance.
- Organic: 4
o Fine tremors in tongue & hands.
o Polyneuropathy.
o Myopathy.
o Myasthenia.
It's important to note that elderly patients with hyperthyroidism may be apathic rather
than nervous ( apathetic thyrotoxicosis ).

29
In Capsule Series Endocrinology

\Eye:\
1. Exophthalmos :
o Bilateral , but may start unilateral.
o May occur even before the appearance of thyrotoxicosis.
o AE : oversecretion of autoantibody called exophthalmos
producing substance (EPS) ~ lymphocytic infiltration of
retrobulbar tissue & extrinsic muscles.
o Orbital pain & photophobia may occur.
o Blindness may occur secondary to corneal ulcers or optic nerve
compression.
2. certain eve signs : ( DR must be very simple )
a. Dalrymple's sign ( Lid retraction) : rim of sclera is seen between the
cornea & upper lid.
b. Rosenbach's sign: fine tremors of eyelids on slight closure of eye.
c. Moebius' sign : lack of convergence due to weak medial recti muscles.
d. Von-Grave's sign: lid lag on looking down.
e. Stellwag's sign: infrequent blinking.

N.B: Ocular manifestations can be divided into 2 types :


✓ Infiltrative ophthalmopathy (mechanical , due to accumulation of
autoantibodies) : specific to Grave's disease
o Exophthalmos
o Mobius sign

✓ Non- Infiltrative ophthalmopathy ( due to j thyroxin) :


e.g. lid lag, lid retraction, stellwag & Rosenbach's sign ---+ occur with any
thyrotoxicosis (not specific to Grave's disease).

30
In Capsule Series Endocrinology

!Gland :I
a. Inspection : mild to moderate enlargement of the gland.
b. Palpation : - consistency : fleshy or firm. - surface : smooth.
c. Percussion : dullness over manubirum-sterni in retro-sternal extension.
d. Auscultation : systolic bruit may be heard due to high vascularity.

fj'hyrotoxic crisisl { thyroid storm) :

o An extreme form of thyrotoxicosis with a mortality of l O%.


o It's rare, life threatening endocrinal emergency.
o Precipitating factors :
- Surgery. - Acute illness : Ml, stroke, infection.
- Parturition - Discontinuation of anti-thyroid drugs
o C/P : sever manifestations of thyrotoxicosis that include :
a) General : fever with temperature > 41 °C, marked weakness
b) C.V.S. : sever tachycardia, hypotension, acute HF & AF.
c) C.N.S. : tremors, marked irritability, confusion & coma.
d) Diarrhea, abdominal pain, hepatomegaly with mild jaundice.
o Treatment : see later

Differential diagnosis :
a. Neurosis : --------+ cold hand & normal sleeping pulse.
b. jappetite with loss of weight: DM , parasitic infection.
c. Hyperdynamic circulation.
d. Muscle diseases: myopathies, myasthenia gravis.
e. Difference between 1ry & 2ry thyrotoxicosis

31
In Capsule Series Endocrinology
- ··~- ~- 1.'11 thyrotorl.cosis :zri, thyrotorl.cosis

- Other name : Grave's disease. toxic multi nodular goiter.

- Cause : Auto-immune. On top of nodular goiter.

- Age of onset : 30-50 year. 50 year.

- Thyroid gland : Diffusely enlarged. Nodular.

- CVS: Mild. Sever.

- CNS: Sever. Mild.

- Exophthalmos : Present. Absent.

- TIT : Usually medical. Usually surgical.

monosymptomatic cases : when one of the manifestations of thyrotoxicosis predominates


e.g. thyrocardia : cardiac symptoms only (AF) .

Investigation : ( Thyroid function tests )

I. Investigations for the cause : ~

1. TSH level : [ n = 0.5 - 5 µu/L l


a) It's the most sensitive test of thyroid function.
b) l l in most cases of thyrotoxicosis by-ve feed back mechanism .
c) ii only in TSH producing pituitary tumor : very rare.
2. Imaging for the gland :
a) Thyroid scan : ( Radioactive nuclear
medicine studies )
Radioactive material is injected into a
vein or swallowed by mouth and the
thyroid gland takes up this material : 'Hor
nodule
o Hot nodule : this tissue is rarely
malignant.
o Cold nodule : l Oto 15% of these tissues are malignant.

32
In Capsule Series Endocrinology

b) U/S: indicated especially in cold solitary thyroid nodule


o Cystic ---+ benign.
o Solid : malignant -+ do needle aspiration biopsy.

So, everyone likes her to be hot & cystic, not cold & solid

c) Fine Needle Biopsy ( FNB) : a very small needle is introduced


into the nodule and cells are aspirated for study.
3. Thyroid receptor antibodies : thyroid stimulating immunoglobulin (TSI)
can be detected in Grave's disease.

The term LATS ( long acting thyroid stimulator) referring to this antibody is no longer used.
II. Assay of the hormone level : 6
a) Total T3 U : [n: 70- 170 ngm%] - not accurate.
b) TotalT4U: [n:5-12µgm% ] ---+ not accurate.

T3 & T4 are over 95% protein bound to TBG ( thyroxin binding


globulin ) so, ....
o Increased TBG as in pregnancy ---+ ii total T3 & T4 but free T3 & T4 are
normal.
o Decreased TBG as in liver cirrhosis & nephrotic syndrome---+ Ltotal T3, T4
but free T3 , T4 are normal.

c) Free T3 ( n: 0.4 ng % )
d) Free T4 ( n: 1.6 ng % ) difficult to measure, we use free T4 index.
e) T3 resin uptake : j ( n = 25% - 35%)
Radioactive T3 is added to the patient's serum it's fixed to the unoccupied
binding sites of TBG -+ the remaining radioactive is then absorbed into a resin
f) Free thyroxin index : j ( > 11 .5) = T3 resin uptake x total T4.

33
In Capsule Series Endocrinology
Ill. Investigations for the function of the hormone :
o Blood:
✓ l cholesterol, j Ca , j Calcitonin in medullary carcinoma.
✓ Lag sugar curve.
o Urine : Polyuria , j glucose.

IV. Suppression test : material : T3 .


✓ Normally -+ l RAIU (radioactive iodine uptake).
✓ In thyrotoxicosis -+ no effect.

WHAT IS YOUR DIAGNOSIS? SHOW ME UR INTELLIGENCE ©©


Thyroid hormones & TSH RAI Uptake scan (RAIU) Diagnosis
1- LTSH, i free T3 & T4 i ..............
2- LTSH, i free T3 & T4 l ...............
3- LTSH, l free T3 & T4 l ...............
Answers to diagnosis column in table can be found at the end of this subject.

Treatment:

I. Medical :I
Indication :
o 1ry thyrotoxicosis ( Grave's disease ).
o 2ry cases: pre-medication before operation.
o Treatment of complications e.g. HF.
o Pregnancy.
Contraindications :
o Huge goiter.
o Retro-sternal goiter.
o Suspicion of malignancy.

34
In Capsule Series Endocrinology
Lines of medical treatment :
& B-blockers ( propranolol)
o Provide rapid symptomatic control : tachycardia, tremors & heat intolerance.
o They also decrease peripheral conversion ofT4 to T3.
a Anti-thyroid drugs :
- Methyl thiouracil : 200mg t.d.s. then reduce after 4-6 w to 100 mg t.d.s.
- Propyl thiouracil : 100mg t.d.s. then reduce after 4-6 w to 50 mg t.d.s.
- Carbimazol (Neomercazol) : 20 mg t.d.s. then reduce after 4-6 w to 10 mg t.d.s

• Duration : 1 - 2 years. (until thyroid hormone stores become depleted)


• Mechanism :
o These drugs inhibit the formation of thyroid hormones.
o Propylthiouracil : blocks iodination and coupling of tyrosines. Also, it
inhibits peripheral conversion of T4 to T3.
• Side effects :
o Agranulocytosis .
o Hypothyroidism---+ TTSH ---+ Goiter.
o Relapse : on sudden stoppage.
o Others GIT upset, cholestatic jaundice.
II. !Radio-active iodine :I
Indication :
o Failure of medical treatment. o Patient is unfit for surgery.
o Recurrence after thyroidectomy. o Toxic adenoma.
Contraindications :
o During pregnancy, lactation & childhood.
o Huge goiter, retro-sternal goiter & progressive exophthalmos.
Side effects :
o Hypothyroidism. o Bone marrow depression.
o Fetal anomalies, if taken in pregnancy.
o Tincidence of malignancy, so---+ not given in childhood.

35
In Capsule Series Endocrinology
Precautions :
It's important to stop anti-thyroid drugs 4 days before & 4 days after radioactive
iodine therapy as it prevents radioactive iodine uptake by the gland.

Ill. ~urgical :I (sub-total thyroidectomy )


Indications :
o 2ry thyrotoxicosis.
o Failure of medical treatment.
o Huge or retro-sternal goiter.
o Suspicion of malignancy.
o Pregnant women not responding to drugs
Pre-operative preparations :
o Antithyroid drugs lead to euthyroid state.
o Lugol's iodine ( 5% iodine in 10% K iodide) : l size
& vascularity of the gland.
Complications :
o Laryngeal nerve palsy.
o Hypocalcaemia due to removal of parathyroid gland.
o Hypothyroidism.

IV. [reatment of complications :I


► Ocular complications :
o Protect eye by ointment, sun glasses, methyl cellulose.
o Prednisone ~ tl lymphocytic infiltration.
o Radiotherapy of the orbit.
o Orbital decompression.
► Heart failure :
o Bed rest, salt restriction, digitalis & diuretics.
o Persistence of AF ~ Digitalis or DC shock.

36
In Capsule Series Endocrinology
► Preitibial myxoedema :
o Remits spontaneously after months or years.
o Local Betamethasone cream ( local steroid ) relieves pruritis.

► freatment of Thyrotoxic crisis : 6


1. Propranolol in full dose is started immediately for tachyarrhythmias
( 160 mg/d orally, in 4 divided doses or I mg/4h IV)
o It blocks the peripheral effects of thyroid hormone.
o Also, it prevents the peripheral conversion of T4 to T3
o The effects are dramatic and results may be seen within 10 min.
2. Propylthiouracil :
o Dose : 600 loading dose then, 200 mg/6h. (orally, rectally or
nasogastric tube ) .
o It inhibits the synthesis of new thyroid hormone & prevents the
peripheral conversion of T4 to T3.
o Clinical effects may appear within 1 hour.
3. Na iodide: 1 gm over 24h 1v --+ !! thyroxin release from thyroid gland.
4. Hydrocortisone : 100 mg/8h IV or IM, to correct the hypotension.

• Thyrotoxic crisis ---+ jj cortisol metabolism ---+ relative adrenal insufficiency ---+
refractory hypotension. Hydrocortisone correct this hypotension.
• Also, steroids prevent peripheral conversion of T4 ---+ T3

5. Treatment of precipitating factor : antibiotics for infections.


6. Symptomatic treatment :
• Dehydration : IV fluid.
• Hyperthermia : ice bags, paracetamol. Aspirin should be avoided as it
decreases protein binding with subsequent increase in free T3 & T4
• Later on restoration of euthyroid state may take up to 8 weeks

37
In Capsule Series Endocrinology

o Medical treatment : n ropylthiouracil.


o Carbimazole not preferred in pregnancy as it crosses the placenta & may cause
congenital defects, including aplasia cutis of the neonate.
o Radioactive iodine : absolutely contraindicated as it is teratogenic & carcinogenic
o Surgery indicated in : appearance of side effects.
o Follow up: should be done to diagnose neonatal thyrotoxicosis because TSI can cross
the placenta & stimulate fetal thyroid gland.

Answers to table : ©
1- Primary hyperthyroidism.
2- Facticious hyperthyroidism or inflammation of the gland (Subacute thyroiditis).
3- Secondary or tertiary hypothyroidism.

Now, and after your journey with Hyperthyroidism,


REMEMBER that ..

0 0
Everyone likes her to be Hot & ~ 'stic .. NOT .. Colcl & Solid

I'm talking about thyroid nodule ....


- Hot nodule by thyroid scan & cystic by US : benign nodule.
- Cold nodule : may be malignant in about I 0%

38
In Capsule Series Endocrinology

II Myxedema
(Hypothyroidism in adult)
I
1. Physiology :
Refer to hyperthyroidism.

2. Etiology & types :

a. 1ry mxyedema : (5l+ H )


i. Idiopathic atrophy of the gland.
II. Iatrogenic :
1. post thyroidectomy.
2. radioactive iodine.
3. anti-thyroid drugs.
4. lithium.
5. amiodarone.
111. Infections : viral infection ( De Quervain'sthyroiditis)
1v. Iodine deficiency.
v. I nfiltration : tumor.
vi. Hashimoto' s thyroditis :
l . auto-immune disease. ( thyroid antibodies: +ve ).
2. gland is replaced by lymphocytic infiltration.
3. gland is firm, symmetrically enlarged & not tender.
4. transient thyrotoxicosis may occur followed by hypothyroidism.

b. 2ry Myxedema : 2ry to pituitary failure e.g. tumor ( t TSH ).

c. 3ry myxedema: deficiency of TRH secreted from hypothalamus.

d. Peripheral resistance to thyroid hormone : very rare.

39
In Capsule Series Endocrinology

3. Clinical picture : ~ > o 30-5oyears.


a. CI p of i cause : Surgery, Anti-thyroid drugs.

b. C / p of i hormone :
In hypothyroidism every thing slows down except the period !!

i. General :
1. Intolerance to cold.
2. Lethargy & fatigue.
3. Weight gain : due to decrease of metabolism, fluid
retention, constipation.
4. Serous effusions (pleural , pericardia! & ascites).
5. Face:
a. Expressionless with puffy eye lids.
b. Eyelids drop (decrease of adrenergic drive)
c. Loss of outer 1/3 of eye brows.
d. Large tongue.
ii. GIT :
1. Slow motility -► constipation
2. Slow absorption -► malabsorption syndrome.
111. Genital :

1. ~ ~ Menorrhagia in 1ry hypothyroidism

2. o ~ Impotence & gynaecomastia.

iv. Cutaneous :
1. Dry, cold, pale & non sweaty skin.
2. Non pitting edema : due to intradermal accumulation
of proteins, not interstitial edema fluid.
3. Yellowish discoloration due to carotinemia.
4. Hair loss.

40
In Capsule Series Endocrinology
V. CVS :
1. Hypercholesterolemia-+ ischemic heart disease.
2. Diastolic HTN due to -0, thyroxin ( thyroxin ¢ VD )
3. Sinus bradycardia.
4. Pericardia! effusion.
vi. CNS :
l . Slow intellectual & motor activity.
2. Depression, frank psychosis (myxedema madness)
3. Deposition of mucopolysaccharide in :
a. Tongue slow speech.
b. Vocal cords hoarseness of voice.
c. Flexor retinaculum: carpal tunnel syndrome.
d. Peripheral nerves : peripheral neuropathy.
e. Joints & muscles: slow relaxed reflexes.

vii. Heamatological :
l . Microcytic hypo-chromic anemia : iron deficiency
anemia due to menorrhagia.
2. Macrocytic anemia : associated pernicious anemia .

v111. Hypothyroid coma ( Myxedema coma ) :


1. Untreated long standing hypothyroidism.
2. Precipitated by cold exposure & infections.
3. Manifested by : 5 H
a. Hypothyroidism with loss of consciousness.

b. Hypothermia. c. Hypoglycemia.

d. Hypoventilation.

e. Heart failure & Hypotension.


Although this condition is called Myxedema coma , frank coma is uncommon!!

4f
In Capsule Series Endocrinology
ix. Gland:
1. Enlarged as -+ Hashimoto's thyroditis.
2. Atrophy-+ 1ry idiopathic forms.
3. Scars of previous operations.

4. Differential diagnosis :
a. Nephrotic syndrome.

b. Depression.
b Other causes of hypothermia : cold weather, shock ..
~ Difference between Primary & 2ry (pituitary) myxedema :

Pituitary mvxedema thyroid myxedema


1. Clinical picture :
o Skin Depigmented, thin Coarse, thick

o MacroQlossia
o MenorrhaQia
with wrinklinQ
No
No
Yes ~--
MCQ
Yes ~
,_,,
2 . Investi~ation :
a- TSH: - Low - Hiqh
b- FSH, LH, ACTH : - Low - Normal
c- Hypercholesterolemia - Rare - Common

5. Investigation :
a. Investigations for the cause :
1. TSH : [ the most sensitive test for 1ry thyroid disease ]
ii in thyroid myxedema, Hin pituitary myxedema .
ii. Thyroid antibodies.

b. Assay of the hormones level :


1. Serum hormones :

a. TSH : ii in 1ry type. [test of choice]


b. T4 & T3 : H
It is important to note that normal T4 not exclude hypothyroidism
2. Radio-active iodine uptake :l

42
In Capsule Series Endocrinology
c. Investigations of the hypofunction of thyroid hormone
i. Blood:
1. CBC: anemia
2. Blood glucose -+ flat sugar curve. DD of low voltage ECG :
3. j cholesterol & Prolactin. • Hypothyroidism.
• Pericardial effusion.
4. j CPK ( creatinine phospho-kinase). • Obesity.
5. i LOH ( lactate dehydrogenase).
ii. ECG findings: sinus bradycardia, low voltage.
iii. BMR low.

6. Treatment :
a. Replacement therapy: life long
• L-thyroxin : start small dose 0.05 mg/d & gradually up to
maintenance dose ( 0.2- 0.3 mg/day).
• The dose is increased very gradually to avoid precipitation
of angina & HF.

li iik¥0Ut4hi•1i;,N!i4•t4 11Eli•hiEI 6
a. Hypothyroidism :
Large doses of T4 200 micro g IV & maintenance 50 micro g/d IV or
T3 : 40 micro g IV & maintenance 10 microgram/d IV.
b . Hypothermia : rewarming slowly to avoid arrhythmias.

c. Hypoglycemia : glucose IV.

d . Hypoventilation : Tracheal intubation & mechanical ventilation.

e. Heart failure & Hypote nsion: cardiac support.

L Treatment of precipitating factors e.g. antibiotics for infections.

43
In Capsule Series Endocrinology

Cretinism
Hypothyroidism during infancy
I
1. Physiology: Refer to hyperthyroidism.

2. Etiology :
! Congenital.

! Endemic cretinism-+ iodide deficiency.


! Excess anti-thyroid drugs during pregnancy.

3. Clinical picture :
• Persistent physiological jaundice.

• Hoarse cry & feeding problems.

• Disproportionate dwarfism.

• Big lips & protruding tongue & Delayed dentation.

• Delayed walking.
• Muscle weakness( pot belly with umbilical hernia).

• Dry & cold skin.


• Mental detoriation if not treated within 6 months.

4. Investigation :
• T4 & T3: H
• TSH : ii
• X-ray of carpal bone : delayed appearance of ossification centers

5. Treatment : Replacement therapy


• L-thyroxin 0.025 mg/day & ii dose up to 0.2 mg/day
• It should be started before the 1st 6 months to avoid mental
retardation.

44
In Capsule Series Endocrinology

Goiter ,

Definition: Goiter (or goitre) is an enlargement of the thyroid gland.

classification:

i. Simple goiter ( Euthyrold goiter ) : ( non toxic , non inflammatory , non neoplastic)
- Persistent low level of thyroid hormones - TTSH - thyroid enlargement.
o Physiological: Puberty, Pregnancy. ( due to increased the demand

of iodine, so there is relative iodine deficiency )


o Iodine deficiency.

o Drugs : Anti-thyroid drugs, lithium.

- C/P: Soft symmetrical thyroid swelling.


- Investigations :
o Slight elevation of TSH.
o Normal thyroid scan & antibodies.
- Treatment :
o 12 supplementation.
o In young : thyroxin 100 - 150 mcg/d orally to suppress TSH.
o In elderly: thyroxin is contraindicated as goitre rarely decreases in size.
o Large goiter : surgery ( for cosmetic purpose or compression symptoms }.

ii. Toxic goiter : ( goiter with hyperthyroidism)


o Diffuse : Grave's disease ( 1ry thyrotoxicosis )

o Toxic nodular goiter ( 2ry thyrotoxicosis, Plummer's disease)

iii. Inflammatory goiter : e.g. Hashimoto's thyroiditis.

iv. Neoplastic goiter.

45
In Capsule Series Endocrinology

HYROID CANCE

Certain risk factors for thyroid cancer :

l It is more common in women ( 2:1 ), but men have worse prognosis.


l It is more likely in a nodule developing in child or patient older than 60.
l A single hard painless nodule showing rapid growth.
l Cold nodule in multinodular goiter.
l History of neck irradiation.
l Enlarged LNs in the neck region.
Cell type Frequency Behavior Spread Treatment

Papillary 70% Commonest. May • Spreads via Total thyroidectomy


be multifocal. l~12h nodes. & adjuvant 131-
Occurs in young · Local LN Iodine.
people (F> M) metastases Thyroxin give if
occurs even with hypothyroid and to
occult tumors < reduce recurrence.
1cm. Good prognQsis
Follicular 20% Single Spreads via As above
encapsulated blood. to
lesion. Mean age lung/bone,...
50 yrs. (F>M)
Anaplastic <5% MQst aggressive. Locally invasive Surgery, chemo,
Hard rapidly radio • useless
enlarging. 80% die within 1 yr.
Wt loss, stridor,
hoarseness
Lymphoma 2% Variable Sometimes
responsive to
radiotherapy
Medullary cell 5% Arises from Lymphatic spread ~ Total
calcitonin has frequently thyroidectomy
secreting developed. ~131-iodine has no
parafollicular C role.
cells. Often ~Poor prognosis,
familial & but indolent
associated with course
MEN syndrome.

46
In Capsule Series Endocrinology

Ca metabolism
- Normal Ca level : 8.4 - 10.2 mg%.
A. Ionized ( active ) : 50%
B. Non-ionized (reserve) : 50%
- 40% bound to albumin. - 10% any other bond ( Ca carbonate, phosphate .... )

Liver cirrhosis & nephrotic syndrome ~ ! albumin ~ hypocalcaemia ,


but no tetany because ionized Ca is still normal.

- Regulation of Ca :
• Ca level is closely affected & related to P [ n. 3 - 4.5 mg% ]
• Notice that Ca X P = constant [ 40 ].
• Serum Ca & P levels are controlled by certain hormones :
a. Parathormone.
b. Calcitonin.
c. Active vitamin D.
• Ca regulation involves 3 sites: bone, intestine & kidney.

Parathonnone* vitaminD Calcitonin

Absorption j Ca !p i Ca ip ! Ca
( intestine I
Reabsorption i Ca !p i Ca ip ! Ca !p
( kidney I
Resorption i Ca !p i Ca ip ! Ca !p
( bone I
Blood j Ca lP j Ca jP l Ca !P
( net result )

*PTH acts directly on bone & kidney & indirectly on intestine ( through activation of vit.D ).

47
In Capsule Series Endocrinology

Hyperparathyroidism
1. Physiology :
a. Hormone : Parathormone ( PTH, 54 amino acid )

b. Gland: Parathyroid gland.


c. Function of the hormone : i Ca, tP
I. GIT : ii absorption of Ca (indirectly through vit D)
ii. Kidney : it reabsorption of Ca.
iii. Bone : ti resorption of Ca.
iv. j Renal Phosphate excretion.

d. Regulation of this hormone :


Hypocalcemia or hyperphosphatemia - i PTH.

2. Etiology :
a. 1ry :
i. Adenoma of the parathyroid gland. (mostcommon, 90%)
ii. Hyperplasia of the parathyroid gland.
iii. As a part of Multiple Endocrine Neoplasia : MEN-1, MEN-2A

MEN-1: 3P MEN-2A:
• Hyper-Parathyrodisim: 95% • Medullary carcinoma of thyroid 90%
• Pituitary tumor : 30% • Pheochromocytoma 40%
• Pancreatic tumor : 70% • Hyperparathyroidism 25%

b. 2ry : Due to J, Ca as in Chronic renal failure & malabsorption


syndrome.
c. 3ry: usually in end stage CRF
- Longstanding 2ry hyperparathyroidism - irreversible
parathyroid hyperplasia. ( start 2ry - end uy)
- Parathyroidectomy is the only appropriate treatment.
d. Para malignant syndrome ( oat cell carcinoma ).
48
In Capsule Series Endocrinology

3. CI P: 50% -+ asymptomatic, more common in ~ > 50 y

Disease of bone, renal stone, abdominal groans & psychic moans JJJJ
I. Bone : ( Ostitis fibrosa cystica )
1. pain.
2. pathological fracture.

Ostitis fibrosa cystic : Fibrous degeneration & cysts formation ( fl ) due to


osteoclastic activity secondary to hyperparathyroidism.

ii. Renal:

1. Repeated stones.
2. Polyuria ( Ca diabetes) & polydepsia.
3. Nephrocalcinosis & renal failure may occur.

iii. Others: 4 X2

1. CNS:
a. Drowsiness.
b. Depression.
2. GIT:
a. Peptic ulcer & Pancreatitis ( abdominal pain ).
b. Constipation.
3. CVS:
a. ECG: short Q-T interval & arrhythmia.
b. Hypertension.
4. Skin:
a. Dry.
b. Itching.
• Ca > nmg%. -+ Clinical manifestations of Ca
• Ca > 13mg%. -+ Renal impairment
• Ca > 14 mg% --+- Coma & cardiac arrest ( endocrinal emergency )

49
In Capsule Series Endocrinology

4. (4 X 3)

a. Endocrine :
i. Hyperparathyroidism ( 1ry & 3ry ) . The most common cause
ii. Hyperthyroidism.
iii. Addison's disease (Cortisone - -- vit.D).
b. Malignancy:
i. Bronchogenic carcinoma .
ii. Multiple myeloma.
iii. Lymphoma - activation of vitamin D.
c. Bone:
i. T.B. ii. Sarcoidosis.
iii. Immobilization.
d. Others:
i. Hypervitaminosis ( vitamin D ).
ii. Drug induced: thiazide , lithium.
iii. Familial hypocalciuric hypercalcemia.

5. Investigation :

a. Investigations for the cause : Parathyroid imaging: CT, MRI , radioisotope scan
b. Assay of the hormone level : parathormone : jj [ n: o.s - 1 ngm/ml J
c. Investigations of the function of the hormone:
i. Blood:
try 2ry 3ry
1. Ca i l
i
i
2. p l i i
3. Alkaline phosphatase i ii iii

NB : Persistent hypercalcemia , hypo P & an elevated PTH confirm the


diagnosis of1ry hyperparathyroidism.
ii. Urine:
1. j Ca [ n: 150 mg/ day ] & j P [ n: 1 gm / day ]
2. j hydroxyproline & c -AMP due to t bone resorption.
50
In Capsule Series Endocrinology

iii. X-ray:
1. bone: 0
a. Loss of lamina dura of the teeth. ( the earliest sign)
b. Subperiosteal bone resorption in the distal phalanges
& distal end of clavicle.
c. Ground glass appearance of the bones (Osteopenia)
d. Skull-+ mottling of the skull. ( salt & pepper skull).
e. Spine-+ cod fish spine ( indentation of vertebrae by discs)
2. renal : urinary stones , nephrocalcinosis.
iv. Bone biopsy : osteomalacia with excess osteoclasts.
d. Suppression test : ( steroid test )
i. +ve ( l Co level ) -+ normal.
ii. -ve ( no suppression)-+ Hyperparathyroidism.

6. Treatment :
a. Surgery: Removal of the parathyroid glands with Ca & vit D supply.

2008 guideline for surgery :

• Serum Ca: > lmg¾ above upper limit of normal.


• Creatinine clearance : < 60ml/min
• BMD: T-score < -2.5 at any site and/or previous
fracture fragility.
• Age : < 50 years.

b. Medical treatment of hypercalcemia :


I. l intake & ii fluid intake.
ii. l absorption : by oral phosphate.
iii. i loss of Ca : saline then fursemide or dialysis in sever cases.
iv. Calcitonin.

v. 13-blocker: J the adverse effect of hypercalcemia on the heart


vi. Cortisone is effective in some cases (related to vitamin D excess)

vii. Alendronate (Osteomax) : lL osteoclastic activity.


51
In Capsule Series Endocrinology

Hypercalcemic crisis ( serum Ca level > 14 mg % )

a. Manifestations ( non secific ) :


1. GIT: Nausea , vomiting ,constipation & abdominal pain.
ii. Renal: Polyuria & dehydration.
iii. CNS : Confusion & coma .
iv. CVS: Arrythmia , shortened QT interval.
b. Treatment:
i. IV normal sline : 4-6 L/day may be necessary. ( the first goal)
ii. Lasix ( 40 - 80 mg IV every 2 hours ) , Should NEVER be
administrated without saline infusion, or else dehydration will
be aggravated.
iii. IV pamidronate ( Aredia ) : drug of choice.
It inhibits the osteoclastic activity. Dose: 30 - 90 mg IV
infusion over 24 h.
iv. Calcitonin : it inhibits bone resorption & has hypercalciuric
effect.
v. Cortisone : is the first line of treatment for hypercalcemia
caused by vitamin D excess e.g. myeloma, lymphoma,
sarcoidosis, but is NOT beneficial in hyperparathyroidism.
vi. Dialysis is effective in removing Ca in patients with renal
failure.

Success usually comes to those who are too busy to be looking for it
Henry David Thoreau

52
In Capsule Series Endocr inology

II Hyp opar athy roid ism


II
(Tetan y)
Definit ion of tetany :
State of jj neuro-muscular irritability due to: ! ionized Ca,! Mg or alkalosis.
Etiolog y of tetany : ( ! ionized Ca , ! Mg or alkalosis )

a. Hypoca lcaemi a :
i. Hypopa rathyroidism :
1. Surgery remova l.
2. Irradiati on.
3. Di- george syndrom e : absent parathyroid & thymus glands.
4. Pseudo hypopa rathyroi dism ( resistance of PTH )
ii. ! Ca intake : starvation.
iii. ! Ca absorption :
l. malabso rption syndrom e.
2. ! active vitamin D.
iv. Precipit ation of Ca in tissue e.g. acute pancrea titis.
v. j Ca excretio n : CRF ( most common cause of hypocal cemla).

N.B. tetany rarely occurs in RF because of acidosis which convert non


ionized Ca into ionized Ca.

b . Hypomagnesaemia : Excess diuretic , Malabso rption syndrom e.

c. Alkalosis : ( Ionized Ca -+ non ionized Ca ).

i. Respiratory alkalosis : any case of hyperve ntilation :


ii. Metabo lic alkalosis :
1. loss of HCI : vomitin g , gastritis , gastere ctomy.
2. milk alkali syndrome.
3. hypoka lemia e.g. Conn's syndrome.

\hyp okale~a ( Kloss) !:; Al-K -Lossis ol


53
In Capsule Series Endocrinology
C I P of Tetany :
iii. Latent tetany : ( serum Ca= 7 - 9 mg%)
1. manifestations of tetany are not present.
2. manifestations appear by provocative tests :
a. Chvostek's test :
tapping over the facial nerve - contraction of the facial muscles.
b. Trousseau's test :
Inflation of a BP cuff above SBP for 3 minutes - carpal spasm.
c. Erb's test: (normally: 8 mili-ampers at least are needed for stimulations)
electric current< 4mili-ampers - muscle contraction.

iv. Manifest tetany : ( serum Ca < 7 mg% )

1. Irritability & parasthesia around mouth & fingers.


2. Muscle twitches & may be convulsion in severe cases.
3 . Spasm:
a. eye lids : blepharospasm.
b. mouth : trismus of jaw.
c. larynx laryngospasm.
d. carpo-pedal spasm.
e. diaphragm : Hiccough.
f. GIT : abdominal colic.
4. Cardiovascular: QT prolongation, refractory HF, hypotension.
5 . On ectodermal structures : ( only in hypocalcemia )
brittle nail , hypoplastic teeth , cataract.

Investigation : ( for the cause )


• In hypoparathyrodism: L PTH, l serum Ca, j P.
• In hypomagnesaemia : Refractory hypokalemia. MCQ
• In alkaosis: i PH , Lionized Ca.

Treatment:
a. Acute attack: IV Ca gluconate 10 ml 10% very slowly.
b. Treatment of the cause :
• Hypocalemia : oral Ca , vitamin D
• Hypomagnesemia : oral Mg.
• Alkalosis : Treatment of the cause , acidifying drugs.

54
In Capsule Series Endocrinology
( PHP) MCQ
- Here, the level of PTH is high but there is resistance to all its action.
- So, there are hypocalc emia & hyperphosphatemia.
- It's a genetic disorder.
- There are 3 different forms :

~ PHP type la :
o Albright's hereditary osteodystrophy : short stature, round face,
and short hand bones.
o Manifestations of hypocalc emia.

~ PHP type lb :
o Resistance to PTH only in the kidneys.
o Normal appeara nce+ clinical picture of hypocalc emia.

~ Pseudopseudohypoparathyroidism : I think it is the longest word in English •;;)

o The same presentation as type 1a, but is biochemi cally normal.


o Hormone resistance is not present.

55
In Capsule Series Endocrinology
• • •I('" . • . _ . .," • • _ -_•.,.•• , . _, -- l . ' , • ,. • -_#

., -:.. / ·~--.::~upr_
arenal·~ gla'nd",~- ;>",'l~- · 1

( Adrenal gland)
- There are 2 adrenal glands at the superior pole of each kidney.

- They are composed of outer cortex & inner medulla :

1. Adrenal cortex ( the outer part) which is formed of 3 separate layers :

o Zona Glomerulosa: secretes aldosterone ( mineralocorticoids ).

o Zona Fasciculata: secretes cortisol ( glucocorticoids)

o Zona Reticularis : secretes androgen.

n. Adrenal medulla ( the inner part ) : secrets catecholamines.

!Disorders of suprarenal gland :I


► Hyperfunction of adrenal cortex :

o Conn's syndrome : j aldosterone.

o Cushing syndrome : j Cortisol.

o Adrenogenital hyperplasia : j androgen.

► Hyperfunction of adrenal medulla : Pheochromocytoma.

► Adrenal cortical hypofunction: Addison's disease ( iry adrenal failure)

56
In Capsule Series Endocrinology

I Conn's syndrome
( 1ry hyperaldosteronism)
I
1. Physiology :
a . Hormone : Mineralocorticoid (Aldosterone).
b . Gland : Suprarenal gland ( Zona glomerulosa ).
c. Function : reabsorb Na & excrete K & H+
i. i Na , j H20.
ii. LK , LH+.
d . Regulation : Aldosterone secretion is stimulated by :
i. Hypovolemia -. j Renin -. j aldosterone

ii. j K ( hyperkalemia ) & LNa ( hyponatremia )


iii. A.C.T.H. -. during stress only.

2. Etiology :
a. Unilateral adenoma of zona glomerulosa: 60% of cases.
b. Bilateral hyperplasia : 30%
c . Paraneoplastic syndrome.

3. Clinical picture :
a . C/p of the cause Adenoma, Paraneoplastic syndrome ....
b. C/p of the hormone Hypertension with hypokalemia
i. Hypematremia & hypervolemia :

1. Hypertension: usually mild.


2. Edema is uncommon inspite of hypervolemia, due to :
a. Renal escape phenomenon.
b. Hypokalemic nephropathy-. polyuria.
ii. H H. ( metabolic alkalosis) : leading to tetany.

57
In Capsule Series Endocrinology
iii. H K. ( hypokalemia ) : muscles

1. Skeletal muscles : weakness, muscle cramping.


2. Smooth muscles (GIT) : atony, constipation.
3. Cardiac muscle : arrhythmia , ECG changes ( prominent uwave)
4. Others : polyuria, paraesthesia.

4. Differential diagnosis : form 2ry hyperaldosteronism (stimulus is extra-adrenal )

try hyperaldosteronism 2ry hyperaldosteronism

Source - Adrenal Gland. - Renal artery stenosis.

- Heart failure.

- Nephrotic syndrome.

- Liver cirrhosis.
Aldosterone +++ +
Renin H ii
HI'N mild. No HTN except in renal
artery stenosis - sever.

lry
j Blood volume
hyper-
aldosteronism
Initiating event
i Na retention l Renin

j Aldosterone
production l Blood volume

Initiating event
j Na retention i Renin

2ry
hyper- j Aldosterone
production
aldosteonism

58
In Capsule Series Endocrinology

5. Investigation : HIGH ALDOSTERONE, Low RENIN. LOW K

a. Investigations for the cause: suprarenal gland imaging: US, CT, MRI
b. Assay of the hormone level :
i. In blood:
l. i aldosterone in serum ( n = 3 -15 ngm%) .
2. l plasma renin activity (due to -ve feed back)
ii. In urine :
High urine aldosterone (tetrahydroaldosterone)
c. Investigation for the function of aldosterone :
i. Blood:
l. l K. ( hypokalemia, <3.SmEq/L)

2. i Na· Again, Hypertension with hypokalemia


3. alkalosis : j serum HC03
ii. Urine : l Na, i K, i H+
d. Suppression test : saline infusion, to confirm the diagnosis.
• Normally : +ve ( suppression of aldosterone )
• In Conn's : -ve ( No suppression)

6. Treatment:
a. Surgical removal of the tumor.
b. Aldosterone antagonist :
i. Aldosterone antagonist : spironolactone ( aldactone 400 mg/d )
ii. ACE inhibitors : captopril in 2ry hyperaldosteronism. MCQ

c. Symptomatic treatment : excess K & low Na.

59
In Capsule Series Endocrinology

Cushing's syndrome
1 . Physiology :
a. Hormone : j Cortisol (glucocorticoid)
b. Gland Suprarenal gland ( Zona fasiculata )

c. Function of the hormone : 10


i. CHO : hyperglycemia.
ii. Protein : catabolic.
iii. Fat
1. lipolytic.

2. Redistribution of fat in abnormal sites ( face, back of neck, trunk)

iv. Water & electrolyte: ( a/dosterone like )


1. i H20 & Na.
2. J K & H+.
v. Stomach j HCI
vi. Bone anti-vitamin D action.
vii. Blood ( 2 ii &3HJ MCQ
1. j RBCs.
2. i neutrophils.
3. Llymphocytes.
4. Lesinophils & basophilis.
Vlll. Psychosis.
ix. Androgenic action.
x. Anti-inflammatory & anti-allergic.

d. Regulation of the hormone : stimulated by ACTH.

60
In Capsule Series Endocrinology
2. Etiology :
I. ACTH dependent : ACTH> 15pg/dl

a. Pituitary adenoma (pituitary Cushing, Cushing disease or 2ry Cushing) :


basophil adenoma of pituitary gland -t secreting ACTH [ 70 %J.
b. Paramalignant syndrome e.g. oat cell carcinom a. (Ectopic ACTH
secretion).
II. ACTH independent : Undetecte d ACTH with elevated cortisol> 15mcg/dl

a. Adrenal adenoma : lry Cushing or Cushing syndrome.


b. Exogenous glucocorticoid therapy. ( Cushinoid picture ) .
3. Clinical picture :
a. CI p of the cause: Pituitary tumor, Paraneoplastic syndrome.

b. C / p of the hormone :
i. CHO : Glucose intoleran ce & may be secondar y DM in 15 %.
ii. Protein : catabolic
1. Muscle wasting & weakness.
2. Capillary fragility: purpura.
3. Osteoporosis.
4. Thinning of skin, stria rubra,
delayed wound healing.
iii. Fat : abnorma l deposition of fat :
1. Face: moon face with acne.
2. Inter-scapular area : buffalo hump.
3. Trunkal obesity with thin limbs ( samboxa shaped obesity)
iv. Water & electrolyte : j Na, H20 & ! K, H+
1. HTN ( due to j Na & 1 the action of catechola mines )
2. Manifestations of hyookale mia.
3. alkalosis.

61
In Capsule Series Endocrinology
v. Stomach peptic ulcer ( perforation ) . -
vi. Bone osteomalacia & osteoporosis.~
vii. Blood: polycythemia (plethoric face), Recurrent infections due to
diminished function of neutrophils.
viii. Psychiatric : depression & suicidal tendency.
ix. Androgenic action
l. Female : amenorrhea, hirsutism.
2. Male L libido , impotence & hirsutism.
x. .S,kin pigmentation only in .S,econdary ( ACTH dependent)
Cushing due to i ACTH

Causes of death : Cardiovascular disease & infections.

Features suggest ectopic ACTH secretion (Paraneoplastic


syndrome):
• Short duration.
• Weight loss rather than weight gain.
• Severe hypokalemia.
• Severe hyperglycemia (OM)

4. Differential diagnosis :
I. DD of the cause : 1ry & 2ry Cushing.
II. Pseudo Cushing : MCQ
a. Obese hypertensive diabetic patients.
b. Females taking oral contraceptive pills.
c. Chronic alcoholism - impaired liver function -+ impaired
metabolism of corticosteroids.
d. Depression.

62
In Capsule Series Endocrinology
5. Investigation :
a. Assay of the hormones level : Initial step : overnight
I. In blood:
dexamethazone suppression test
confirmed by 24 h urine free cortisol
l . cortisone : j ( N : s- 20 µg % )

a . Early: loss of circadian rhythm (not of cortisol level by night).


b. Later : persistent high level.

Single random cortisol level is not reliable.


2. ACTH : ( N : 60 pg/ml )
a. j -+ pituitary Cushing or Paramalignant syndrome.
b. l -+ adrenal Cushing.
ii. In urine : 24-bours urine free cortisol

High in Cushing's syndrome, depression & obesity. This is the single


best biochemical marker of Cushing syndrome. MCQ

b. Investigations for the cause :


i. CT scan of the adrenal if the ACTH is low.
ii. If the ACTH is high : MRI of the pituitary or CT of the chest looking
for an ectopic focus.
iii. Inferior petrosal venous sampling of ACTH : is an invasive procedure
used to confirm pituitary origin of the tumor in difficult cases.

c. Investigations of the function of cortisone :


i. Biochemical changes: lK, jNa, jglucose.
ii. Blood picture : j RBCs, neutrophils & ! lymphocytes, esinophils.

d. Suppression test : ( Dexametha:zone test ).


l. Low dose: 1mg overnight oral dexamethazone test
i. Normal response: +ve (morning suppression of cortisol level< 5 µg %)
ii. -ve (no suppression, plasma cortisol > 5 µg % ) : In Cushing,
depression or obesity.

63
In Capsule Series Endocrinology
2. High dose: 1 mg/ 6h -+ for 2days.
i. +ve (L cortisol level by more than 50% of the baseline) ---. iry (pituitary)
Cushing.
ii. -ve ( no suppression ) ---. 1ry Cushing & ectopic ACTH secretion.

6. Treatment :
I. Surgery:
- Surgical resection of the pituitary, adrenal or ectopic ACTH
producing tumor, followed by replacement therapy.
- Nelson syndrome : big pituitary adenoma with very high ACTH &
hyperpigmentation after bilateral adrnalectomy in a case of pituitary Cushing.

II. Irradiation :
Less effective than surgery, reserved for high surgical risk patients.
Ill. Medical : ( Steroid antagonists ) MCQ
a) Aminoglutethimide: Antisteroid drug
b) Mitotane : anti-neoplastic drug used in the treatment of
adrenocortical carcinoma.
c)Metyrapone : it blocks cortisol synthesis by inhibiting steroid 11 ~-
hydroxylase.
d) Ketoconazole: antifungal & also has antiglucocorticoid effects.

Medical adrenalectomy is done by all EXCEPT


a) Aminoglutethimide.
b) Mitotane.
c) Mexiletine.
d) Metyrapone.
Answer: C, Mexiletine : Class IB anti-arrhythmic.

64
In Capsule Series Endocrinology

II
Adrenogenital syndrome
( Congenital adrenal hyperplasia)
I
1. Physiology :
a. Hormone : Sex hormone (androgen).
b. Gland : Suprarenal gland ( zona reticularis ).
c. Function of the hormone : Gametogenesis & 2ry sex characters.
d. Regulation of this hormone ACTH.

2. Etiology:
Enzyme deficiency in the cortical synthetic pathways ( 21 hydroxylase enzyme) ~ !
cortisol level ~ j ACTH ~ act on zona reticularis ~ j sex hormone secretion

3. C / P : depends on age & sex of the patient.

a. Pre-natal : Female pesudohermaphrotidism :


( small vagina, big fused labia majora simulating scrotum)
b. Post-natal & before puberty:
I. In males : pesudo-precocious puberty ( musculinization, deepening of
voice, but with small testis & no spermatogenesis
ii. In females: virilism (amenorrhea, musculinization & hirsutism)
N.B. : Hirsutism before menarche is suggestive for congenital adrenal hyperplasia.
c. After puberty : there is virilism in females.

4. Investigation :
a. Investigations for the cause : Imaging for the gland.
b. Assay of the hormones level :
i. In blood : j testosterone, j ACTH.
ii. In urine : j metabolite of testosterone (17-hydrotestosterone).

5. Treatment : Cortisone ~ Suppress ACTH ~ ! Androgens.

65
In Capsule Series Endocrinology

(Addison's disease, adrenal insufficiency)


Failure of the adrenal cortex leads to a decrease in its hormones.
o Primary adrenal insufficiency: Addison disease.
o Secondary adrenal insufficiency: pituitary disease (panhypopituitrism)
o Tertiary: hypothalamic disease.

1. Etiology ot pnmary adrenal failure:


a. Idiopathic atrophy, most probably Autoimmune: the most common 80%
b. Tuberculosis -> 15%
c. Surgery & Irradiation.
d. Sarcoidosis & Hemochromatosis. JJJJ
e. Adrenal hemorrhage: in meningococcal septicemia.

2. C / P : The gradual & nonspecific symptoms often lead to an incorrect initial diagnosis.

a. C I p of the cause :
Associated with other autoimmune diseases: e.g. type 1 DM,
thyroid disease, vitiligo ( polyglandular autoimmune syndrome)
b. c / p or tne normo, ,e .
i. Hypoglycemia : ( due to l cortisone )
1. Drowsiness, lack of concentration, hunger pain, coma.
2. There may be absence of the usual warning signs due to
lack of adrenaline ( notice that J cortisone -.J action of catecholamine)
ii. Hypotension : (due to ! cortisone & aldosterone)
1. l Cortisol & aldosterone -> l Na --+ Hypovolemia &
decreased response of the blood vessels to the action of
catecholamines-> hypotension.
2. Postural hypotension is common due to l action of
catecholamine.

66
In Capsule Series Endocrinology
Iii. Hyper-Pigmentation : (due to high level of ACTH in Primary adrenal faliure)

1. ijACTH ~ direct action on melanocytes.


2. It is not present in secondary adrenal failure.
ll,l::-.=-..--....-:;~~ii1
3. Site :
a. Sun exposed areas : face & neck.
b. Friction & pressure areas : elbow.
c. Pigmented areas : areola.
d. Recent scars & nails.
e. Mucous membrane : mouth & tongue.

iv. GIT manifestations :


1. Anorexia, nausea, vomiting.
2. Alternating diarrhea and constipation.
3. Abdominal pain: due tot Na, tender renal angle due to TB.

v. Asthenla : marked muscle weakness, fatigue & weight loss (100%)

1. Laldosterone ~ hyperkalemia & hyponatremia.


2. Lcortisol ~ hypoglycemia.
3. Landrogens ~ -ve nitrogen balance.

vi. Others :
1. Polyuria : due to Na diuresis.
2. Neuropathy.
3. Deficiency of adrenal androgen : manifests in females
only, resulting in loss of axillary and pubic hair ( both are
under influence of adrenal androgen only in females ).
vii, Complications: "Addisonian crisis" ..... see later

67
In Capsule Series Endocrinology
3. Differential diagnosis :
a. 2ry adrenal insufficiency :
- Hypopituitarism _. l ACTH - l cortisone & ! androgen with no l aldosterone.
- In 2ry adrenal failure there are :
• No skin pigmentation. ( due to t ACTH)
• Minimal hypotension ( due to normal aldosterone ).
• Manifestations of panhypopituitarism.
b. Skin pigmentation :
- Endocrinal diseases : l ry Addison , 2ry Cushing , DM , thyrotoxicosis.
- Chronic renal failure. -Skin diseases
- Malignancy. - Pregnancy. JJJJ
- Hemochromatosis. - l ry biliary cirrhosis. JJJJ

4. Investigation : Low cortisol level< 5 µgldl confirmed by ACTH stimulation test

a. Investigations for the cause :


i. Imaging for suprarenal gland : Abdominal U/S , CT, MRI.
ii. Adrenal antibodies : in autoimmune cases.
iii. Chest X-ray : may show evidence of old or active TB.

iv. Investigations for thyroid disease & OM should be done in


suspected polyglandular autoimmune syndrome.

b. Assay of the hormones level :


l. Low morning cortisol level< 5 µg/dl is highly suggestive. The
diagnosis of Al is ruled out if morning cortisol level > 18 µg/dl
2. ACTH level : elevated (~ 50 pg/ml) in l ry adrenal failure, t in
2ry adrenal failure.

3. Aldosterone is deficient in primary Al & not deficient in 2ry


adrenal failure as it is under control of renin not ACTH.

68
In Capsule Series Endocrinology
c. Investigations of the functions
i. Biochemical changes :
o Serum Na : L (< 135 mEq/L)
o Serum K : j (> 5 mEq/L)
o Fasting glucose : < 50 mg/di
o HC03 : < 15 mEq/L (metabolic acidosis)
o BUN:> 20 mg/di due to prerenal azotemia secondary to
~ypovolemia.
ii. Blood picture: CQ
1. L Neutrophils.
2. j Lymphocytes, esinophils, basophils.

d. Stimulatory test: ACTH stimulation test


o It is the best diagnostic tool to confirm the diagnosis.
o Synacthen (synthetic ACTH) 250 mcg IV or IM followed by
measurement of serum cortisol levels at O, 30 , 60 minutes.
• Normal response: +ve ( j cortisol level> 18 µg/dl )
• 1ry Adrenal failure : -ve response (cortisol level< 18 µg/dl)

5. Treatment:
a. Treatment of the cause e.g. TB.
b. Diet : j ( Na , CHO & protein ) .
c. Replacement therapy :
i. Glucocorticoid replacement :
& Hydrocortisone 20 mg am, 10 mg pm. Or
& Prednisolone 5 mg am, 2.5 mg pm. Or
& Dexamethasone 0.5 mg am & 0.25 mg pm.
ii. Mineralocorticoid replacement: Fludrocortisone: 0.1 mg/d

to replace aldosterone, especially in severe hypotension.

69
In Capsule Series Endocrinology

During stress ( Infection or Surgery ) :


• lntercurrent infection : The dose of hydrocortisone is doubled.
• Surgery : premedication with l 00 mg hydrocortisone IM, then
100 mg/6h IM for one day in a minor surgery ( e.g. hernia) &
for 3 days in a major surgery .
____________________~
._____

I ( Acute adrenal failure)


It is a medical emergency, that may be fatal if not recognized and
treated immediately.

Etiology:
i. Acute on top of chronic :
1. Addison's disease subjected to: surgery, infection, bleeding .
2. Panhypopituitarism : if treatment is initiated with thyroxin.
ii. Acute from the start :
1. Surgery: bilateral adrenalectomy.
2. Sudden withdrawal of chronic cortisone therapy.
3. Meningococcal septicemia (Waterhouse-Freidrichon's syndrome)
4. Massive thrombosis of adrenal vein.

CI P:
a. C I p of the cause : Surgery, Infection.
b, C I p of the hormone :
I . Sever hypoglycerrna coma.
2. Sever hypotens·on + shock.
3. Sever asthenia + confusion.
4. Sever skin pigmen"ation • desquamation.
5. Sever aiarrhea -+ dehydration & acJte abdomen.
70
In Capsule Series Endocrinology
Investigation : As Addison's disease.

Acute adrenal failure should be suspected in any patient in the ICU who develops
hypotension of unclear etiology or has refractory hypotension ( hypotension not
corrected by saline & vasopressors)

Treatment : ( S-S-S) Steroid, Sugar, Saline ©


i. IV fluids :

► 1 - 2 L of normal saline ( 0.9 %NaCl).


► 1 -2 L of 10% glucose.

ii. Cortisone is started immediate!'! as follow :

► Hydrocortisone, 50 - 100 mg /6 h IV until the patient is


clinically stable then shift to oral cortisone.
► Dexamethasone ( Decadron ) can be given before or during
the ACTH stimulation test. It will not interfere with plasma
cortisol assay.
► Fludrocortisone is NOT needed during adrenal crisis as
glucocorticoids in high dose have mineralocorticoid effects.

iii. Treatment of the precipitating factors : e.g. infection or DIC.

71
In Capsule Series Endocrinology

Pheochnomocytoma
II II
(Hyperfunction of adrenal medulla)

• It's the commonest tumor arising from the adrenal medulla.

• Ruleof10:

► 10 % familial.

► 10 % bilateral.

► 10 % malignant.

► 10 % extra-adrenal ( in the sympathetic chain ) .

C/ p : t}
a. Paroxysmal hypertension.

b. Paroxysmal headache.
c. Paroxysmal sweating.
d . Paroxysmal tachycardia (palpitation)
e. Anxiety & psychiatric disturbance.

Investigation : t}
a. Urinary ii V MA ( Valenyle Mandelic Acid ) : j

b. Urinary catecholamine : i
c. Plasma catecholamine : i
d. CT scan , MRI for abdomen.

e. Radioisotope scanning : scanning with Meta lodo Benzyl Guanidine


( MIBG) produces specific uptake in sites of sympathetic activity.

Treatment:
a. Surgery is the treatment of choice.

b. Medical: combined a & ~ blockers.

~ If you start with~ blocker- unopposed a action - Hypertensive crisis.

72
In Capsule Series Endocrinology

I Steroid preparations II
Uses:
l . Replacement therapy :
a. Addison's disease.
b. Congenital adrenal hyperplasia.

2. Supplementary therapy in non endocrine diseases :


a . Anti-inflammatory & anti-allergic :
i. CNS:
1. cerebral edema with neoplasm.
2. j ICT.
ii. CVS :
1. rheumatic fever.
2. Post infarction syndrome ( Dressler's syndrome )
3. hemolytic anemia.
4. ITP.
111. GIT:
1. ulcerative colitis & Crohn's disease.
2. chronic hepatitis.
iv. Respiratory :
1. bronchial asthma .
2. COPD.
3. sacroidosis.
v. Renal: certain types of GN .
vi. Collagen disease: SLE, polymyositis ,PAN, Rh.arthritis.
vii. Allergic disease : anaphylaxis.
viii. Inflammatory conditions :
l. eye ~ conjunctivitis.
2. skin ~ eczema.
b. Anti-stress & anti-shock.

3. Suppression therapy :
a. Tissue transplantation.
b. Lymphoma & leukemia.

73
In Capsule Series Endocrinology
, ~ ettecrs & contraindications
~((fJrofl
~
-□
( . . UlflNt••11t••-•~~

1. From prolonged treatment


a-Metabolism:
• CHO - Hyperglycemia. - D.M .
• Fat - Moon face . - Cushing
- Buffalo hump. syndrome.

• Protein - Osteoporosis. - Myopathy.


- muscle weakness. - Wasting.
• Electrolytes - Na & water - Hypertension.
retention. - Hypokalemia.
- Hypokalemia. - Rickets &
- Hypocalcaemia. osteoporosis.
b-Systems:

• General - Delay healing of - Uncontrolled


wound. infection.
• Eye - Cataract. - Cataract.
- Glaucoma. - Glaucoma.
• CNS - Nervousness . - Psychosis.
- Insomnia.
- Psychosis.
• CVS - HTN. - HTN .
- HF. - HF.
- Thrombosis.
• Respiratory - Chest infections. - T.B.
• GIT - j Gastric acidity. - Peptic ulcer.
• Genital - Teratogenecity. - Pregnancy .
- Menstrual
disturbance.
- Hirsutism.

2. From sudden withdrawal of steroids


- Adrenal crisis. - Sudden
withdrawal.

I
74
In Capsule Series Endocrinology

It's a clinical syndrome in which there is an error of CHO metabolism ; due


to absolute or relative insulin deficiency, ending in chronic hyperglycemia,
glucosuria, vasculopathy, neuropathy & disturbed fat & protein metabolism.

Etiology: ( 1ry - 95% & 2ry ► 5%)


L Primary : ( includes 3 types : )

a. Type 1 : previously called Insulin-Dependant DM (IDDM) or juvenile-onset DM.


b. Type 2: previously called non insulin-dependant DM (NIDDM) or adult-onset DM.

c. S,Q_ec·a1 tvnes :
:, MODY : Mature Onset Diabetes in Young patient.
- MODY is the term used to describe type 2 diabetes
occurring in patients under the age of 25 with a strong family
history. MODY does no· always require insulin treatment.
- It is due to single gene lefect with an autosomal dominant
mode of inheritance. ( 1onoqenic form of diabetes ).
- There are 6 variants : M 'DY l, 2, 3, 4, 5, 6 due to different
gene mutation. All of 'v :h limit the ability of the pancreas
to produce insulin.

~ LADA: 1atent Autoimmune Diabetes of the Adult.

A condition in which Type 1 diabetes develops in adults. Here, the


autoimmune ~ cell destruction is slow. So, it is frequently initially
misdiagnosed as having Type 2 diabetes.

The old classification of insulin dependant & non-insulin dependant DM is misleading


because many patient type 2 DM eventually require insulin for control hyperglycemia.

75
In Capsule Series Endocrinology
~ Secondary
a. Pancreatic causes e.g . Chronic pancreatitis, Hemochromatosis.
b . Endocrinal :
o Cushing. o Acromegaly.
o Thyrotoxicosis. o Pheochromocytoma.
o Somatostatinoma , glucagonoma.
c. Drugs : p agonists, Cortisone, Thiazide, Contraceptive pills.
d. Genetic syndromes sometimes associated with diabetes :
o Down's syndrome. o Turner's syndrome.
o Friedreich's ataxia. o Myotonia dystrophy
e. Others : Gestational diabetes , Rubella, DIDMOAD syndrome.
Pathogenesis :
Type 1 : 15 %.

• An autoimmune destruction of the pancreatic f3-cells leads to

absolute insulin deficiency.


• Genetic factor play an important role ( the combination of HLA DR:J
& DR4 makes a person more likely to develop type 1 DM )
• Viral infection may play a role.
• Without insulin, these patients are prone to develop ketoacidosis.
• Although typically diagnosed before age 30, it can present at any
age due to variability in rate of f3-cell destruction. ( Peak incidence is at
about 12 years of age ) .
Type 2 : 85 %
• It's characterized by peripheral insulin resistance, so hyperglycemia
develops despite above average level of insulin.
• In addition, it may be due to abnormal structure of insulin or due to
anti-insulin hormones e.g. glucagons.
• Factors that may play a role in pathogenesis include : genetic
predisposition & obesity.

76
In Capsule Series Endocrinology
• - il • -'tl
• Incidence : 15% 85%
• Pathogenesis : Insulin deficiency due to Insulin resistance.
damage of 13-cells.
• Insulin level : H Normal or even li
• Age of onset : Younger (usually< 30y). Older (usually > 30y).
• Body weight : Thin. Obese (usually 80 %) .
• HLA association HLA DR3/4 No HLA association.
• Hereditary : - 30% in identical twins - Near l 00% in identical
- Usually no family history. twins.
- Both parents affected : - Strong family history.
10% risk for child. - Both parents affected :
70-100% risk for child.
• C/ P:
. Severity : Sever. Mild or moderate.
. Ketoacidosis : Common . Rare, need ppt factors.
. Complication : More common. Less common.
• Autoantlbodies : - Islet cell Ab ( ICA ) . No association with
- GAD ( Glutamic acid antibodies.
decarboxylase ) Ab.
• Treatment :
. Oral hypoglycemic Ineffective. Effective.
. Insulin: Necessary (essential for life ) Usually not required

Stages of DM :

I. Pre diabetes : ( IFG, IGT )


a. Pre-diabetes is a condition in which blood g lucose levels
are higher than normal but not high enough for a
diagnosis of diabetes. This condition is sometimes called
impaired fasting glucose (IFG) or impaired glucose
tolerance (IGT) , depending on the test used to diagnose it.
b. It's an intermediate category between normal & DM.

77
In Capsule Series Endocrinology
c. There is a risk for future diabetes & CVS diseases.
d . Criteria for d iagnosis of prediabetes :
o IFG: 100-125 mg%
o IGT : 140 - 199 mg%
o Hb Al c: 5.7 - 6.4
e. Prognosis :
l IGT (impaired glucose tolerance) leads to type 2 DM in
about 34% of cases over 5 years.
t IGT & IGF together: lead to type2 DM in 65% of cases.
t Prediabetics are at high risk for cardiovascular
complications.
f. This group includes :
i. +ve family history.
ii. Obesity.
iii. ~ with bad obstetric history- macrosomia.
iv. Renal glucosuria.

II . Latent diabetes :
Diabetes appears only on exposure to stress & disappears
after removal of stress e.g. pregnancy.
Ill . Chemical diabetes : Raised blood glucose with no symptoms.
IV. Clinical diabetes :
a. Uncomplicated : Classic triad of symptoms : 3 p

► polyuria : due to osmotic diuresis induced by sugar.


► polydepsia : due to loss of fluid.
► polyphagia with weight loss : ! insulin - no glucose can enter
satiety center - ii of satiety center. While loss of weight is
caused by fluid depletion , fat & muscle breakdown.
b. Complicated : May be the ist presentation. see later

78
In Capsule Series Endocr inology

Invest igation s of DM :

I. Blood :

1. Blood sugar tests :

i. Fasting blood glucose : ( no caloric intake for at least 8 hours)


• 70 - 110 mg % - normal.
• ~126mg %(~7.0 mrnol/l) -tDM.
• 110 - 125 mg % j6. I - < 7.0 mmol/1) - Impaire d
fasting glucose (IFG)
ii. 2 h. post-pra ndial :
• < 140 mg% - normal.
• ~ 200 mg % (J 1.1 mmol/1 ) -t DM.
• 140 - 199 mg % - Impaire d glucose toleranc e ( IGT )

Q To convert mmol/1 of glucose to mg/di, multiply by 18. ( ... X 18)


Q To convert mg/di of glucose to mmol/1, divide by 18 or multiply by 0.055.

2. Oral glucos e toleran ce test : ( OGTT )

i. Patient should be fasting over night.


ii. Fasting blood sugar is done.
iii. The patient drinks a glucose solution
contain ing 75 g glucose within four
200
minutes (WHO).
iv. Take blood samples every 1/2 h. for
2- 3 h.
V. Normal curve : 3 criteria 0
2 Hours

1) Fasting : 70-110 mg %
2) Reach maximal point in l h. but still under 180 mg %
3) Return to normal within 2 h.

79
In Capsule Series Endocrinology
~ Cortisone glucose tolerance test :

1. Oexamethazone 3 mg is given before OGTT.


ii. It will induce hyperglycemia in latent & pre diabetes.

II. Urine :
L. Glucosuria : occurs when glucose serum level exceeds 180 mg %
( renal threshold); but it's not a good indicator for OM d iagnosis.
2. Ketonuria : for diagnosis of diabetic ketoacidosis.

Ill. Monitoring of treatment :


L. Plasma glucose monitoring.
2. Glycosylated hemoglobin ( HBA1c )
i. Normally it's less than 5.5 % of total HB.
ii. A 1c ~ 6.5 in OM. ( prediabetes : 5.7 - 6.4 )
iii. > 12% - poor glycemic control in the past 3 months.
IV. Investigations for complication
L. Plasma lipids.
2. Renal : Urine analysis, serum creatinine, electrolytes.
3. Imaging : Chest X-ray, ECG, echo, duplex carotid & LL
V. Investigations for the cause : If 2ry diabetes is suspected

Diagnostic criteria for diabetes : ( American Diabetes Association J


1 . A glycated hemoglobin level ~ 6.5%

2 . Fasting plasma glucose of 126mg/dl (7 mmol/L) or greater on IWO. separate


occasions.

3. Classic symptoms of hyperglycemia ( e.g. 3 Ps ) PLUS random glucose of >


200 mg/dl.

4. 2 hours postprandial glucose of 200 mg/dl (11.1 mmoljl) or greater confirm


the presence of diabetes.

80
In Capsule Series Endocrinology

!Pathogenesis of diabetic complications :


1. Recurrent infections : not only common but are often more severe

It is due to impaired PMN functions (particular when acidosis is present) :


► Impaired Lecukocyte migration & phagocytosis.
► Decrease of antioxidant activities.
► Acidosis can inhibit antibodies production.
2. Glycosylation of blood vessels :
This may lead to thickness of basement membrane of capillaries with
narrowing of their lumen, leading to vasculopathy (Micro & macro
angiopathy)
3. Blochemlcal { accumulation of sorbltol ) :
Glucose is reduced to sorbitol by aldose reductase - .0. of ATPase activity -
leading to neuropathy & nephropathy.

Fomplication!' of DM]
Cutaneous:
1. Infections
• Carbuncles & recurrent abscesses.
• Fungal infections : especially in the ano-genital region & interdigitals.

2. Pruritis : especially pruritis vulvae due to monilial infection.


3. Delayed healing of the wounds.
4. Xanthelasma ; due to hyperlipidemia.
5. N ecrobiosis lipoidica diabeticorum :
• Red painless papules with yellow center.
• Site : Usually over the anterior surface of the legs.
• Cause : cutaneous blood vessels occlusion.
6. Cutaneous features of diabetic foot.
7. Acanthosls nigricans : black patches due to insulin spillover into the
skin in type 2 DM ( excess insulin ). MCQ
8. Lipodystrophy: at the sites of insulin injection.

81
In Capsule Series Endocrinology

Cardiovascular
---~.:,_ ·-
~ -. ·: . - . ., _;

1. Microangiopathy : ( Microvascular complications )


a) Diabetic retinopathy
b) Diabetic nephropathy
-- retina .
glomeruli.
c) Diabetic neuropathy

2. Macroangiopathy : ( Atherosclerosis )
- vasa nervosa.

a) Cerebral : thrombosis & ischemia.


b) Coronary : angina & Ml, may be painless due to neuropathy
c) Peripheral : gangrene & intermittent cludication.
d) Renal reno-vascular hypertension.

3. Cardiomyopathy due to microangiopathy.

4. Blood pressure

a) systemic hypertension.
b) postural hypotension due to autonomic neuropathy.

-,_Cq~ses ofHT.ff in-diabetic patient:


--- ---- -- - - - - - - -- - - -
1- Diabetic nephropathy.
2- Endothelial dysfunction.
3- Insulin - retention of Na.
4- 2ry diabetes : Cushing & acromegaly.
5- Type 2 DM may be a part of metabolic syndrome X which consists of :
( DM, hypertension, Hyperlipidemia, obesity).

Chest:
1. Recurrent chest infection e.g . T.B. ( T.B. follows DM as its shadow ).
2. Rapid deep respiration Kussmaul respiration & acetone smell in
DKA.

82
In Capsule Serles Endocrinology

Gastrointestinal : Diabetics never hove normal bowel habits


1. Mouth : gingivitis , loosening of teeth.
2. Stomach :
o Gostroparesis .
o Nausea, vomiting & abdominal pain in DKA.
3. Intestine :
o Diarrhea due to : ABCD
✓ Autonomic neuropathy.

✓ Bacterial overgrowth.

✓ Celiac disease ( autoimmune ) .

✓ Drugs : e.g. metformin, incretins.

o Constipation: due to vagal neuropathy.


4. Liver : fatty liver (NAFLD)
5. Gall bladder : chronic cholecystitis , gall stones.

Genital:
1. In J Impotence ( psychological, neuropothy, voscu/opothy)
2. In ~ Infections & pruritis vulvae.
3. Effects ofDM on pregnancy :
- On mother :
i. Eclampsia.
11. Post Partum hemorrhage.
iii. Puerperal sepsis.
- On fetus
iv. High birth weight.
v. Hypoglycemic baby.
vi. Congenital anomalies.

83
In Capsule Series Endocrinology
4. Effects of pregnancy on DM:
i. j needs for insulin due to j anti-insulin : estrogen.
ii. Lrenal threshold for glucose.
iii. j incidence of complications.

1. Lids : Infections ( chalazion , conjunctivitis) , Xanthelasma.


2. Iris : New vessels formation in iris ( rubeosis iridis )
3. Lens:

• Senile cataract ( occur at an earlier age )


• Repeated error of refraction secondary to osmotic lens changes
with fluctuating glucose level : Hyperglycemia leads to myopia,
Hypoglycemia leads to hypermetropia.
4. Nerves : Optic neuritis, Cranial nerve palsy ( 3, 4 & 6 nerves).
~- Diabetic retinopathy :I
a. It's the most characteristic form of diabetic eye diseases.
b. It is related to the duration & control of the disease.
c. It eventually occurs in nearly all patients of type l DM l O years after
diagnosis. It is present in l 0% of type 2 DM at diagnosis, 50% after l O
years and 80 % after 20 years.
d. It is usually accompanied by nephropathy & neuropathy.
e. Detected by fluorescin angiography.
f. There are 2 important types :
- Non proliferative ( background retinopathy)
✓ Characterized by i capillary permeability, Microaneurvsms,
hemorrhage , exudates.
✓ During this stage of diabetic retinopathy, no local treatment
required, except in a case of diabetic maculopathy, just good
control of diabetes and hypertension and stopping smoking.

84
In Capsule Series Endocrinology

- Proliferative :
Characterized by neovascularization, retinal detachment. this type
must be treated as early as possible by laser photocoagulation.
~ ·' without treatment 50% of proliferative patients become blind within 5 - 10 years.
• Proliferative retinopathy : more common in type 1 DM.
• Diabetic maculopathy : more common in type 2 DM.

Renal:
l . Interstitial Injury :
- Pyelonephritis : • Fever • Pain. • Dysuria.
- Acute necrotizing papillitis: fever, pain, dysuria and hematuria.

2. !D iabetic nephropathy :I (Diabetic glomerulosclerosis)


Definition :

Clinical syndrome characterized by t in urine albumin excretion ,t BP


up to end stage renal failure with progressive rise in CV risk.
Incidence:

o Long standing DM ( 30% of type 1 , 20 % of type2 ) .


o Genetic factor may play a role.

Pathogenesis : Glomerular sclerosis :


i. Early Thickening of basement membrane of glomeruli &
mesangial expansion .
ii. Hyalinization of afferent & efferent arterioles.
IGlomerular hyperfilteration C? proteinuria C? end stage renal disease.I
DM affects efferent more than afferent aiterioles ._So ; there is more narrowing in efferent than in
afferent, this iT intra-glomerular pressure ~ this will allow plasma proteins to escape into the urine
~ Some of these proteins will be taken up by the proximal tubular cells, which can initiate an
inflammatory response that contributes to interstitial scarring eventually leading to fibrosis.

85
In Capsule Series Endocrinology
Types:
i. Nodular ( Kimmelestiel-Wilson's syndrome}
ii. Diffuse.
Clinical picture :

i. Long standing DM especially poorly controlled diabetes after about


10-20 years. (rarely develops before l0y duration of the disease}
ii. Proteinurea : the early clinical sign of diabetic nephropathy
1. micro-albuminuria : 30-300 mg/day It is reversible by ACEls .
2. macro-proteinuria : > 300 mg/day
3. heavy proteinurea ( nephrotic syndrome }
Nearly 100% with gmss proteinuria will progress to End Stage CRF in 5 -15 y ®
\ iii. Lately, progressive loss of renal function ( CRF } .
iv. Hypertension, diabetic retinopathy& neuropathy are usually present.
G
.!
In patient with lypel OM, nephropathy almost always occurs in
the presence of retinopathy

Show me your intelligence : ©


Mamdouh, 21 year old male with type 1 DM, 4 years ago comes for follow up.
He has noted that his urine is foamy but otherwise has no complaint. Eye
examination shows no retinopathy. Urine analysis shows 4+ proteinuria. The
most likely cause of proteinuria in this patient is :

a) Diabetic ncphropathy
b) Membranous GN
c) Minimal change disease
d) Post streptococcal GN

WHY??
This case is NOT diabetic nephropathy. The absence of eye disease as seen
in our patient should prompt consideration of other causes of nephropathy.
As membranous GN is the most common cause of nephritic syndrome in
adult, answer b is the correct answer.

86
In Capsule Series Endocrinology
Stages of diabetic nephropathy :

A) Incipient nephropathy :

o Stage I: Nephromegally , j GFR.

o Stage II : Stage I + microalbuminuria on exercise.


o Stage Ill: as stage 2 but persistent microalbuminuria > 30 mg/d.
This stage may remain silent for up to l O- 15 years.
B) Overt nephropathy:

o Stage IV:

✓ Macro proteinuria > 300 mg/d. ( rarely the proteinuria

exceed 5 gm /d)
✓ ! GFR.
✓ Hypertension is common.

o Stage V: End stage CRF.


Investigation :

1- Urinalysis : Screening of microalbuminuria is a mandatory .


It should be performed in type 1 patients who have had diabetes > 5 years and all
type 2 diabetic patients starting at diagnosis (In type 2 DM , microalbuminuria can
be present at the time of diagnosis).
2- Blood : urea & creatinine.
3- U/S : Normal sized kidneys with j echogenicity.(not shrunken as usual CRF)
4- Biopsy: may be done if there is any doubt in the diagnosis. It is not always
necessary if the case is straightforward, with a documented progression of
proteinuria over time and presence of diabetic retinopathy.

Treatment : ~

1- Diabetic control can reverse the microalbuminuria.


2- Control of hypertension is a mandatory by ACEls & ARBs which
decrease the albuminuria even in normotensive patients.
3- Treatment of CRF : 30 + T see nephrology book

81
In Capsule Series Endocrinology
Diabetic foot:
Definition :
The foot of a diabetic patient that has the potential risk of pathologic
consequences, including infection, ulceration.
Pathogenesis : t
Neuropathy, vasculopathy, & infection combine to produce tissue
necrosis.
1) Neuropathy :
a. Sensory neuropathy ➔ -l, awareness of injury to the foot.
b. Motor neuropathy ➔ Abnormal motor function of the intrinsic muscles
of the foot ➔ foot deformity ➔ maldistribution of weight.
c. Autonomic neuropathy ➔ -l, sweating ➔ dry & fissured skin ➔ infection.

- In severe cases the abnormal distribution of weight on the sole can


result in repeated painless fractures and displacement of joints
producing the so-called Charcot joints.
- Charcot joint: ( neuro-arthropathy) : Non-infectious destruction of
bone & joint associated with neuropathy.

2) Vasculopathy : ( Impaired peripheral circulation )


lschemia will affect the ability to fight infections since delivery of
antibiotics to the site of infection will be impaired.
3) Infection :
• Diabetic foot infections are polymicrobial in nature.
• Lack of wound healing, systemic sepsis, neuropathy & vasculopathy
can lead to extensive tissue necrosis and gangrene requiring
amputation to prevent more proximal limb loss.

88
In Capsule Series Endocrinology
Clinical features of diabetic feet :
- Site of ulceration : at pressure points e.g.
• Base of the big toe ( head of the first metatarsal bone )
• Base of the 5th metatarsal. • Heel.

- Neuropathic & lschemlc feet :


Neuropathicfeet Ischemicfeet
-Warm -Cold
- Dry - Trophic changes/ hairless
- Foot pulses felt - Foot pulses not felt
- Usually painless ulcer - More often painful
- Callus present - Rest pain & claudication.

Wagner's classification of diabetic foot lesions:

Grade 0 High risk foot-no ulceration.


Grade 1 Shallow ulcer-not infected.
Grade 2 Deep ulcer with cellulites but no abscess or bone involvement.
Grade 3 Deep ulcer with bone involvement or abscess formation.
Grade 4 Localised gangrene (toes, heel).
Grade 5 Gangrene of whole foot.

Risk factors for foot ulcers or amputation include : 5 Ps

~ Poor glycemic control & Diabetes for> 10 years.


~ Peripheral neuropathy.
~ Bony abnormalities & callus.
t Peripheral vascular disease.
~ Previous foot trauma or surgery.
ifreatment :
I. Prevention of foot ulcers : 6 Ps
1. Pediatric care ( footcare ) : regular visits, examinations.
2. Patient education : control of diabetes, foot cleaning, earful nail cutting.
3. Pulse examination.

89
In Capsule Series Endocrinology

4. Protective shoes : avoid bare foot & tight shoes.


5. Pressure reduction : Crutches, Cushioned insoles, foot cast.
6. Prophylactic surgery : to reduce bony prominence.

II. Treatment of diabetic foot infections :


1. Wound care e.g. dressing.
2. Empiric antibiotic coverage, modified by culture findings.
3. Debridement of necrotic tissue.
4. Angioplasty to improve the distal circulation.
5. Amputation may be necessary in extensive cases.

Diabetic foot ulcer lschemic gangrene

Charcot foot

90
In Capsule Series Endocrinology

Diabetic infections :
1. Skin : Staph & candida . 3. Chest: T.B. & pneumonia.
2. Urinary tract : Pyelonephritis & 4. Genital : Pruritis vulva.
perinephric abscess. 5. GB : Cholecystitis.

6- Life threatening infections in diabetics :

t Malignant otitis externa :


o It is an uncommon but potentially life-threatening infection, almost always due

to Pseudomonas aeruginosa.
o Patients report a history of weeks to months of severe pain, otorrhea, and
hearing loss. Intense cellulitis is combined with edema of the ear canal.
o CT and MRI studies are essential for defining the extent of bone and soft-

tissue involvement.

t Rhino-cerebral mucormycosis :
o Mucormycosis is a life-threatening fungal infection.
o Patients present with facial or ocular pain and nasal obstruction followed by
proptosis and loss of visual acuity. Generalized malaise and fever, necrotic
turbinates may be found.

o Treatment consists of surgical debridement of the involved sinuses and


prolonged intravenous therapy with amphotericin B.

t Emphysematous cholecystitis :
o It is a severe form of acute cholecystitis characterized by gas production in the
gallbladder wall. Clostridium spp. are often isolated from bile cultures in
addition to other enteric flora.

o It is frequently associated with gangrene, perforation, and death.


o Diagnosis involves the demonstration of gas within the lumen or walls of the
gall bladder by ultrasound or CT scan.
o Surgical intervention is necessary in addition to broad-spectrum antibiotics.

91
In Capsule Series Endocrinology
Neurological :
I. Diabetic macroangiopathy
II. Diabetic neuropathy.
Ill. Diabetic comas.

I. Diabetic macroangiopathy: ( due to atherosclerosis}


a . Cerebral hemorrhage. b. Cerebral thrombosis.
II. Diabetic neuropathy:
Pathogenesis : unknown but there are 4 important theories:

l . Hypovitaminosis: polyuria washes water soluble B complex.


2. Microangiopathy of vasa nervosa.
3. Metabolic ketosis : toxic effect of keton bodies.
4. Transformation of glucose to sorbitol by aldose reductase enzyme.
C/ P : ( may precede the discovery of DM)
l . Mononeuropathy or mononeuropathy multiplex, affecting :
a. Peripheral nerves : femoral , ulnar & median nerves.
b. Cranial nerves : 3, 4, 6.
2. Polyneuropathy :
a . Sensory affection is more than motor affection.
b. Superficial sensation: Symmetrical parasthesia especially in
LL followed by stock & glove hyposthesia.

c. Deep sensations: nocturnal calf spasm is common.


d. Motor weakness is late.
3. Autonomic neuropathy :
- CVS:
o Postural hypotension.
o Painless myocardial infarction.
o Persistent tachycardia .

92
In Capsule Series Endocrinology
- GIT :
o Gastroparesis : delayed gastric empting.
o Diarrhea : severe, nocturnal & alternating with constipation.
- Genito-urinary :
o Impotence.
o Incontinence.
- Skin :
o Generalized sweating.

o Edema of the feet due to loss of the vasomotor tone.


o Trophic ulcers.

Treatment :
► Strict control of DM: diet, oral hypoglycemic or insulin.

► Reassurance may be all that needed.


► Physiotherapy if motor weakness is present.
► Drugs: Low benefit

a. Analgesics of neurotic pain : carbamazepine ( Tegretol J


b. Vit. B complex ( Tri B ) & ATP ( odenoplex ) .
c. Aldose reductase inhibitors : sorbinil.
d. Cerebrolysin. ( nerve growth factor)

Ill. Diabetic comas: ( Acute diabetic complications )


1. Hypoglycemic coma. ( insulin reaction ) : The most common
2. Diabetic ketoacidosis. ( DKA)
3. Hyperglycemic hyperosmolar non ketotic coma.
4. Diabetic lactic acidosis.
5. Other causes of coma : uremic, hepatic, ........ .

93
In Capsule Series Endocrinology

!Hypoglycemic coma!

a. Etiology :
• Missed meal or severe exercise after insulin or oral hypoglycemic drugs.
• Over dose of insulin or decreased elimination of insulin as in cases of
renal failure.

b. C/P : severity of symptoms can vary with the individual.

i. Adrenergic symptoms : ( warning signs ) ~


due to i catecholamines when blood glucose < 50 mg %
1- Tachycardia ( palpitation ).
2- Sweating.
3- Tremors.
4- Anxiety.
5- Hunger pain.

ii. CNS symptoms : ~

due to cerebral dysfunction when blood glucose< 30 mg%


1- Headache.
2- Blurring of vision.
3- Lack of concentration.
4- Convulsion may occur.
5- Coma.

Masked symptoms of hypoglycemia may occur in the following


conditions : ( not associated with adrenergic symptoms )
• During night.
• j)-blockers.
• Old age.
• Recurrent hypoglycemia.

94
In Capsule Series Endocrinology
c. Investigation : Low blood glucose< 50 mg% in male & 45 % in female.
d. Differential diagnosis :
o From diabetic ketoacidosis see later.
o From other causes of hypoglycemia : see later.
e. Treatment : Recovery is very rapid except in irreversible brain damage
L If patient is consciou s: oral glucose in the form of candy.
ii. If there is coma : IV glucose 50 gm 50 % .
iii. Hypoglycemia due to oral hypoglycemic drugs ( sulphonylurea ) : may
persist for long time ( days ) so glucose has to be infused for long time.

a. Definition :
DKA is an extremely serious metaboli c complica tion of DM due to sever
insulin deficienc y, it's characte rized by triad of: t
- Acidosis. - Ketosis. - Hypergly cemia ( usually >250 mg% ).
NB : There is no relation between the severity of hyperglycemia & the
severity of ketoacidosis.
b. Etiology: ketONE bodies are seen in type ONE DM ©
• Occurs mainly in type 1 DM, due to sever insulin deficienc y e.g.
• Previously un diagnose d DM
• Missed insulin ( neglecte d treatmen t ) .
• Precipitating factors :
• Infections ( lead to increased insulin requirem ents ) .
• Infarction ( I mean myocard ial infarction ) .
• Intoxicati on ( Alcohol ).

In type 1 DM : DKA occurs with or without Precipitating factors.


In type 2 DM : DKA occurs with Precipitating factors only.

95
In Capsule Series Endocrinology

I Diagnosis of infection in DKA may be difficult because :


~ Firstly, The temperature in patients with OKA is subnormal so the absence of
fever is not enough to exclude infection.
~ Secondly, leucocytosis occurs in OKA in the absence of infection.

c. Pathogenesis & c/P : sever insulin deficiency leads to : ft

1. Glucose can't enter the c e lls - hyperglycemia > 250 mg %.


11. Fat : ii lipolysis to produce energy - j production of keton
bodies ( P-hydroxy buteric acid, acetoacetic acid & acetone) -
ketoacidosis ( PH < 7.3 )
iii. Effects of ketoacidosis : gradual onset of : ~ X2

1- Muscles:
a. Generalized weakness.
b. Muscle pain.
2- Kidney : ketonuria together with glucosuria lead to sever
a . Polyuria .
b. Dehydration.
3- GIT:
a. Anorexia, nausea & vomiting.
b. a bdominal pain.
4- Respiration :
a . Kussmaul respiration ( deep rapid)
b. Acetone odor of breath.
5- CVS:
a. Depressed contractility & low blood pressure.
b. Rapid weak pulse.
iv. Hyperkalemia due to shift of K outside cells in absence of insulin.

v. Coma due to combined effect of : Ketone bodies, dehydration &


electrolyte disturbance.
In Capsule Series Endocrinology
d. Investigations :

i. Blood examination
o j glucose > 250 mg %, j ketone bodies.
o Acidosis ( PH < 7.3} with high anion gap.
o Electrolytes : jj K . Hypokalemia may occur due to polyuria
11. Urine examination : Polyuria, glucosuria & ketonuria.
e. Treatment : ~

Estimation of blood glucose, PH, blood gases & electrolytes {K, Na}
i. Fluid therapy :
o 4-8L is usually required.

o Normal saline 1 L/hour for the first 2 h followed by ½ L/h until


the deficit is replaced. 4-8 Lis usually required.
o At first hour, normal saline is given, then change to glucose
5% saline if blood glucose becomes< 250 mg/di.
ii. Insulin :
• Short acting soluble insulin ( regular, crystalline insulin ).
• Regimen : IV or IM
✓ IV insulin infusion : 5 - 10 u/ h infusion.
✓ Repeated IM : 20 U at the start then 6 U/ h
✓ SC route is ineffective because of hypotension &
reduction of skin blood flow.
• First hour : 5 - 10 U/h IV infusion
• Second hour :
✓ Continue insulin 5-10 U/h IV infusion as long as acidosis is
present.
✓ If blood glucose does not decrease by l 0% after the first
hour the insulin dose is increased.
In Capsule Series E ndocrinology
✓ If b lood glucose b ecom es below 250 mg/di, a dd glucose

5% IV infusion.
• Third hour : continue insulin IV infusion till PH becomes > 7.3,
HCO3 > 18, AG becomes normal, then give SC regular insulin

before meals.

iii. K therapy :
The serum K falls during insulin therapy ( intracellular shift), and this

fall may be dramatic. so add ( 20- 40 mEq) to each 1L saline.


iv. Na bicarbonate: in sever acidosis [PH< 6.9]

v. Treatment of precipitating factors e.g. infection : antibiotic s.

Complications during the treatment of DKA:


• Hypoglycemia.
• Hypokalemia.
• Hypervolemia.
• Rebound cerebral edema : it occurs in children & young adult if
plasma glucose is decreased rapidly ➔ shift of water to the cells
➔ brain edema. This can be prevented by gradual reduction of

plasma glucose (75 mg/h)

Hypogly cemic coma OKA


• History: - Missed meal. - Missed insulin.
• Onset : - Acute. - Gradual.
• Pulse : - Strong. - Weak & rapid.
• Bl.P. : - 1 - L
• Skin: - Sweaty. - Dry.
• Pupils: • Dilated. • Normal.
• Respiration: • Normal. • Rapid , deep.
• Urine: - Normal. • +ve su ar & acetone.
• Coma: - Irritable. - Not irritable.
• IV glucose : - Rapid recovery. - No effect.

98
In Capsule Series Endocrinology

• There is low insulin level which is enough to prevent ketosis , but not enough
to inhibit hyperglycemia, so there is hyperglycemia without ketosis.
• It occurs in old type 2 DM.
• Patients typically having sever hyperglycemia {> 600 mg/di), sever dehydration.
Plasma osmolarity > 340 mOsm/l (N : 290) is the landmark of this condition.
• Clinical picture :
o Sever hyperglycemia: polyuria, polydepsia.

o Sever dehydration ¢ renal failure & thrombotic complications.

o Sever hyperosmolarity ¢ CNS symptoms (convulsions, coma).


• Treatment The same as DKA but
o Aggressive fluid replacement. ( the most important measure)
o Low rate of Insulin infusion ( 3 U/h ) .

o S.C. heparin to prevent thrombotic complications.

o Na bicarbonate is not given.

!Lactic acidosi~

o It occurs in diabetics using high dose of biguanides e.g. metformin


o Biguanides ¢ tissue hypoxia ¢ anaerobic glycolysis ¢ accumulation of
lactic acid.
o C/P :
a. Rapid deep respiration ( Kussmaul! respiration ).
b. Confusion, coma.
o Treatment : NaHC03
In Capsule Series Endocrinology
DIA-GNOSTIC CRITERIA OF-DIABETIC COMAS

DKA HHNK Lactic acidosis


o Hyperglycemia > 250 o Hyperglycemia > 600 o Severe acidosis with
mg/dL. mg/dL. hyperventilation.
o Acidosis with blood PH < o Serum osmolality >310 o Acidosis with blood PH <
7.3 with high anion gap. mosm/kg. 7.3 with high anion gap.
o Serum bicarbonate < 15 o No acidosis, blood PH> 7.3 o Serum bicarbonate< 15
mEq/L o Serum bicarbonate > 15 mEq/L
o Serum positive for mEq/L. o Absent serum ketone.
ketones. o Serum lactate > 5 mmol/L.

► Acute diabetic complications : diabetic coma.


► Micro-vascular complications : diabetic triopathy.
► Hyperglycemic coma : DKA, HHNK.

Treatment of DM :
I. General measures.
II. Diet.
III. Oral hypoglycemic.
JV, Insulin.
V. Treatment of complications.

I. General measures :
a. Reassurance.
b. Education about nutrition & lifestyle modifications.
c. Exercise.

100
In Capsule Series Endocrinology
II. Diet :
a. It's an important item in control of DM
b. Diet alone can control mild coses of type II DM
c. Total calories/day: depending on weight & physical activity
1. Mild activity -+ 1500 cal/d.
ii. Moderate activity -+ 2500 cal/d.
iii. Severe activity & pregnancy -+ 3500 col/ d.
d. Food components :
i. CHO : 50% of calories . avoid simple sugars.
ii. Fat : 30% of calories . avoid saturated fat.
iii. Protein : 20% of calories.
iv. Vitamins : B-complex & vit. A
V. il fibers : j satiety.
e. Number of meals :
3 main meals + 2 snacks in between, to avoid hyperglycemia
or hypoglycemia.

Ill. Oral hypoglycemic drugs


a. Biguanides :
- Mechanisms of action :
l . j anaerobic glycolysis.
2. H intestinal glucose absorption.
3. H appetite.
- Preparations : Metformin ( Cidophage) 500 - 850 mg t.d.s.
- Indications :
l . type 2 DM not controlled by diet alone esp. in obese patients.
2. combined with suphonylurea or insulin to achieve control.

101
In Capsule Series Endocrinology
- Side effects :
~ Contraindicated in renal failure (discontinue usage if
creatinine ~ 1.5)
~ Nausea, vomiting, diarrhea, and abdominal upset.
~ Lactic acidosis.

b. Sulphonylureas :
- Mechanisms of action :
1. ii insulin secretion from pancreas. (main action)
2. ii peripheral action of insulin (insulin sensitizer)
3. H hepatic production of glucose.
- Preparations :
Drug Trade name Dose ( mg/d) Duration of action
• Old generation :
- Cholropropamide Pamidine 100 - 500 long acting
- Tolbutamide Diamol 500 - 3000 short
• New generation :
- Glibenclamide Doanil. 2.5 - 15 long acting
- Glimepiride Amory/. 1- 6 long acting
- Gliclazide Diamicron. 80 - 480 intermediate
- Glipizide Minidiab 2.5 - 30 short

- Indication :
Type 2 DM not controlled by diet alone especially in non obese patients.
- Side effects :
- Allergy.

- Aplastic anemia.

- Hypoglycemia.

- Hepatitis.

- Hyponatremia due to increased ADH by cholropropamide.

102
In Capsule Series Endocrinology
c. Recent drugs :

1- Alpha glucosidase inhibitors : Acarbose ( glucobay )


o Prevent breakdown of CHO in intestine ➔ J. glucose absorption.
o Disadvantage : Contraindicated in hepatic patient , abdominal
pain.

2- Glinides : Repaglinide , Nateglinide.


o Stimulate secretion of insulin from beta cells.
o Rapid onset & short duration , acts on post prandial
hyperglycemia.

3- Thiazolidinediones : ( Rosigitazone ) CQ
o Increases insulin sensitivity

o Can be combined with sulfonylurea, metformin or insulin.


o Advantage: improve diabetic dyslipidemia.
o Disadvantages :
✓ Contraindicated in mild liver impairment.

✓ Given cautiously to patient with heart failure, renal impairment.

✓ Weight gain, mild edema.

All oral hypoglycemic drugs are given cautiously to patient with hepatic,
renal & heart failure.

4- lncretins : ***
o Incretins are hormones that are released from the gut into the blood within minutes after
eating. Incretin honnones are insulinotropic ( they induce insulin secretion after eating).
o There are two incretins :
a) Glucose-dependent insulinotropic peptide {GIP) : It is released from K cells in the
duodenum and jejunum.
b) Glucagon-like peptide-I (GLP-1): from L cells small intestine & ascending colon.
o Both incretins are rapidly deactivated by an enzyme called dipeptidyl peptidase 4 (DPP4).

103
In Capsule Series Endocrinology

o Since 2005, two new classes of drugs based on incretin action have been

approved for lowering blood glucose levels in type 2 DM :


I. lncretin mimetic (exenahae ) :
- Exenatide is injected subcutaneously twice daily, 1h prior to
meal
- It used in combination with metformin or sulfonylureas.
II. Dipeptidyl peptidase IV inhibitors (gliptins) :
e.g. Sitogliptin - Saxagliptin - Vildagliptin
- DPP-4 inhibitors work by blocking the action of DPP-4, an
enzyme which destroys the incretin hormone.
- The DPP4 inhibitors have an advantage in that they are
effective when administered orally, but they do not have as
potent effect as exenatide.
o Mode of action of incretins : increase postprandial active incretins

(GLP-1 & GIP), which in turn decrease blood glucose level by:
- Increasing insulin secretion.
- Decreasing glucagon.
Decreasing gastric emptying.
o Advantages :

- No hypoglycemia, no weight gain, well tolerated.


- HbA 1c reduction of 1% and seems to improve ~-cell function.
o Side effects :
Nausea, vomiting & diarrhea.
Pancreatitis.

104
In Capsule Serles Endocrinology
IV. Insulin

- Human insulin is a peptide hormone composed of 51 amino acids. It is


synthesized in the pancreas within the ~-cells of the islets of Langerhans.
- A typical blood level between meals is 8-11 µIU/ml. MCQ

- Action:

i. Rapid action :
L sulir stimulates 2 things to go Into the cells : glucose & K ©
ii. Gradual action :
1. CHO hypoglycemia
a. j glycogensis. b. J gluconeogenesis.
c. j peripheral utilization of glucose.
2. fat : - ii lipogenesis. - H lipolysis.
3. protein : anabolic.
- Prepar~t1ons : was dJ~covered in 1921.
Type Trade names Duration of action

1- Short-acting: - Neutral ( regular ). Onset : < ½ h.


(soluble) - Humulin R Peak : 2-4 h.
- Semilent. Duration : 6 - 8 h.
2- Intermediate : - lsophane (NPH) Onset : 2 h.
- Humulin N Peak : 6-8 h.
- Lente. Duration : 12 - 24 h.

3- Long-acting : - Protamine zinc insulin Onset : 6-8 h.


- Humulin L Peak : 12-24 h.
- Ultralente Duration : up to 36 h.

4- Insulin mixture : [ 30% short + 70% NPH ( mixtard)

- Indications ·

i. All type 1 DM.


ii. Type 2 OM not controlled with diet & oral hypoglycemic.
iii. During pregnancy, infection & surgery.
iv. OKA & HHNK.

105
In Capsule Series Endocrin ology
v. ii K ( Hyperkalemia ) .
vi. Insulin stimulation test in pan-hpypopituitarism.
- Dose : trial & error ( s. c. , 1 cm mixtard = 40 u )
i. Start with 20 - 30 U / d ( 20 U in non obese , 30 U in obese patient )
ii. The dose is gradually t ( by 5 u /d) until blood glucose is controlled.
111. 213 of dose before breakfast & 1h of dose before lunch.
iv. 213 of dose intermed iate & 1h of dose short acting insulin (mixtard)

- If mixtard is not available--+ 1/3 regular insulin+ 2/3 NPH can be mixed in
the syringe & injected as rapid as you can.

- Multiple SC insulin injection ( MSII) : in cases ofpoor glycemic control


o 75% of the dose is given as regular insulin before each meal.
o 25 % of the dose is given as intermediate insulin before sleep.

- Administration :
i. S.C.
11. Insulin pump ( Continuous S.C Insulin Infusion , CSII ) .
iii. IV infusion or IM : in case of DKA, HHNK
iv. Insulin pens.
v. Oral, nasal & rectal insulin --+ under trial ?

- Side effects :
1. Hypoglyc emia & hypoglyc emic coma.
ii. Allergy: use human insulin.
iii. Insulin resistance :
o obesity--+ mild resistance.
o antibodies against insulin.
1v. Insulin lipodystro phy: atrophy or hypertrophy of s.c. fat at
the site of insulin injections.

106
In Capsule Series Endocrinology
v. Insulin edema: Na & H20 retention¢ Hypertension.

v1. Weight gain.


vii. Pseudo insulin resistance ( Somogi effect) : nocturnal
hypoglycemia due to over insulin dose at night - t ii anti-
insulin hormones - t rebound hyperglycemia in the
morning. This condition is treated by reduction of the
evening insulin dose. MCQ

- Pramlintide : amylinomimetics"
11

o Is synthetic amylin analogue.


o Used in type 1 or insulin requiring type 2 DM.

o Given with current diabetic regimen.

o Mode of action : Pramlintide injected SC just before a meal slows gastric

emptying & suppresses glucagon but doesn't alter insulin levels.


o S/E : nausea , vomiting , frequent dosing injection ( 3 times daily )

V. TTI of the complications.

Q : criteria for good diabetic control .

• Lab.:
Fasting plasma glucose ( 90 - 130) mg/ell
Postprandial plasma glucose : < 180 mg/ell.
HbAlc < 7%.
• Clinical:
No symptoms of DM : .......
No symptoms of hypoglycemia : ......

Brittle diabetes : ( Labile diabetes)

This term is used to describe unpredictable fluctuations in blood glucose levels.


Theses cause either recurrent hypoglycemia or hyperglycemia.

107
In Capsule Series Endocrinology
Scheme for treahnent ofDM:

Diabetic patient

I
I I
Type1 Type 2

I I
Diet & insulin Diet

/ controlled I uncontrolled j

Oral hypoglycemic I
I
obese Non obese
I I I I
Biguanides sulfonylurea
I I
I I I I
uncontrolled controlled controlled uncontrolled

I I
Add sulfonylurea Add biguanides

I Uncontrolled I
I
Insulin

108
In Capsule Serles Endocrinology

I Hypoglycemia
11

Causes:

I. Fasting hypoglycemia : mnemonic EXPLAIN


o Exogenous drugs : insulin, oral hypoglycemic, alcohol.

o Pituitary insufficiency, hypothyroidism.

o Liver failure.

o Addison's disease.

o lnsulinoma : tumor of pancreatic ~ cell that produce too much

insulin.
o Non- pancreatic tumor( fibrosarcoma , mesothelioma ) that

produce Insulin like growth factor-I ( IGF-1)


II. Postprandial {reactive) hypoglycemia :
o Alimentary hypoglycemia : gastrectomy. this is due to rapid glucose

absorption with excessive insulin release ~ glucose metabolized

rapidly but insulin level remains high resulting in hypoglycemia.


o Idiopathic type.

o Early type II OM : The peak value of insulin occur after 2 hours or

later when the absorption of CHO from the intestine has been
completed.
Clinical picture : see hypoglycemic coma 5&5

Investigations :

l - Fasting plasma glucose : < 50 mg% in males , < 45 mg% in females.


2- Plasma level of insulin , thyroxin , cortisol.
3- The insulin / plasma glucose ratio : normally < 0.4 . It is higher in patients
with insulinoma.

109
In Capsule Series Endocrinology

4- Liver & renal function tests.


5- Abdominal US & CT for tumors.
Treatment:
I. Fasting hypoglycemia :
► The acute attack : see hypoglycemic coma.
► lnsulinoma : MCQ

• Surgical removal .
• Medical to prevent insulin release e.g. diazoxide, octreotide.
II. Postprandial hypoglycemia :
► Diet : Frequent small meals & avoid simple CHO.
► Drugs:
• Probanthine : 7 .5 mg 2 h before meals.
• Phenytoin : it inhibits insulin secretion.

We think so because all other people think so, or ... because we were told to think so, and
think we must think so ...
Rudyard Kipling

110
In Capsule Series Endocrinology

Hirsutism
Definition : ii growth of body hair in androgen dependant areas.
Etiology:
a. Ovarian :
1. polycystic ovary syndrome > 90% of cases.
2. ovarian tumors secreting androgens.
b. Adrenal:
1. congenital adrenal hyperplasia ( CAH ) .
2. Cushing's syndrome.
c . Drugs:
- minoxidil. - phenytoin.
- androgens. - diazoxide.
----------------------------------------------------------------------------------------------------------
Gynecomastia
Definition:
► Enlargement of the male breast, this is due to a decreased
androgens: estrogen ratio.
► It should be differentiated from fatty breast which lacks the
glandular element.
► It's unrelated to galactorrhea, breast enlargement is not
necessary to make milk.
Etiology : mnemonic : gynecomastia
- Genatic : Klinefelter syndrome
• Young boy (puberty)
• Neonate.
- Estrogen.
- Cirrhosis, cimetidine.
- Old age.
- Marijuana.
- Acromegaly.
- Spironolactone.
- Tumor ( testis , adrenal & bronchogenic carcinoma )
- INH.
- Alcoholism.

111
In Capsule Series Endocrinology

Definition :
Obesity is an increase of body adipose (fat tissue} mass.

Etiology:
• Genetic factors.
• Excessive caloric intake.
• Diminished caloric consumption : physical inactivity.
• Endocrinal :
- Cushing's syndrome.
- Hypothyroidism.
- Hypogonadism.
- lnsulinoma.
• Hypothalamic disorders : ( DI , polyphagia , obesity , hypersomnia }
• Drugs : Cortisone , Contraceptive pills , Insulin , sulfonylureas , anti-psychotics.

Diagnosis of obesity :

- Body Mass Index ( BMI ) : BMI Classification


18.5-25 normal weight
25 - 30 overweight

BMI = weight (kg) 30-35 class I obesity


35 - 40
height2(m2) class II obesity
Over 40 class UT obesity

- Skin fold thickness e.g. over triceps ( N: 20 mm in O, 30 mm in ~ ).

- Estimation of body fat : ( Waist/ hip ratio )

a. Central obesity : more fat in the upper body ( Apple


shaped) , associated with more morbidity .

b. Peripheral obesity : more fat in lower body ( pear


shaped ) , associated with less morbidity .

112
In Capsule Series Endocrinology

( Hazards of obesity )
1¢ardiovascularl:
o Hypertension.
o Atherosclerosis & coronary heart diseases.
o DVT & pulmonary embolism.

INeurologicall :
o Stroke. o Migraine.

JEndocrinologyj :
o Type 2 OM. o Menstrual disorders.

~ :
o Cholecystitis & gall stones. o Fatty liver.
o GERO. o Hernias.
IResplratoryl :
o Oyspnea (restrictive hypoventilation )
o Sleep apnea syndrome.

Qointsl :
o Osteoarthritis. o Back pain. o Gout.
IPsychologicall : depression & social stigmatization.
IMalignanc~: Increased incidence of: cancer colon, breast, prostate.
!Metabolic syndrome! : ( syndrome X )
• Obesity. • Hyperlipidemia. • Hypertension. • OM.

- Life style & diet modification: Low caloric, balanced diet with mild exercise.
- Drugs :
• Orlistat ( Xenica/ J : inhibits the pancreatic lipase so decrease fat absorption.
• Sibutramine ( Meridia) : Appetite suppressant.
- Surgery e.g. Gastric plication, liposuction: indicated with BMI > 40
- Treatment of the cause & complications.

113
In Capsule Series Endocrinology

Causes ofpreco cious puber ty


11
II
• In boys: puberty before age of 9 is conside red precoci ous.
• In girls : Breast develop ment or pubic hair with onset < 7 years.
I. lsosexual precocity :

a) lG111i'ill : (Gonadotropin - dependent precocious puberty) :


- It is caused by prematu re activati on of GnRH
- Causes :
• Idiopath ic ( constitutional ) .
• Hypotha lamic hamart oma : produce s pulsatile gonado tropin-
releasing hormon e (GnRH).
• Damag e to the inhibitory system of the brain : due to infection ,
trauma, or irradiati on.

b) m;Q,.jj:(Gonadotropin - independent precocious puberty)


- Androg en from testis or adrenal gland is increas ed but gonado tropin is low.
- Causes :
• Congen ital adrenal hyperplasia.
• Gonada l tumors (such as arrheno blastom a)
• Adrena l tumors.
• Germ cell tumor.
• McCune -Albrigh t syndrome : is a genetic disorder of bones, skin
pigmen tation and hormon al problems along with prematu re puberty .
• Exogenous sex steroid administration.

II. Heterosexual :
Refers to the prematu re develop ment of estrogenic features in boys, such
as breast develop ment.

114
In Capsule Series Endocrinology
I Diagnostic criteria of precocious puberty :
• Breast development in boys before appearance of pubic hair or
testicular enlargement.
• Pubic hair or genital enlargement (gonadarche) in boys with onset
before 9 years.
• Pubic hair (pubarche) before 8 or breast development (thelarche) in
girls with onset before 7 years.
• Menstruation (menarche) in girls before 10 years.

Criteria & causes of delayed puberty

• In girls:
- No breast development by 13 years, or
- No menarche by 3 years after breast development (or by 16).

• In boys:
- No testicular enlargement by 14 years,
- Lack of pubic hair by age of 15, or
- More than 5 years to complete genital enlargement.
Causes :
a) Constitutional delay: it runs in families. ( The most common)
b) Systemic disease, e.g. chronic renal failure, thalassemia major.
c) Undernutrition e.g. anorexia nervosa, zinc deficiency.
d) Hypothalamic defects e.g. Kallmann syndrome, craniopharyngioma
e) Pituitary defects e.g. hypopituitarism.
f) Gonadal defects e.g. Turner syndrome, Klinefelter syndrome, Testicular
failure due to mumps orchitis.
g) Endocrine disorders e.g. hypothyroidism, Cushing's syndrome.

115
In Capsule Series Endocrinology

11 II
Etiology:
I. Loss of weight inspite of good appetite:

i. increased caloric utilization :


o Thyrotoxicosis.

o Anexity.
o Severe exercise.

ii. Decreased intestinal absorption : malabsorption syndrome & diarrhea.


iii. Diabetes mellitus.
iv. Parasitic infection : Ascaris.

II. Loss of weight associated with decreased appetite:

i. Psychological disorders :
o Depression.

o Anorexia nervosa.
ii. Gastro-intestinal disorders : e.g.

o Esophageal diseases.
o Peptic ulcer.

o Inflammatory bowel diseases.

o Hepatobiliary diseases.

111. Systemic disorders: e.g.


o Malignancy.

o Cardiovascular diseases.

o Infection.

iv. Iatrogenic : Alcohol, Amphetamines: cause appetite suppression


leading to weight loss.
v. Smoking.

116
In Capsule Series Endocrinology

Endocrine syndromes affecting multiple glands


Multi le Endocrine Neo lasia (MEN) MCQ

MEN-1 :
o Parathyroid adenoma.
o Pancreatic endocrine tumors : e.g. Gastrinoma, lnsulinoma.
o Pituitary tumors: e.g. Prolactinoma, Acromegaly.

MEN-2A:
o Medullary carcinoma of thyroid.
o Pheochromocytoma.
o Hyperparathyroidism.
MEN-2B :
o Medullary carcinoma of thyroid. ( 100% ) : very aggressive with most patients.
o Pheochromocytoma. ( 50%)
o Mucosa! neuroma ( 100%) : painless nodules on the lips or tongue.
There may be enough neuromas in the body of the lips to produce
enlargement and a "blubbery lip" appearance. MCQ

Pol landular Autoimmune S dromes

- Association of multiple autoimmune endocrine disorders.


- There are 2 different syndromes :
Type I:
- Characterized by triad of
o Mucocutaneous candidiasis.
o Autoimmune hyperparathyroidism.
o Adrenal failure.
- Other autoimmune features :
o Primary hypogonadism.

117
In Capsule Series Endocrinology
o Autoimmune hypothyroidism.
o Alopecia.
o Autoimmune chronic hepatitis.
o Pernicious anemia.

Type II : more common


- Common features :
o Adrenal failure.
o Autoimmune thyroid disease e.g. Grave's disease.
o Type 1 DM.
- Other features :
o Primary hypogonadism.
o Myasthenia gravis.

o Pernicious anemia .
o Alopecia.
o Vitiligo.
o Caeliac disease ( gluten sensitive enteropathy ) .

First they ignore you, then they laugh at you, then they fight you, then you win.
Mahatma Gandhi

118
In Capsule Series Endocrinology
Dyslipidemia
- Dyslipidemia is elevation of plasma cholesterol, triglycerides {TGs), or both,
or a low high-density lipoprotein level that contributes to the development of
atherosclerosis.
- Cholesterol & triglycerides are the major circulating lipid. Both are water insoluble
& are transported in the blood stream as macromolecular complexes, called
lipoproteins.

- Five principle types of lipoproteins are found in the blood :


1. Chylomicrons :
• Made by : the small intestines postprandially.
• Absorbed into: the lymph vessels, then --> moves into the blood.
• Rich in : TGs.
• Function : Deliver TG's to body cells to be used as fuel.

2. VLDL (Very Low Density Lipoproteins) :


• Made in : the liver from excess dietary carbohydrate and protein.
• Secreted into : the bloodstream.
• Rich in : TGs
• Function: Deliver TGs to body cells.

3. IDL ( Intermediate Density Lipoproteins) : These are produced after removal


of triglycerides from VLDL.

4. LDL ( Low Density Lipoproteins ) :


• Made in : the Liver as VLDL
• Arise from: VLDL once it has lost a lot of its TG's.
• Secreted into : the bloodstream.
• Rich in : Cholesterol ( main carrier of cholesterol ) .
• Function : Deliver cholesterol to all body cells

5. HDL ( High Density Lipoproteins) : " .t!_ealthy cholesterol"


• Made in : the Liver and Small Intestine.
• Secreted into : the bloodstream.
• Function : Pick up cholesterol from body cells and take it back to the
liver "reverse cholesterol transport".
• Potential to help reverse heart disease.

119
In Capsule Series Endocrinology
Health risks of dysllpidemia :

1. There is a strong association between both total & LDL cholesterol

concentration & coronary artery disease, peripheral vascular


disease & cerebrovascular disease. Low HDL levels are also
associated with increased cardiovascular risk.

2. Triglycerides levels more than 1000 mg/di are associated with


increased risk of acute pancreatitis.
Causes of dyslipidemia :
I. Primary causes :
Primary causes are single or multiple gene mutations that result in
either overproduction or defective clearance of TG and LDL
cholesterol, or in underproduction or excessive clearance of HDL.
II. Secondary causes :
• Sedentary lifestyle with excessive dietary intake of saturated
fat, cholesterol & alcohol.
• Diabetes mellitus.
• Hypothyroidism.
• Nephrotic syndrome.
• Chronic renal failure & dialysis.
• Obstructive jaundice.
• Drugs : ~ blockers, Thiazides, estrogen, cortisone.

• Diabetes is an especially significant secondary cause because patients


tend to have an atherogenic combination of high TGs; high LDL and
low HDL.
• The cardiovascular risk in women with DM is similar to that of men.
• Patients with type 2 diabetes are especially at risk due to combination
of obesity & poor control of diabetes.

120
In Capsule Series Endocrinology
Screening :
• Patients at risk should be screened e.g. presence of xanthelasma,
obesity, DM, hypertension, acute pancreatitis, coronary artery
disease, ...
• Ideal levels :
Total cholesterol < 200 mg/di
LDL cholesterol < 100 mg/di

HDL cholesterol ~ 60 mg/di


Triglycerides < 150 mg/di

Management :
I. General measures :
• Treatment of secondary causes if possible.
• Dietary change : include decreasing intake of saturated fats and
cholesterol, increasing the proportion of dietary fiber, and complex
carbohydrates; and maintaining ideal body weight.
• Regular exercise.
• Needless to say stop smoking.
II. Drugs: are the next step when lifestyle changes are not effective.
A) If the predominant disturbance is cholesterol level :
~ Statins :
o Lovaststin, Atrovastatin : 20-80 mg/d
o Statins inhibit hydroxymethyl-glutaryl CoA reductase, a key
enzyme in cholesterol synthesis.
o Side effects: muscle toxicity & hepatotoxicity.
~ Cholesterol absorption inhibitors ( e.g. ezetimibe)
o Inhibit intestinal absorption of cholesterol.
o Side effects: diarrhea, abdominal pain, back pain.

121
In Capsule Series Endocrin ology
& Nicotinic acid : inhibit lipid synthesis in the liver.
& Bile acid binding resins (Bile acid sequestrants)
e.g. cholestyramine. Safe in pregnanc y.
B) If the predominant disturbance is triglyceride level :
-::s. Fibrates:

- e.g. gemfibrozil. They mainly promote lipoprotein lipase action.


- Are drugs of choice, reduce TGs by about 50%.
~ Omega 3 fatty acids : ( marine fish oil )
o Reduce hepatic VLDL secretion
o Omega-3 fatty acids can be a useful adjunct to other therapies.
o Side effects : eructatio n and diarrhea

122
In Capsule Series Endocrinology

Endocrinal emergencies :
a . Diabetic comas.
b. Thyroid emergencies :
i. Thyrotoxic crisis.
ii. Myxoedema coma.
c . Parathyroid emergencies:
i. Acute hypercalcemia.
ii. Tetany.
d. Adrenal crisis.

Endocrinal manifestations of systemic diseases :


a. Liver cell failure : (j ADH & j aldosterone )
i. In i : gynecomastia , ~ pubic hair, l libido.
ii. In ~ : breast atrophy , i pubic hair , l libido.
b. Chronic renal failure :
i. 2ry & 3ry hyperparathyroidism.
ii. Hyperprolactinemia.
c. T.B. ( miliary) : Addison's disease.
d. Heamochromatosis :
i. DM.
ii. Hypogonadism.
e. Sarcoidosis :
i. DI.
ii. Addison 's disease.
f. Malabsorption: 2ry hyperparathyroidism.
g . Bronchogenic carcinoma,Hepatoma& hypernephroma
Endocrinal manifestations of paramalignant syndrome.

Endocrinal causes of HTN :


a. Pituitary acromegaly.
b. Thyroid - hypothyroidism .--► j diastolic BI.P.
- hyperthyroidism - i systolic Bl .P.
C. Parathyroid hyperparathyroidism.
d. Pancreas DM.
e. Suprarenal : Conn's, Cushing syndrome, Pheochromocytoma.

123
In Capsule Series Endocrinology
Endocrinal causes of hyperlipidemia :
a. Hypothyroidism.
b. DM.
C. Obesity.

Endocrinal causes of osteoprosis :


a. DM.
b. Hyperthyroidism.
c. Hyperparathyroidism .
d. Cushing syndrome.
e. Osteomalacia.
Macro & micro-vascular complications of OM :
a. Macro:
1. HTN .

ii. Atherosclerosis.
iii. Non healing ulcers & amputations.
b. Micro : ( Triopathy )
i. Retinopathy.
ii. Nephropathy.
iii. Neuropathy.
Immune mediated endocrinal diseases :
a. Type l OM.
b. Addison's disease.
c. Grave's disease, Hashimoto thyroditis.
Iatrogenic endocrinal diseases :
o Iodine
o Lithium
-- hyperthyroidism.
hypothyroidism.
o Amiodarone thyroid dysfunction.
o Ketoconazole hyoadrenalism.
o
o
Metoclopramide
Sympathomimetic
--Hyperprolactinaemia .
picture like thyrotoxicosis.
o
o
ACE s inhibitors
Steroid therapy
-- hypoldosteronism.
DM & Cushing syndrome.

o
o Thyroxin therapy
Vitamin D -
- thyrotoxicosis ( factitious ) .
hypercalcemia .
o Insulin & oral hypoglycemic - hypoglycemia.

124
In Capsule Series Endocrinology

roach to atient with ol ria & ol de sia :

I
I
Specific gravity ii Specific gravity H
I I
If there is +ve sugar H20 deprivation for 12 h.
in urine

I I
I
DM No response Good response

I I
DI Psychogenic
polydepsia
I
Give ADH

I
I
Good response No response

I
Pituitary DI Nephrogenic DI

roach to atient with h oth oidism :

Determ ine TSH & free T4


I I

I I I I
Free T4 : normal Free T4 : ! Free T4: ! Free T4 : j
TSH : normal TSH :i TSH :normal or l TSH :i
I I I I
I euthyroid I 1ry 2ry Thyroid hormone
hypothyroidism hypothyroidism resistance

125
In Capsule Series Smater not Harder

I. SAQ (Short Answer Questions)


For cases, MCQ & oral exam
1- A middle aged woman presented with constipation, hoarseness of voice and cold
intolerance. What is your probable diagnosis?
Hypothyroidism.

2- A 24 year old pregnant woman at 10 weeks gestation with Hashimoto's


hypothyroidism and ask you for advice about any change in her thyroid medication.
What is your advice?
o She will likely have to increase her thyroid hormone dose by 33% during

pregnancy. (Double the daily dose two days each week).

o TSH levels should be monitored monthly.

During pregnancy, the dose of thyroid medication often requires a 33% increase in total dose to keep
TSH within normal range and, if possible, in the low normal range. High TSH is associated with lower
intelligence in the future for the child.

3- What is the most common cause of primary hypothyroidism?


Iodine deficiency e.g. in mountainous regions

4- What are the most common causes of transient hypothyroidism?


o Subacute thyroiditis

o Postpartum thyroiditis

5- How can you differentiate primary and secondary hypothyroidism?


points primary secondary
Clinically Weight gain Weight loss
Myxedematous Lean and thin
Biochemically j TSH ! TSH
6 - What questions will you ask if you suspect hypothyroidism?
o Do you notice change in your weight? (weight gain)
Hypothyroidism can
o Do you notice any change in your voice? (hoarseness of voice)

o Do you prefer hot or cold? (cold intolerance)


telephone &
be diagnosed by over

o What is your bowel habit? (constipation)

o What is your menstrual history (in female only © )? Menorrhagia.


126
In Capsule Series Smater not Harder

7- If single investigation is asked for hypothyroidism, which one?


TSH

8- What are the ECG findings in hypothyroidism?


Sinus bradycardia with low voltage ECG

9- How will you treat the patient?


o Lifelong thyroxine therapy

o 50 µg (one tab.) daily for 3 weeks (given in morning in empty stomach) 100 µg

for 3 weeks. Then maintenance dose at 100 - 150 µg daily. 6 weeks should pass

before repeating thyroid function tests following a dose change.

o It is necessary to measure thyroid function every 1-2 years.

10- In which conditions initial small dose (25 µg) is used?


o Patients with IHD

o Elderly patient

11- What is myxedema madness?


Psychotic condition developed in a patient with hypothyroidism.

12- What are the clinical criteria of remission of hypothyroidism?


o Reduction of weight

o Decreased edema

o Relief of constipation

o Husky voice takes about 3-6 months to normalize.

13- What is the most common cause of hyperthyroidism?


Grave's disease

14- What are the common causes of transient hyperthyroidism?


o Subacute thyroiditis

o Postpartum thyroiditis

15- What is the pathogenesis of weight loss in hyperthyroidism?


Increased metabolic rate

127
In Capsule Series Smater not Harder

16- What questions will you ask if you suspect hyperthyroidism?


o Do you notice any change in your weight? (weight loss)

o What is your appetite? (increased appetite)


o Do you prefer hot or cold? (hot intolerance)
o What is your bowel habit? (diarrhea)
o What is your menstrual history (in female only)? Scanty menses
17- Enumerate eye signs in hyperthyroidism?
See endocrinology book ... (mnemonic: DR Must be Very Simple)
18- What advice would you give to the patient on anti-thyroid drugs?
To seek immediate medical attention if they develop unexplained fever or sore

throat (agranulocytosis, baby)


19- What are the clinical presentation of subacute thyroiditis (de Quervain
thyroiditis)?
Viral upper respiratory infection followed by tender enlarged thyroid, can begin

with hyperthyroid symptoms, then hypothyroid symptoms.


20- What are the possible side effect of radioactive iodine ablation?
o Hypothyroidism
o Thyrotoxic crisis secondary to the release of thyroid hormone into the blood.
21- What is the mortality rate of thyroid storm?
Up to 50%
22- What is subclinical hypothyroidism?
Elevated TSH levels but with normal thyroid hormone levels and with no clinical

symptoms.
23- What is the most common type of thyroid cancer?
Papillary cancer
24- Which type of thyroid carcinoma is associated with MEN 2 & 3 (2B)?
Medullary cancer
25- Identify the TWO most common causes of hypercalcemia
o Primary hyperparathyroidism 55%

o Malignancy 30%

128
In Capsule Series Smater not Harder

26- A 40 year old male presents with progressively worsening headache and
sweating for 6 months. On query he admitted history of clash with door and
bystander, increasing size of shoes and ring. What is your diagnosis?
Acromegaly

2 7- What is acromegaly?
It is a clinical condition characterized by hypersecretion of GH after closure of bony

epiphysis. (aero means periphery, megaly means large)


28- What happens if it occurs before closure of epiphysis?
Gigantism

29- What is the initial investigation in a case of acromegaly?


IGF-1 confirmed by glucose suppression test
30- What is the explanation for raised prolactin level in a case of acromegaly?
It is explained by damage to pituitary stalk ¢ loss of inhibitory regulation of

prolactin secretion by the hypothalamus. Plus prolactin & GH are secreted from

acidophil cells of anterior pituitary.

31- Plain X-ray skull in a case of acromegaly. What about it?


o Widening of sella turcica and erosion of posterior clenoids.

o Widening of nasal sinuses.

o Prominent supraorbital ridges & occipital protuberance.

o Prognathism with wide separation of teeth.


32- What is the first line therapy in a case of acromegaly?
o Surgery: Transsphenoidal resection.

o Recently : Stereotactic surgery (stereotaxy) : minimally invasive form of surgical

intervention with assistance of image guidance (MRI).

o Radiotherapy is the 2nd line therapy. Medical is the 3rd •

33- What malignancy are patients with acromegaly at increased risk for?
Cancer colon

34- What cranial nerve can be affected by prolactinoma?


Cranial nerve Ill
129
In Capsule Series Smater not Harder
35- What is the Cushing's disease?
Cushing's syndrome due to pituitary cause e.g. pituitary adenoma
36- What are the most common cause of Cushing syndrome?
Exogenous corticosteroids

37- A patient with bronchial asthma was under good control with medication. He
went to see his daughter far away from his house but forget to bring his medicines.
Without medication he was free from breathless but in the next morning he
developed nausea, vomiting, and collapsed. What is your diagnosis?
Adrenal crisis.

38- What is the finding of chest X-ray in Addison's disease?


Small heart (microcardia).
39- Mention some endocrinal emergencies?
o Hypoglycemia

o OKA

o Pituitary apoplexy

o Thyrotoxic crisis

o Myxedema coma

o Addisonian crisis

o Severe hypercalcemia
40- What are the presentation of DM?
o Asymptomatic: detected on routine investigation.

o Acute presentation with classic triad polyuria, polydipsia and polyphagia with
weight loss.

o Non-specific symptoms : lack of energy, visual blurring, pruritis vulvae ...

o With complications : OKA, microangiopathy.


41- What are the early symptoms of DM?
o The three polys: polyuria, polydipsia & polyphagia with weight loss
42- What is the first line treatment for type 2 DM?
o Metformin

130
In Capsule Series Smater not Harder
43- How can you differentiate hypoglycemic coma and DK.A at bed side?
HYPOGLYCEMIC COMA DKA
History 0 No food 0 Little or no insulin
0 Vigorous exercise 0 Infection
0 Insulin overdose
Onset 0 In good previous health 0 Ill health for several days
before the last insulin 0 He comes with his wife
injection. ~
0 His GF usually brings him.
Symptoms 0 Palpitation, sweating, 0 Polyuria, thirst,
hunger abdominal pain,
vomiting
Signs 0 Moist skin and tongue 0 Dry skin and tongue
0 Full pulse 0 Weak pulse
0 Normal or raised BP 0 Low BP
0 Shallow or normal 0 Rapid deep respiration
breathing 0 Diminished reflexes
0 Hyperreflexia 0 Smell of acetone.
0 No smell of acetone.

44- How can you differentiate between diabetic and hypertensive retinopathy?
Diabetic retinopathy Hypertensive retinopathy
Silver wiring (arteriolar Absent Present
thickening, tortuous, and
increased reflectiveness.
Arteriovenous nipping Absent Present
Hemorrhage Dot and blot due to rupture F1ame-shaped
ofmicroaneurysms.
Papilledema Absent Present in grade 4
Neovascularization Present in proliferative Absent.
retinopathy.

45- What type of fatal fungal infection can diabetic get?


o Mucor, especially sinusitis (Mucormycosis is a life-threatening fungal infection)

131
In Capsule Series Smater not Harder

46- Mention three life threatening infections in diabetics.

l Malignant otitis externa.

! Rhino-cerebral mucormycosis.
! Emphysematous cholecystitis.
47- What preventive measures are recommended to minimize diabetic
complications?
o HgAlc < 7

o Lipid control : LDL< 70, TG< 150

o BP< 130/80

o Annual foot examination

o Check for proteinuria

o Annual funduscopic examination.


48- Criteria for diagnosis of prediabetes
o IFG (Impaired fasting glucose) : 100 -125 mg% (S.6-6.9 mmol/L)

o IGT {Impaired glucose tolerance) : 140 - 199 mg% (7.8-11.0 mmol/L)


o HbAlc:5.7-6.4
Prognosis:

l IGT (impaired glucose tolerance) leads to type 2 OM in about 34% of cases over 5 Y.

l IGT & IGF together : lead to type2 OM in 65% of cases.


l Prediabetics are at high risk for cardiovascular complications.
49- Diagnostic criteria of DM
Four ways to diagnose diabetes are possible, and each must be confirmed, on a
subsequent day.

A. FPG ~ 126 mg/dl (7.0 mmol/L)

B. 2-hr PG ~ 200 mg/dl (11.1 mmol/L) during OGTT {75-g)

C. AlC ~ 6.5% {48 mmol/mol)

D. Random PG ~ 200 mg/dl {11.1 mmol/L) In individuals with symptoms of

hyperglycemia e.g. polyuria, polydipsia, ..

132
In Capsule Series Smater not Harder

50- Genetic syndromes associated with diabetes


o Down's syndrome.

o Turner's syndrome.

o Feridreich's ataxia

o Myotonia dystrophy.
51- Why might severe headache, muscular inability, loss of consciousness and
seizures occur with SIADH?

Cerebral edema associated with hyponatremia

52- What are the 4 hallmark characteristics of DI?

o Polyuria
o polydipsia
o dehydration
o hypematremia

Enjoy! ~

133
In Capsule Series Smater not Harder
II. Enumerate : written & oral

1. Features of polyglandular autoimmune syndrome (type 1 & 2)

2. Causes of Cushing syndrome. (ACTH dependent, ACTH independent)

3. Causes of secondary diabetes.


4. Causes of stunted growth ( Dwarfism, short stature ) . P17
5. Endocrinal causes of short stature. P 17

6. Causes of hypercalcernia.

7. Causes of hypocalcemia.

0 Chronic renal failure.

0 Vitamin D deficiency.

0 Hypoparathyroidism.

0 Hyperphosphatemia.

0 Acute pancreatitis.

8. Causes of hypoglycemia in adult. Mnemonic: EXPLAIN


a) Exogenous drugs : insulin, oral hypoglycemic, alcohol . ..... .
b) Pituitary insufficiency..... .
c) Liver failure ....
d) Addison's disease ....
e) Islet cell tumor (insulinoma), Immune hypoglycemia (anti insulin receptor
antibodies in Hodgkin's disease) ....
f) Non pancreatic neoplasm especially retroperitoneal fibrosarcoma.

9. Causes of hirsutism.

10. Causes of gynecomastia.

11. Causes of tetany.

12. Causes of hypoadrenalism.

13. Causes of hypothyroidism.

14. Causes of obesity.


134
In Capsv/e Series Smater not Harder

15. Causes ofloss of weight inspite of good appetite. P 116

16. Causes of nephrogenic DI.

1 7 . Causes of Hyperprolactinemia. . . . . . . . . 3X5

18. Pharmacological causes of Hyperprolactinemia : 4 anti+ others

19. Endocrinal causes of hypertension.

20. Side effect of metforrnin.

2 1. Endocrinal causes of abdominal pain.

22. Acquired causes of hyperlipidernia P120

23. Criteria for good diabetic control.

24. Laboratory tests for hyperthyroidism.

25. Acute diabetic complications: Corna, infection


26. Clinical stage of diabetic nephropathy
27. Skin complications ofDM
28. Cardiovascular, neurological, urogenital complications of DM.
29. Causes of diarrhea in diabetes mellitus. ABCD

o Autonomic neuropathy.

o Bacterial overgrowth.

o Celiac disease (autoimmune).

o Drugs : e.g. metformin induced diarrhea.

30. Endocrinopathies ,that may be associated with hyperglycemia.

o Acromegaly

o Cushing's syndrome

o Pheochromocytoma

o Thyrotoxicosis.

o Somatostatinoma, glucagonoma.

31- Causes of precocious & delayed puberty


135
In Capsule Series Smater not Harder

jll(. GIVE A SHORT ACCOUNT oNj

1. Management of DKA

2. Clinical picture & treatment of hypoglycemic coma.

3. Different types of insulin.

4. Indications, adverse effects of insulin.

5. Diabetic nephropathy.

6. Diabetic neuropathy

7. Microvascular complications of DM

- Retinopathy - nephropathy - neuropathy

8. Oral hypoglycemic drugs

9. Management of type II diabetes ( drug details needed)

1a.Diagnosis , treatment of hyperthyroidism.

11. Diagnosis & treatment of hypothyroidism.

12. Thyroid emergencies

o Thyrotoxic crisis ( thyroid storm )

o Myxedema coma

13. Clinical picture, investigations & treatment of primary hyperparathyroidism.

14. Management of Cushing's syndrome.

15. Causes, Clinical picture, Investigations & treatment of chronic adrenal

failure.

16. Acute adrenal failure

17. Hazards of obesity.

18. Diagnosis, treatment of acromegaly.

136
In Capsule Series Smater not Harder

19. Pituitary apoplexy.

o Def : sudden enlargement of the pituitary by hemorrhage into the

tumor

o C/P:

- Pressure manifestations : Headache , vomiting , visual defect.

- Panhypopituitarism : ..... .

o Investigations :

- Lab : evaluate pituitary hormones , electrolytes , glucose.

- Imaging : CT , MRI ( MRI is the most sensitive )

o TTT : Replacement therapy , Transsphenoidal surgical decompression

of the tumor. See panhypopituitarism

20. Sheehan syndrome.

21.Management of tetany.

22. DD of polyuria.

23. DD of short stature.

24. DD of hypercalcemia.

25. DD of skin pigmentation.

Smile!

It is only an exam!

137
In Capsule Series Smater not Harder

1- Acrome galy
o Adult
o Glove, ring, or shoe size : acutely j Don't forget 5 endocrinal
manifestations associated
o Coarsening of facial features, prognathism.
with acromegaly.
o Headache and visual affection.
o Dx: IGF-1, glucose suppressio n test, CT/MRI
o Tx:
Surgery is first choice.
- Radiation is the 2 nd .
- Medical : somatostatin analogue.
2- Prolact inoma :
o Headache, diplopia, CN III palsy.
o In male : Gynecomastia, impotence.
o In female : secondary amenorrhea, single with galactorrhea.
o Manifestations of panhypopituitarism may occur, compression baby.
o Etiology : 3 X 5 (5 physiological, 5 pharmacological, 5 pathological)
o Dx : CT/MRI, prolactin level > 250 ng/ml
o Ts : first line : dopamine agonist
o Large tumor or refractory : transsphenoidal surgery resection.
3- Sheeha n syndrom e (panhypopituitarism after sever postpartum hemorrhage )
o Female developed postpartu m hemorrhage, she could not lactate her baby
o General weakness and fatigability
o Dx:
- Low thyroxin, cortisol, TSH, ACTH, FSH, LH
- MRl is indicated to exclude mass lesion in pituitary gland
o Tx: Hrmone replacement. Don't forget Cortisone replacement before
thyroxin.

138
In Capsule Series Smater not Harder

4- Cushing's syndrome:
o Usually iatrogenic or due to pit. Adenoma (ACTH dependent, Cushing
disease).
o Recently discovered OM
o Increased facial fullness (moon face) with truncal obesity.
o Hypertension.
o Lab : jNa, lK
o NB : Cushing with loss of weight, heavy smoker : consider paramalignant
syndrome in a case of bronchogenic carcinoma.
o Ox: 24h urine cortisol and dexamethasone suppression test.
o Tx : surgical with postoperative cortisone replacement.
o Medical : mitotane, metayrapone, aminoglutethimide, ketoconazole.

5- Chronic adrenal failure :


o Usually female, suffers from easy fatigability, loss of weight.
o Skin pigmentation may occur in primary type.
o Hypoglycemia, hypotension
o tNa, i K, eosinophilia
[Chronic adrenal failure + type 1 OM + thyroid disease= polyglandular syndrome type 21

o Ox : Low morning cortisol level < 5 µg/dl is highly suggestive with ACTH
stimulation test.
o Tx : 3 S (sugar, salt, steroid)

6- Acute adrenal failure


o As a complication of chronic adrenal failure in stress situations : infection,
surgery
o Sudden withdrawal of cortisone : e.g. patient with SLE or bronchial asthma.
o Patient with panhypopituita1ism : treated by thyroxin without cortisone.

139
In Capsule Series Smater not Harder

7- Pheochromocytoma:
o Patient with acute onset episodes of hypertension, palpitation, flushing,
sweating.
o Few months ago: his BP was normal
o Dx : j catecholamine, CT adrenal
o Tx:
Surgical excision after preoperative a blocker
Ca channel blocker for hypertensive crisis
- Combined a. & ~ blockers for inoperable patients
- Meta Iodo Benzyl Guanidine (MIBG) for metastatic disease.

8- Hyperthyroidism : Graves disease


o Young female.
o Diffuse painless goiter.
o Manifestations of hyperthyroidism: tachycardia, isolated systolic.
hypertension, atrial fibrillation, tremors, weight loss with j appetite.
o Infiltrative ophthalmopathy & dermopathy (brawny pluritic hairy, non-
pitting edema on the shins of tibia.
o Dx: ! TSH, j free T4, T3, TSI (Thyroid stimulating immunoglobulin).

9- Subacute thyroiditis :
o Tender enlarged thyroid. Sometimes the pain can radiate to the jaw.
o Typically following viral upper respiratory infection (de Quervain's
thyroiditis) or autoimmune (postpartum thyroiditis).
o It begins with hyperthyroid symptoms, then hypothyroid symptoms.
o Dx : elevated ESR, antithyroid antibodies in a case of postpartum
thyroiditis (anti-thyroid peroxidase, anti-thyroglobulin Ab).
o Tx : typically self limited within weeks or months. Just cortisone or
aspirin in severe cases & ~ blocker to decrease palpitation in some cases
during hyperthyroidism phase.

140
In Capsule Series Smater not Harder

10- Hypoth yroidism :


o Female, obese, hypertens ive, bradycard ia, husky voice, constipation,
mental slowness, puffy dull face with dry skin, thin lateral eyebrows.
o Pericardia! effusion (increased cardiotho racic ratio) may occur.
o Causes include Hashimot o's thyroiditis and subacute thyroiditis.
o Dx: j TSH, t free T4 & T3. Antithyroid peroxidase Ab in Hashimot o's
& postpartu m thyroiditis.
o Tx : lifelong thyroxin in Hashimot o's thyroiditis.

11- Hyperp arathyr oidism :


o Hypercalc emia (N 8.5-10.5 mg%) and hypophos phatemia (N 2.5 - 5 mg%)
& may be asymptom atic patient.
o CIP : Disease of bone, renal stone & others .... See endocrino logy book

12 - Hypopa rathyro idism :


o Cbvostek 's and Trousseau 's signs are positive
o Paresthes ia, tetany, seizures, muscle cramps.
o CT scan may reveal bilateral symmetric al basal ganglia calcification : chorea
or parkinson ism are present in 20 - 30% of patients with basal ganglia
calcification.
o Hypopara thyroidism is the most common cause of pathological basal ganglia
calcification.
o Dx : low Ca, low PTH, j phosphoru s
o Tx : Ca and calcitriol (vitamin D) with follow up.

13- Diabete s mellitus : 4 scenarios


a. Type 2 DM : Obese patient,> 40 y, Classic triad (3Ps), HbAlc > 6.5%
b. Impaired glucose tolerance :
o Patient (usually obese), presented with fasting glucose: 100 - 125
mg%, 2 H post-prandial 140 - 199 mg% or Hb Ale: 5.7 - 6.4%
o Many patients will progress to DM, recommen d weight loss and
exercise to prevent.
141
In Capsule Series Smater not Harder

c. OKA:
1. OKA in OM type 1: Young patient (e.g. 14 y), presented to ER with
abdominal pain, repeated vomiting, polyuria and polydipsia in last
month. Metabolic acidosis with increased AG, urine is positive for
acetone.
11. OKA in DM type 2 : Poorly controlled DM type 2, with infection e.g.
fatal fungal infection mucormycosis (blocked nose, facial pain, proptosis
, vision loss).
o Dignosis of DKA requires 3 things :
✓ DM (type 1 or 2, but type 1 at higher risk)
✓ Ketosis : blood & urine ketones, blood hydroxybutyrate
✓ Acidosis : metabolic acidosis with high anion gap.

d. Diabetic nephropathy :
o Long standing DM
o Proteinuria.
o Hypertension & the 3 microvascular complications (Retinopathy,
neuropathy, nephropathy)
o US kidney: Normal sized, hyperechogenicity.

BU1' l'f DOES MOVE ,,


GAUl,EO

142
In Capsule Series Smater not Harder

Cases
1- Male patient of 42 years old, presented with parasthesia in his hands & feet of
6 months duration. During this period he was admitted to the emergency hospital
with disturbed level of consciousness and left sided body weakness. He was fully
investigated and received medical treatment and discharged well after one week.
He advised to be kept on special diet regimen and specific drug ( injection ) used
daily. He gave history of palpitation & fainting attacks on standing. Clinical
examination revealed cachexia, white patches over the tongue. Pulse 98b/m, BP
standing 140/85 and recumbent 180/ 105 mmHg. Heart accentuated S2 with
ejection systolic munnur over the mitral area, Liver was enlarged 2 fingers with
rounded border but not tender. Neurological examination revealed superficial &
deep sensory affection.
a) What is the provisional diagnosis?
b) Give an explanation to symptoms & signs in this patient.
c) If there are other complications to the primary disease ?
d) What is the pathogenesis of these complications ?
e) What are the investigations suggested for diagnosis ?

a) This is a case of complicated DM and hypertension.


b)
✓ Peripheral neuropathy : parathesia of hands & feet
✓ Autonomic neuropathy : fainting & postural hypotension & tachycardia.
✓ Monilial infection : white patches over the tongue.
✓ Accentuated S2 is due to hypertension
✓ Ejection systolic murmur is due to functional AS
c) complications ofDM see endocrinology book
d) pathogenesis of DM & its complications
✓ Metabolic
✓ Recurrent infections.
✓ Vascular: micro & macro angiopathies
✓ Biochemical: accumulation of sorbitol ~ neuropathy.
e) Investigations : see endocrinology book

143
In Capsule Series Smater not Harder

2- Female patient aged 32 years presented with backaches and fatigability with
polyuria. She was overweight. BP 140/100 mmHg. Fasting blood glucose 200mg%

a) Mention 4 signs that help the diagnosis.


b) What is the possible diagnosis and differential diagnosis ?
c) How would you treat such case ?

a)
✓ Plethoric face.

✓ Hirsutism and acne may be apparent in female

✓ Pigmentation & purpura.

✓ Truncal obesity with stria rubra.

✓ Moon face : rounded face , bloated cheeks.

✓ Muscle wasting & weakness.

b)
Possible diagnosis : Cushing's syndrome
DD:
✓ Obese hypertensive diabetic patients.

✓ Female taking oral contraceptive pills.

✓ Chronic alcoholism.

✓ Frolich's syndrome : trunk obesity, hypogonadism, polyphagia.

✓ Differentiate between lry & 2ry Cushing. ( . .... )

c) Treatment of Cushing : see endocrinology book p 53

144
In Capsule Series Smater not Harder

3- A 40-year-old man is seen because of headaches, muscle aches, and chronic


low back and joint pain. As he enters the office, you notice his coarse facial
features, frontal bossing, and large jaw. When you shake his hand, you find he has
large, doughy, sweaty palms and, when he smiles, you note his teeth are widely
spaced. He has not seen a physician in 10 years and is taking no medications. His
back and joint pain have been worsening for 6 years, but his headaches started 6
months ago. His physical examination findings are significant for a BP of 150/100
mm Hg, pulse of 60 per minute, and respiratory rate of 12 per minute.
He returns in 2 weeks with old photographs that confirm a change in his physical
appearance over time, and the laboratory test results confirm your clinical
rmpress1on.
a) What is your initial diagnosis in this patient?
b) Besides the back and joint pain and the headaches, what other symptoms
would you look for to confirm or refute your diagnosis?
c) Besides the physical features you observe initially, what other abnormalities
would you look for on physical examination?
d) What laboratory tests should be performed initially?
e) What additional testing should be performed once the initial laboratory
results are known?
f) What is the preferred treatment in this patient?

a) What is your initial diagnosis in this patient?


Acromegaly.

b) Besides the back and joint pain and the headaches, what other
symptoms would you look for to confirm or refute your diagnosis?
Other symptoms to look for in this patient include a change in glove, ring, and
shoe sizes, spaces between the teeth, decreased libido and impotence, sweating,
new snoring, polyuria, polydipsia, and a change in vision.

145
In Capsule Series Smater not Harder

c) Besides the physical features you observe initially, what other


abnormalities would you look for on physical examination?
Other physical features to look for in this patient include thick coarse skin,
enlarged extremities and organs, entrapment neuropathies, visual field
abnormalities, and decreased body hair and testicular size. Old pictures would
confirm the clinical suspicion.

d) What laboratory tests should be performed initially?


Initial laboratory tests in this patient would consist of the measurement of
IGF-1 (somatomedin C) and fasting GH levels. If the levels are elevated it
would suggest the diagnosis of acromegaly, which would be confirmed if
the GH levels did not suppress to less than 2 ng/mL in response to a
glucose load ( suppression test )

e) What additional testing should be performed once the initial laboratory


results are known?
I. Investigations for the function of GH e.g. fasting blood sugar test should
be performed to rule out diabetes.
2. Pituitary tests should include measurement of the prolactin, FSH, LH.
3. MRI scan can show the extent of the tumor, and formal visual field
testing should be performed.
4. Echocardiography and colonoscopy should be performed to evaluate for
cardiomegaly and colon polyps.

t) What is the preferred treatment in this patient?


I. The prefen-ed initial treatment is surgical removal of the tumor.
2. Bromocriptine or a somatostatin ( octreotide) analog may be useful as
medical adjuncts.
3. Radiation therapy may be indicated for the destruction of residual tumor.
Postoperative hormonal testing is indicated to reassess pituitary function.

146
In Capsule Series Smater not Harder

4- A woman aged 50 years old, her weight is 95 kg & her height is 160 cm. She
was discovered by chance to be diabetic.

a) What are the laboratory criteria for diagnosis of DM?


b) Discuss oral anti-diabetic drugs ?
c) Describe one method by which you can know if she is obese.
d) Enumerate the hazards of obesity.

The answer: Needless to say that she is type 2 DM

5- A 25 year old pregnant female developed severe post-partum hemorrhage


which was successfully controlled. The baby was artificially fed as his mother's
breast failed to secrete milk. The mother began to feel weak and she noticed
atrophy of her breasts.

a) What is the most probable diagnosis?


b) What is the expected BP of the patient ?
c) How to prove your diagnosis by investigations ?

Three drugs were prescribed after the results of investigations were known, but the
patient started only by one of them, in order not to distress her stomach. Next day
the patient complained of severe abdominal colic, vomiting and diarrhea. Her BP
was dropping rapidly and she became irritable and confused.

d) What is your diagnosis ?


e) How to manage the case ?

The answer:
a) Sheehan syndrome.
d) Acute adrenal failure.

147
In Capsule Series Smater not Harder

6- A 60 year old woman comes to the emergency room in a coma. The patient's
temperature is 35°C. She is bradycardiac. Her thyroid gland is enlarged. There is a
bilateral hyporeflexia.

a) What is the most probable diagnosis ?


b) What is the clinical signs and symptoms you should look for in this patient ?
c) What are the predisposing factors and what is the management of this case ?

a) Myxedema coma.
c) Predisposing factors : exposure to cold, infection, trauma, and CNS depressants.

7- A 42 years-old female presents with polyuria and polydepsia that proved to be


due to diabetes mellitus diagnosed for the first time. The physician noticed
enlargement of the hands and feet, deep voice, greasy skin, prognathism and
enlargement of the tongue.

a) What is the most probable diagnosis?


b) What are the investigations necessary to confim1 your diagnosis ?
c) What are the cardiovascular and neurological effects of the condition?
d) What are the endocrinal changes of the condition ?
e) What is the medical treatment of this patient ?

a) Acromegaly.
c) See endocrinology book
d) Endocrinal changes of acromegaly :
1-DM
2- Hyperprolactinemia : due to cosecretion from acidophils & pituitary stalk
compress10n.
3- Hypogonadism due to hyperprolactinemia
4- Deficiency of other anterior pituitary hormones due to compression.
5- Deficiency of ADH (DI) : rare

148
In Capsule Series Smater not Harder

MCQ

1- A 66 -year old man with type 2 diabetes notices painless skin lesions on his legs.
They have an irregular raised border with a flat depressed center that is
hyperpigmented brown in color. What is the most likely diagnosis?
a) N ecrobiosis lipoidica.
b) Pyoderma gangrenosa.
c) Erythema multiforme.
d) Candidiasis.

2- Which of the following best describes the effect of propylthiouracil on thyroid


hormone production?
a) It inhibits uptake of iodide by thyroid cells.
b) It blocks the release of hormones from the thyroid glands.
c) It prevents the release of thyroid hormone from thyroglobulin.
d) It blocks iodination and coupling of tyrosine.

3- A 53-year old woman with a past medical history of chronic kidney disease due
to diabetic nephropathy is noted to have hyperphosphatemia & hypocalcemia.
The disturbance is likely a result of metabolic bone disease seen in patients with
chronic kidney disease. Which of the following findings is most likely associated
with this electI·olyte disturbance?
a) Lethargy.
b) Neuromuscular irritability.
c) Anorexia, nausea and vomiting.
d) Tachyarrythmias.

4- Which of the following is the most common manifestation of multiple endocrine


neoplasia, type 1 (MEN 1)?
a) An adrenal adenoma.
b) Primary hyperparathyroidism.
c) Acromegaly.
d) Islet cell tumor of the pancreas.

5- A 7-year- old boy has demineralized bones with pseudofractures seen on x-rays.
Physiologic doses of vitamin D don't result in improvement. Which of the
following is most likely to be associated with the syndrome?
a) Hyperphosphatemia.
b) Low vitamin D levels.
c) Alopecia
d) Mental retardation.

About 50% of cases of vitamin D-resistant rickets have alopecia. It is afamilial disorder (X-linked
recessive disorder).

6- Hyperthyroidism can be treated by all but which one of the following?


a) Triiodothyronine.
b) Iodide.
c) Methimazole.
d) Propylthiouracil.
Notice that Triiodothyronine is T3, baby

149
In Capsule Series Smater not Harder

7- Which of the following is the most likely metabolic effect of insulin on adipose
tissue?
a) Decrease in lipolysis.
b) Decrease of glucose transport.
c) Decrease in lipoprotein lipase.
d) Decrease in glucose phosphorylation.

8-A 15-year- old youth has not gone through puberty. Which of the following is the
most likely diagnosis?
a) Inadequate diet.
b) Normal variation.
c) Pituitary tumor.
d) Drug side effects.

9- Insulin is contraindicated in :
a) Type1 DM
b) Complicated type 2 DM
c) DM with pregnancy.
d) Hypoglycemic coma.

10- Hypoglycemia is characterized by all the follm--ving EXCEPT


a) Blurring of vision.
b) Headache.
c) Dryskin
d) Pupillary dilatation.
11- Gynccomastia may be seen in all the following EXCEPT

a) Newborn infants
b) Klinefelter's syndrome
c) Hypopituitarism.
d) Turner's syndrome
12- Secretion of which of the following hormones does not primarily occur at night?
a) Insulin
b) Growth hormone
c) Melatonin
d) Prolactin
13- Earliest changes by ophthalmoscope in background retinopathy in diabetes
a) Venous dilatation
b) Microaneurysms
c) Increased capillary permeability
d) Arterio-venous shunts
14- Which is NOT a part of metabolic syndrome X
a) Hyperlipidemia
b) Obesity
c) lschemic heart disease
d) Hypertension
15- Thiazolidinedione group of antidiabetic is
a) Voglibose
b) Nateglinide
c) Kosiglitawne
d) Glimepiride
150
In Capsule Series Smater not Harder

16- All are features of diabetic ketoacidosis EXCEPT


a) Hyperthermia
b) Drowsiness
c) Dehydration
d) Air hunger
17- Commonest cause of coma in a diabetic is
a) DKA
b) Lactic acidosis
c) Hyperosmolar hyperglycemic non ketotic coma
d) Hypoglycemia

18- A patient of impaired fasting glucose ranges blood glucose value in between
a) 96 - 106 mg/ dL
b) 106- 116 mg/dL
c) 100 -125 mg/d.L
d) 116 - 130 mg/dL

19- Microalbuminuria is urinary albumin excretion ratio between


a) 10 - 20 µg/min
b) 20 - 200 µg/min
c) 30 - 300 µg/min
d) 40 - 400 µg/min

20- Myxedema coma is characterized by


a) Hypertension
b) Tachycardia
c) Euthermia
d) Hypoventilation
21- Thyrotoxicosis may be associated with all of the following EXCEPT
a) Irritability
b) Constipation
c) Exophthalmos
d) Tachycardia
22- Which cranial nerve is not involved in acromegaly
a) VIII
b) Ill, IV, VI
c) V
d) II
23- Pseudo-Cushing's syndrome may be found in all EXCEPT
a) Myxedema
b) Chronic alcoholism
c) Obesity
d) Depression.
24- Gynaecomastia may be produced after treatment with all EXCEPT
a) Spironolactone
b) Digitalis
c) Cimetidine
d) Rifampicin

151
In Capsule Series Smater not Harder

25- Pheochromocytoma is not associated with


a) Weight gain
b) Fear of death
c) Paroxysmal hypertension
d) Constipation
26- A 27 year old female presenting with diarrhea, sweating & palpitation with
diminished TSH & elevated T4. What is the next step in management?
a) Thyroglobulin
b) RAIUscan
c) Thyroid antibodies
d) Fine needle aspiration biopsy
Decreased TSH & elevated free T4 confirm a diagnosis of primary hyperthyroidism. The next best step is RAIU
scan. If the RAIU scan reveals diffuse iodine uptake, then it is Graves (if it is homogeneously diffuse) or toxic
multinodular (if it is heterogeneously diffuse).
27- Froehlich's syndrome is characterized by all EXCEPT
a) Infantilism
b) Truncal obesity
c) Diabetes mellitus
d) Mental retardation
28 - Which is not a part of multiple endocrine neoplasia type 1
a) Pheochromocytoma
b) Tumor of pituitary
c) Tumor of pancreas
d) Hyperparatbyroidism
29- Tertiary hyperparathyroidism is commonly found in
a) Rickets
b) Pseudohypoparatbyroidism
c) Chronic renal failure
d) Malabsorption syndrome
30- Commonest cause of thyrotoxicosis is
a) Multinodular goiter
b) Hashimoto's tbyroiditis
c) Grave's disease
d) Well differentiated carcinoma
31- Osmoreceptors are present in
a) Atria
b) Kidney
c) Anterior hypothalamus
d) Adrenal cortex
32- In Somogyi phenomenon commonly associated with type 2 DM, the dose of
insulin should be
a) Increased
b) Stopped
c) Decreased
d) Needs no change
33- Orlistat is used to treat
a) Diabetic nephropathy
b) Gout
c) Obesity
d) Anorexia nervosa.
152
In Capsule Series Smater not Harder

34- Commonest cause ofunilateraJ exophthalomos is:


a) Cavernous sinus thrombosis.
b) Retrobulbar tumor.
c) Chloroma.
d) Thyrotoxico sis.
35- Sleeping pulse rate is NOT increased in
a) Anxiety neurosis .
b) Rheumatic carditis.
c) TB
d) Atropinised patient.
36- Cardiovascular finding of thyrotoxicosis do not include
a) Loud S1
b) Means-Lerman scratch.
c) Water hammer pulse.
d ) Ejectio n click.
Means-Lerman scratch is an uncommon type of heart murmur which occurs in patients with hyperthyroidism. It is a
mid-systolic scratching sound best heard over the upper part of the sternwn or second left intercostal space at the
end of expiration. The murmur results from the rubbing of the pericardium against the pleura in the context of
hyperdynamic circulation and tachycardia, and may mimic the sound of a pericardia] rub.
37- Myxedema is characterized by all EXCEPT
a) Butterfly rash in face.
b) Sinus bradycardia.
c) Solid edema.
d) Madarosis.
Madarosis : hair loss of the eyebrows (superciliary madarosis) or loss of eyelashes (ciliary madarosis).
38-Acromegaly is associated with all of the following EXCEPT
a) Acanthosis nigricans
b) Fibromata mollusca
c) Micrognathia.
d) Cardiomegaly.
39- Klinefelter 's syndrome is characterized by:
a) Small, soft testes.
b) Chromosomal pattern 46, XO
c) Upper segment> lower segment of body.
d) Gyneco m astia.
40- Tall stature is NOT characteristic of
a) Klinefelter's syndrome.
b) Marfan's syndl·ome.
c) Homocysteinuria.
d) Turner's syndrome.
41- All of the following are featured by skin pigmentation EXCEPT
a) Conn's syndrome.
b) Bronchogenic carcinoma.
c) Addison's disease.
d) Hemochromatosis.
42- MedicaJ adrenaJectomy is done by all EXCEPT
a) Aminoglutethimide.
b) Mitotane.
c) Mexiletine.
d) Metyrapone.
153
In Capsule Series Smater not Harder

43- Sheehan's syndrome presents with


a) Cardiac failure.
b) Persistent lactation.
c) Fever
d) Cachexia.
44- Hypocalcemia is produced by all EXCEPT
a) Hysterical hypoventilation.
b) Acute pancreatitis.
c) Chronic renal failure.
d) Osteomalacia.

45- Thyrotoxicosis may be featured by all EXCEPT


a) Myopathy
b) Pretibial myxedema.
c) Hypernatremia.
d) Atrial fibrillation.
46- Hyperparathyroidism is NOT featured by
a) Acute pancreatitis.
b) Nephrocalcinosis.
c) Palpable neck swelling.
d) Pseudogout.
47-Malignant hypercalcemia is treated by all EXCEPT
a) Pamidronate.
b) Calcitonin
c) Calcitriol
d) Glucocorticoids.
Calcitriol (active vit D)
48- Commonest cause of Addison's disease is
a) Granuloma
b) Idiopathic atrophy
c) Inflammatory necrosis.
d) Malignancy
49- Moon face is characteristic of:
a) Addison's disease
b) Cushing' disease
c) Sjogren's syndrome
d) Simmond's disease
50- Features of Addison's disease do not include
a) Diarrhea
b) Dizziness
c) Dermatitis
d) Dehydration.

154
In Capsule Series Smater not Harder

51- Addison's disease is characterized by all of the following EXCEPT:


a) Skin pigmentation
b) Asthenia
c) Hypokalemia
d) Hypotension
52- Otitis fibrosa cystica is associated with
a) Addison's disease
b) Hypothyroidism
c) Hyperparathyroidism
d) Hyperthyroidism
53- All of the following are side effects of prolonged use of cortisone EXCEPT
a) Anemia
b) Growth retardation
c) Muscle wasting
d) Increased susceptibility to infection
54- Glycated fructosamine gives an indication of glycemia control for last
a) 1month
b) 3 months
c) 3 days
d) 14days
o Fructosamine levels typica]ly reflect albumin glycation. Because albumin has a half-life of 20 days, the
plmf!na fructosamine concentration reflects relatively recent (up to 2 weeks) changes in blood glucose.
o It is rarely used in clinical practice (hemoglobin A.le testing are preferred).
o Notice that glycated hemoglobin (HbAlc) measures average glucose levels over 3 months.
55- Confirmatory evidence of growth hormone deficiency is documented by
a) CTscan
b) Low IGF-1
c) Low growth hormone levels after provocative tests
d) Delayed bone age
56- Insulin requirements for diabetes are reduced in :
a) Infections
b) Exercise
c) Emotional stress
d) Hype1thyroidism
57-All are causes ofhyperprolactinemia except :
a) Bromocriptine
b) Phenothiazine
c) Methyldopa
d) Metoclopramide

155
In Capsule Series Smater not Harder

58-Which drug is essential in Sheehan's syndrome:


a) Estrogen
b) Cortisone
c) Thyroxin
d) Growth hormone
59- Addison's disease is characterized by following except-
a. Hyperkalemia
b. Hypotension
C. Hyponatremia
d. Hypocalcemia
60-All of the following statements about Hashimoto's disease are true except:
a) Many patients are entirely asymptomatic
b) Not all patients become hypothyroid
c) Most cases of obesity are attributable to Hashimoto's disease
d) Hypothyroidism may be subclinical
Although weight gain may be a symptom of Hashimoto's disease, the majority of obese people have normal thyroid
function.
61-The most common benign tumor of the pituitary gland is:
a) Glioma
b) Prolactinoma
c) Carcinoid tumor
d) Islet cell tumor
62- Thyroid acropachy is found in :
a) Subclinical hypothyroidism
b) Grave's disease
c) Myxedema
d) Medullary carcinoma of thyroid

63- Causes of short stature in childhood include EXCEPT :


a) Klinefelter's syndrome
b) Turner's syndrome
c) Cushing's syndrome
d) Primary hypothyroidism

64- Adverse effects of oral corticosteroid therapy include EXCEPT


a) Peptic ulcer
b) Hypertension
c) Avascula:r bone necrosis
d) Pseudo-gout
e) Insomnia
Cortisone sometimes used to treat severe pseudo-gout
156
In Capsule Series Smater not Harder

65- As regard to thyrotoxicosis, which is incorrect?


a) Grave's disease is the most common cause
b) Uninodular goiter is more common than multinodular goiter
c) Amiodarone therapy is occasionally responsible
d) The thyroid gland is diffusely hyperactive in Grave's disease
e) There is an increased prevalence of HlA DR3 in Grave's disease

66- Causes ofhypercalcemia include EXCEPT:


a) Carcinomas secreting PTH like peptides
b) Severe Addison's disease
c) Severe hypothyroidism
d) Chronic sarcoidosis

67- Calcium level in the blood is regulated by :


a) Parathyroid and thyroid
b) Adrenal medulla and pancreas
c) Testis
d) Parathyroid and thymus
68- In a case of Graves disease, Which of the following is a possible side effect of
RAJ treatment?
a) Agranulocytosis.
b) Recurrent laryngeal nerve injury
c) Worsening of Graves ophthalmopathy
d) Cholestasis
With RAI treatment, iodine concentrates in the follicular cells of the thyroid & ablates them completely within 2 to
4 months. It will not decrease circulating levels of TSH receptor antibodies, so it will not treat Graves
ophthalmopathy. Through an unclear mechanism, it can actually worsen ophthalmopathy. In order to prevent this,
some doctors give cortisone prior to RAI treatment.
69- Karyotype 47, XYY is
a) True hermaphroditism
b) Supermale
c) Klinefelter's syndrome.
d) Gonadal dysgenesis.
70- A 54 year old woman with a history of type 1 DM and autoimmune thyroid disease
presents with weakness, anorexia, nausea and vomiting. She has a BP of 82/64 mmHg and
physical exanrination reveals spotty hyperpigme ntation of her mucous membranes in her
oral cavity. Laboratory results reveal K of 6.1 mEq/L and elevated ACfH level. Which of the
following is the underlying diagnosis in this patient?
a) MEN l
b) MEN2
c) Polyglandular autoimmune syndrome type 1
d) Polyglandular autoimmune syndrome type 2
157
In Capsule Series Smater not Harder
71- As regard to thyroid nodule, which of the following features is NOT associated
with an increased risk of malignancy?

a) Hard and immobile mass


b) Cold nodule on RAIU
c) Cervical lymphadenopathy
d) Hot nodule on RAJU
72- Type of insulin best used for treatment of diabetic ketoacidosis
a) Crystalline insulin
b) NPH insulin
c) Lente insulin
d) A and b
e) Aandc
73- Symptoms of Grave's ophthalmopathy include all of the following except:
a) Bulging eyeballs
b) Dry, irritated eyes and puffy eyelids
c) Cataracts
d) Light sensitivity

74- Which one of the following inhibits growth hormone secretion from the
anterior pituitary gland?
a. Somatostatin.
b. GH releasing hormone.
c. Hypoglycemia.
d. Arginine.
e. Serotonin.
GHRH , Hypoglycemia , insulin , arginine , serotonin -+ t GH.

75- In hypertensive individual , which one ofthe following is the likely finding in a
patient with renal artery stenosis and is helpful in distinguishing the condition
from Conn's syndrome?
a. A high aldosterone level
b. A high renin and a low aldosterone level.
C. A high renin level
d. A low aldosterone level
e. A low renin and a high aldosterone.
Renal artery stenosis results in high renin and high aldosterone levels in contrast to Conn's syndrome in
which the aldosterone level is high but the renin is characteristically suppressed.
76- A 45-year-old obese man without known medical problems complains of
feeling very sleepy during the day and often falling asleep while listening to
friends. The most likely cause of this patient's problem is
a. narcolepsy
b. upper airway obstruction at night
c. glucocorticoid excess
d. growth hormone excess
e. estrogen excess
158
In Capsule Series Smater not Harder
77- Obese persons are at an increased risk for which of the following disorders?
a.
Hypothyroidism
b.
Cholelithiasis
c.
Type 1 diabetes mellitus
d.
Elevated levels of HDL cholesterol
78- Which of the following regimens is best for the preoperative management of a
patient with a known pheochromocytoma?
a. Propranolol alone
b . Propranolol followed by phenoxybenzamine
C. Phenoxybenzamine followed by propranolol
d. Prazosin alone
e. Propranolol followed by prazosin
79- Which of the following may be a direct consequence of severe magnesium
defciency?
a. Hypophosphatemia
b. Hypercalcemia
C. Hypokalemia
d. Hyponatremia
e. Shortening of the QT interval
So- A 55-year-old woman presents to her physician with mild fatigue. Her past
medical history is unremarkable. She is taking no medication. No abnormalities
are detected on physical examination. The only abnormality detected on routine
blood testing is an elevated calcium (11.9 mg/dL) and a serum inorganic
phosphorus of (2 mg/dL). An immunoreactive parathyroid hormone level is
undetectable. The most likely etiology for this patient's high serum calcium is
a. primary hyperparathyroidism
b. malignancy
C. hypervitaminosis
d. hype1thyroidism
e. familial hypocalciuric hypercalcemia
hypercalcemia and hypophosphatemia without elevated levels of PTH are likely to have the hypercalcemia of
malignancy. Patients with excessive levels of vitamin D would
not be expected to have a low serum phosphate. Second, patients with familial hypocalciuric hypercalcemia, an
autosomal dominant trait, often have nom1al or slightly low levels
of immunoreactive parathyroid hormone. It is now clearly recognized that many solid tumors, inclurung
carcinomas of the lung and kidney, may produce (PTH like) that will not be
identified by the currently available assays that detect true parathyroid hormone elaborated
from the parathyroid gland.

81- The most common presentation of primary hyperparathyroidism is


a. bone fracture
b. increased serum creatinine
c. osteitis fibrosa cystica
d. calcium kidney stones
e. asymptomatic hypercalcemia
82- Which ofthe following medications is lmown to cause hyperprolactinemia?
a. Metoclopramide
b. Levothyroxine
C. Glucocorticoids
d. Propanolol
e. Cigarette use
159
In Capsule Series Smater not Harder
83- A 23-year-old woman is diagnosed with Graves'disease shortly after
discovering she is pregnant. Appropriate therapy includes
a. radioactive iodine to ablate her thyroid gland
b. propylthiouracil with the goal of maintaining her thyroid function tests in slightly high range.
C. methimazole therapy
d. a beta blocker
e. propylthiouracil with care taken to maintain her thyroid function tests in the mid normal
range
Radioactive iodine should never be given to a pregnant woman. In addition, both methimazole and beta blockers
should be avoided in pregnant women. Antithyroid drugs, including propylthiouracil, cross the placenta and affect
fetal thyroid function Studies have shown that when a treated pregnant woman's thyroid function is in the mid-
normal range, the fetus is hypothyroid. When the mother's thyroid tests are maintained in the high-normal or
slightly hyperthyroid range, the fetus is likely to have normal thyroid function.

84- A 19-year-old man with type 1 diabetes mellitus presents to the emergency
room with nausea and vomiting. His arterial pH is 7.16 with potassium of 5.4
mEq/L, bicarbonate of 7 mEq/L, sodium of 132 mEq/L, phosphate of 3.0 mg/dL,
and glucose of 475 mg/dL. In addition to intravenous saline & insulin, which
electrolyte should be considered in treatment of this case?
a. Bicarbonate
b. Potassium
C. Dextrose
d. Phosphate
e. None of the above
The mainstay of therapy for diabetic ketoacidosis (DKA) is insulin and intravenous fluids.
DKA cannot be reversed without insulin. Although the serum K concentration is high,. The K
concentration will drop quickly due to insulin therapy (due to intracellular shift) .
Because glucose levels drop more quickly than ketones disappear from the plasma, it is usually
necessary to give intravenous dextrose when the blood glucose level drops below about (250 to
300 mg/dL). This allows continued administration of insulin to clear the ketones from the
blood. Bicarbonate therapy is not recommended unless the arterial pH falls below 7. 10 or 7 .00
because the rapid alkalinization may impair oxygen delivery to tissues and impair left
ventricular function. In addition, insulin therapy is effective in reversing the acidemia without
the assistance of bicarbonate therapy.
85- The diagnosis of diabetes mellitus is certain in which of the following
situations?
a. Abnormal oral glucose tolerance in a 24-year-old woman who has been dieting
b. Successive fasting plasma glucose 147, 165, and 152 mg/dl in an asymptomatic woman
C. A serum glucose level 100 mg/dL in a woman in her twenty-fifth week of gestation
after a 50 g oral glucose load
d. Persistent asymptomatic glycosuria in a 30-year-old woman
e. Persistently elevated nonfasting serum glucose levels
Persistent fasting hyperglycemia even if it is asymptomatic, has been recommended by the National Diabetes Data
Group as a criterion for the diagnosis of diabetes.
Abnormal glucose tolerance- whether after eating or after a standard uglucose tolerance test"-can be caused by
many factors (e.g., anxiety, infection or other illness) Similarly, glycosuria may have renal as well as endocrinologic
causes. Therefore, these two conditions cannot be considered diagnostic of diabetes. Gestational diabetes is
dfagnosed in women between the 24 - 28 weeks of gestation, first using a 50-g oral glucose load, if the 1-h glucose
level 140 mg/dL; a 100-g oral glucose test is performed after an overnight fast. Gestational diabetes is initially
treated with dieta.r y measures; if the postprandial glucose level remains elevated, insulin therapy is often started.
About 30% of women with gestational diabetes will eventually develop true OM.

160
In Capsule Series Smater not Harder

86- A 46-year-old woman arrives in your clinic for routine examination. She has
no specific complaints, and a full review of systems is unrevealing. On physical
examination she has normal vital signs and a 1.5 cm thyroid nodule is palpated in
the right lobe of her thyroid; there
are no other abnormal findings. A laboratory test reveals a serum TSH level of 2 .3
m U /L. Which of the following would be the most appropriate recommendation?
a. Fine-needle aspiration biopsy
b. Unilateral thyroid lobectomy
c. Thyroxine suppressive therapy
d . Radioiodine therapy
e. No intervention needed; a wait and watch approach is recommended
87- Which ofthe following has been associated with an effective approach
towards the prevention of diabetic retinopathy?
a. A reduction in the serum tliglyceride level
b. Improved control of blood glucose concentrations
c. Use of an ACE inhibitor
d. Use of aspirin therapy
e. Smoking cessation

88- Which of the following medications is known to cause hypoglycemia?


a. Acetaminophen
b. Pentamidine
C. Epinephrine
d. Verapamil
e. Thiazides
Insulin, sulfonylureas, disopyramide, and pentamidine (Antiprotozoal) all cause hypoglycemia through
hyperinsulinemia. Thiazides can cause hyperglycemia. Acetaminophen when taken in normal dosages does not affect
the serum glucose concentration; however, an overdose causing hepatic damage could lead to severe hypoglycemia.
89- Causes of fasting hypoglycemia that are due primarily to overutilization of
glucose include
a. acromegaly
b. hepatoma
C. alcohol ingestion
d. congestive heait failure from cor pulmonale
e. Hypopituitarism
90- Manifestations of hypothyroidism include
a. prolongation of the QT interval
b. depressed serum cholesterol
c. increased serum creatine phosphokinase
d. decreased serum lactate dehydrogenase level
91- Which of the following is most sensitive for detecting diabetic nephropathy?
a. Serum creatinine level
b. Creatinine clearance
C. Urine albumin
d. Glucose tolerance test
e. Ultrasonography

161
In Capsule Series Smater not Harder
92- Which of the following statements regarding adrenal insufficiency is true?
a. The most common cause of adrenal insufficiency is tuberculosis
b. The critical test for the diagnosis of chronic adrenal insufficiency is Synthetic ACTH test
C. ACTH levels greater than normal in seconda1y adrenal insufficiency.
d. 2ry adrenal insufficiency is characterized by skin pigmentation.
93- Which of the following statements regarding hyperaldosteronism is true?
a. The most common causes of secondary hyperaldosteronism are congestive heart failure and
cirrhosis w ith ascites
b. Treatment of primary adrenal hyperaldosteronism is spironolactone
C. Patients with primary adrenal hyperaldosteronism usually present with hypertension,
hypokalemia, and metabolic acidosis
d. Diagnosis of primary adrenal hyperaldosteronism is confirmed by elevations in the levels of
both renin and aldosterone
e. Patients with primary adrenal hyperaldosteronism usually present with generalized edema.
Primary byperaldosteronism is caused by benign adrenal adenomas, which are typically unilateral and are usually less
than 2.5 cm in diameter. Patients present with mild hypertension without edema due to renal escape phenomenon.
Laboratory testing shows hypernatremia, hypokalemia and metabolic alkalosis not acidosis. Diagnosis is confirmed by
elevated plasma aldosterone levels(> 14 ng/ dl) & suppressed plasma renin activity .The treatment of primary
hyperaldosteronism is unilateral adrenalectomy. preferably by a laparoscopic procedure. Secondary
hyperaldosteronism may or may not be associated with hypertension.

94- A 42-year-old man with long-standing type 1 diabetes presents with


gastroenteritis that has been worsening for 5 days. His blood pressure is 90/55
mm Hg, and his heart rate is 135 beats/min. Other laboratory findings are as
follows: blood glucose: 656 mg/dl; sodium: 127 mEq/L; potassium, 4.2 mEq/L;
HCO3 : 14 mEq/L; anion gap,25; and pH, 7.05. Which of the following would not be
an appropriate step in the immediate treatment of this patient?
a. I.V. administration of 0.9% saline
b. Admission to the intensive care unit
C. I.V. administration of potassium chloride
d. I.V. administration of insulin
e. I.V. administration of sodium bicarbonate
Administration of bicarbonate is not generally required in most cases of DKA and is generally reserved for
treatment of severe acidosis (pl-I < 7.0 or HC03 < 10)

95- As regard to secondary diabE>tes, which is correct ?


a. A patient can be assumed not to be ketosis-prone
b. Most patients with panbypopituitarism are diabetic
C. Classical diabetic complications do not occur
d. Thiazide diuretics and beta-blockers can both impair insulin secretion
e. Most patients with acromegaly are diabetic
Secondary diabetes causes all the same complications as primary diabetes, including DKA.
15% of patients with acromegaly are diabetic.

96- The physiological effects of insulin include ......


a. increased glycolysis
b. increased glycogenolysis
C. increased lipolysis
d. increased gluconeogenesis
e. increased protein catabolism
162
In Capsule Series Smater not Harder

9 7- The most likely etiology for the eating disorder anorexia nervosa is
a. decreased levels ofluteinizing hormone- releasing hormone (LHRH)
b. decreased levels of growth hormone
C. decreased levels of insulin-like growth factor I
d. low levels of serum thyroxine
e. psychiatric disorder
98- Which of the following is most likely to be associated with increased insulin
sensitivity ?
a. Acromegaly
b. Smoking cessation
C. Pheochromocytoma
d. Polycystic ovary disease
e. Weight loss
99- In metabolic diabetic ketoacidosis ( DKA) , the treatment of choice is :
a . Normal saline, IV insulin, metformin.
b. Normal saline , IV insulin , potassium
C. Normal saline , insulin , heparin
d. Normal saline , IV insulin , cortisone
e. IV 5% glucose.
100- In Cushing syndrome which of the following is true?
a. Esinopenia
b. Basophilia
C. Esinophilia
d. Lymphocytosis
e. Neutropenia
101- Syndrome of inappropriate ADH secretion may result from EXCEPT:
a. Meningitis
b. Bronchial carcinoma
C. Digoxin therapy
d . Head injury
e. Pneumonia
102- Causes of DI include the following EXCEPT :
a. Congenital disorder
b. Craniopharyngioma
C. DIDMOAD syndrome
d. Severe hypocalcemia
e. Sarcoidosis
103- Which of the following test would most reliably defect a deficiency of the GH
axis in a 19 year-old man ?
a . ACTH stimulation test
b. Arginine test
C. Glucose test
d . Insulin tolerance test
e. TRH administration test
163
In Capsule Series Smater not Harder
104- Causes of nephrogenic DI include the following EXCEPT :
a. Lithium
b. Hypokalemia
C. Congenital disorder
d. Chlorpropamide & carbemazepine therapy
e. Renal amyloidosis
105- A 20 year old lady with a six month history of sweating and diarrhea . She is
otherwise well. Her father and grandmother died in middle age but she is unsure
of the reason why. On examination, no abnormality could be found. Which one of
the following diagnoses would it be important to exclude?
a. Cushing's disease
b. Grave's disease
C. Acromegaly
d. Medullary thyroid carcinoma
Medullary thyroid carcinoma (MTC) can secrete a variety of peptides and prostaglandins, resulting in extra-thyroid
symptoms. Diarrhea and sweating are the most common. It is an important tumor to exclude since it can
metastasise early. MTC can be inherited as part of MEN type 2.
Acromegaly can also cause sweating due to sweat gland hypertrophy, but usually not diarrhea. Cushing's disease
also rarely presents with these symptoms. Both these conditions also rarely show such strong family history

106- A patient comes in for a fasting plasma glucose test .On two separate occasion
, the result has been 115 and 120 mg/dl which of the following is the most
appropriate next step ?
a. Reasurance that these are normal blood sugar
b. Recommend weight loss and exercise.
C. Diagnose DM and start a s ulfonylurea agent
d. Recommend cardiac stress test
e. Hospitalize him urgency
107- A 45 year-old obese woman presents for follow up of her diabetes. She
currently take glipizide ( sulfonylurea) 10 mg twice per day, and her fasting
morning glucose is 180 mg/dl. Her last HbAtc was 7.9 . She states that she
conscientiously follows her diet and that she walks 45 minutes daily. "What is the
best next step in her care ?
a. Add an insulin pump
b. Add metformin
C. Hospitalize her urgency
d. Add antibiotic
108- The finding of reduced free T4 & TSH is compatible with :
a. Hypopituitarism
b. 1ry hypothyroidism
C. Nephrotic syndrome
d. 2ry hyperthyroidism
109- A 33-year old woman is noted to have Grave's disease .Which of the following
is the best therapy ?
a. Long term propranolol.
b. Lifelong oral propylthiouracil ( PTU )
C. Radioactive iodine ablation
d. Surgical thyroidectomy
It is the definitive treatment for Grave's disease. Surgery is indicated for obstructive symptoms or for women
during pregnancy.
164
In Capsule Series Smater not Harder
110- In try hyperaldosteronism ( Conn's syndrome) ...
a. peripheral edema is usually present
b. proximal myopathy is due to hypokalemia
C. severe hypertension is characteristic
d. DM is often present
e. hypertension is associated with hyperreninemia

Now, and after your journey with Endocrinology ... REMEMBER that ..

0 0
Everyone likes be1· to be Hot & Cystic ... NOT ... Cold & Solid

I'm talking about thyroid nodule ....


- Hot nodule by thyroid scan & cystic ,y US : benign nodule.
- Cold nodule: 15% malignant.

165
AUTHOR'S AVAILABLE BOOKS:

► Cardiology
► Neurology
► Nephrology
► Gastroenterology
► Endocrinology
► Pulmonology
► Hematology
► Rheumatology

COMING SOON............

► Clinical medicine
► ECG
► Mowafy Medical symphony

© All rights reserved. This book is protected by copyright. No part


of it may be reproduced,stored in a retrieval system,or transmitted
in any form by any means, electronic,mechanical, photocopying,
recording or otherwise,without written permission from the author.
SMARTER NOT HARDER

"Addiction-Free Nation Program shoud consider lncapsule Series it's 1st priority, as it has been
proved that all Internal Medicine seekers are addicted to it"
"Studying Medicine without teacher is like salllng without a boat, and studying Internal Medicine
without Dr. Ahmed Mowafy is like not going to the sea at all"
Dr. Alsayed Dawoud SERIES
Kasr-Alainy school of Medicine
"Ultimate Medicine experience, Internal Med icine Impulse t hrough the Mind".
Dr. Mohamad Rashed
Medical student, Artist

DESIGNED BY
University Book Cen tr
· Sayed M ahmou
103 M athaf El Manial St. - Ca1r
02-25329005 • 0101111 7938 • 0115 720 69 45

You might also like